MDCAT Past Papers

MDCAT Past Papers

PUNJAB MDCAT SOLVED PAST PAPER-2019

PUNJAB MDCAT SOLVED PAST PAPER-2019

For F.S.C. and Non-F.Sc. Students 

Time Allowed: 210 Minutes (3- ½ hours)

Instructions: 

i. Read the instructions on the MCQs Response Form carefully. 

ii. Choose the Single Best Answer for each question.

iii. Each correct Answer carries One Mark. There is No Negative Marking  

iv. Candidates are strictly prohibited from giving any identification mark except Roll. No. & Signature in the specified columns only. 

BIOLOGY

1. In chemiosmosis the proton (H+) pumps moves from ________ 

A. Stroma to Lumen

B. Stroma to cytoplasm

C. Lumen to Stroma

D. Cytoplasm to Stroma

Explanation:  In chemiosmosis, the protons move from higher concentration to lower concentration region. They move from lumen to the stroma through thylakoid membranes. 

2. Microtubule subunits (for spindle fibers) are synthesized in ________ phase.

A. G2

B. M

C. S

D. G1 

Explanation: G2 is the phase of rapid growth. Cell prepares for mitosis. This phase has double the number of chromosomes. Energy required for the movement of chromosomes is accumulated during this phase.  Microtubule subunits are synthesized.

3. If stimulation is above _________, impulses travel to the brain along the sensory neuron.

A. Action Potential

B. Threshold

C. Resting Potential

D. Recovery Period

Explanation: When a neuron receives sufficient stimulation to reach the membrane threshold, successive sodium gates along the entire neuron membrane open. The opening of the sodium gates allows sodium ions to move into the neuron

4. Substances responsible for increasing the set point of the hypothalamus are called:

A. Pepsin

B. Pyrogens

C. Prions

D. Androgens

Explanation: Pyrogen is the substance produced by bacterium, it is responsible for displacing set point of hypothalamus above normal body temperature.

5. During inspiration the space inside the chest cavity is increased due to.

A. Increased pressure

B. The relaxation of the muscles of the diaphragm

C. Relaxation of the external intercostal muscle

D. The contraction of the muscles of the diaphragm

Explanation: During inhalation, diaphragm contracts and becomes less dome shaped. The space inside chest cavity increases and lungs have more space to expand by inhaling the air. 

During exhalation, chest cavity space decreases due to relaxation of diaphragm. 

6. Which of the following hormone stimulates the ovulation from the follicle into oviduct?

A. Luteinizing hormone

B. Follicle stimulating hormone

C. Estrogen

D. Progesterone

Explanation: LH is produced by anterior pituitary. It has a role in ovulation. FSH induces follicle growth. Progesterone is pregnancy maintaining hormone whereas, estrogen is responsible for development of secondary sex characters.

7. The covalent bond or bridge between two monosaccharides to form a disaccharide is called a:

A. Carboxyl bond

B. Hydroxyl bond

C. Hydrogen bond

D. Glycoside bond

Explanation: Glycosidic bond forms between 2 sugar molecules. Water is removed. 

8. Site of protein synthesis is:

A. Ribosomes

B. Lysosomes

C. Golgi body

D. Cisternae

Explanation: Ribosomes are the organelles where proteins are synthesized. These are attached to endoplasmic reticulum. The prepared protein leaves ribosomes and enter golgi bodies where it is packaged to be transported outside of the cell. 

9. Water and minerals move down their concentration gradient through plasmodesmata, to cells of cortex, endodermis, pericycle and the not sap in the xylem cells. This is also known as the

A. Symplastic pathway

B. Mineral absorption pathway

C. Vacuolar pathway

D. Apoplastic pathway

Explanation: Symplast is the system of interconnected protoplast in the root cells. The neighboring cells are connected through plasmodesmata. It is the cytoplasmic projections that extend through pores in adjacent cell walls.

Mostly sugar travels through this pathway.

10. Given below is the diagram of nephron without vascular supply.

What is name of Part C?

A. Collecting tubule

B. Proximal tubule

C. Distal tubule

D. Loop of Henle

Explanation:

11. Xerophytes have small thick leaves to:

A. Help them float on water

B. Help them survive in salty environment

C. Limit water loss by reducing the surface area

D. Limit water loss by increasing the surface area

Explanation: Xerophytes are plants which grow in regions where there is little liquid water. They have adaptations to reduce the rate of transpiration to conserve water. Less surface area means less water loss through transpiration. 

12. Among followings which cellular organelle contains circular DNA similar to those found in bacteria?

A. Ribosome

B. Lysosome

C. Chloroplast

D. Nucleus

Explanation: Mitochondria and chloroplasts have their own DNA, which they use to produce many proteins and enzymes required for their function.

13. A person was married to his cousin and both are heterozygous for sickle cell anemia. Among their four kids, what will be proportion of affected homozygotes?

A. 50%

B. 25%

C. 75%

D. 100%

Explanation: Parent 1 = heterozygous anemic Ss

Parent 2 = heterozygous anemic Ss

       S

s

 

  S

 

SS

 

Ss

 

s

 

Ss

 

ss

Only ss is affected homozygote i.e. 1:4 or 25%. 

14. The route of urine excretion from kidney to outside of body is:

A. Kidney – ureter – urinary bladder – urethra 

B. Urinary bladder – kidney – ureter – urethra 

C. Kidney – ureter – urethra – urinary bladder

D. Kidney – urethra – urinary bladder – ureter 

Explanation:

15. The phase of mitosis is which sister chromatids move towards opposite poles:

A. Prophase

B. Anaphase

C. Telophase

D. Metaphase

Explanation:

Prophase 

Metaphase 

Anaphase 

Telophase 

Condensation of chromosomes occurs

Chromosomes become visible

Breakdown of nuclear envelope

Centrosomes start moving towards the opposite poles

Kinetochores have appeared at the centromeres

Spindle apparatus has formed, spindle fibers are now originating from the opposite poles

Chromosomes line up at the metaphase plate

Each sister chromatid attaches to a spindle fiber through centromere

Centromeres split in two and each sister chromatid is pulled towards opposite pole

The cell begins to elongate 

Sister chromatids (now called chromosomes) reach the opposite poles

Chromosomes de-condense

Nuclear envelope appears again. This time 2 nuclear envelops appear surrounding each set of chromosomes

Mitotic spindles break down and continue to push apart towards opposite poles

16. The Plasmid pBR322 has antibiotic resistance genes for:

A. Tetracycline and Doxycycline

B. Streptomycin

C. Doxycycline and Ampicillin

D. Ampicillin and Tetracycline

Explanation: Plasmid pBR 322 has antibiotic resistance gene for tetracycline and ampicillin

Plasmid pSC 101 has antibiotic resistance gene for tetracycline

17. The nitrogen containing bases in nucleotide are of two types; Purines and Pyrimidines; the purines bases are:

A. Guanine and Cytosine

B. Adenine, Guanine and Cytosine

C. Adenine and Guanine

D. Adenine and Thymine

Explanation: Adenine (A) and Guanine (G) are larger and are called purines. Cytosine (C) and Thymine (T) are smaller and are called pyrimidines.

18. The following flowchart depicts the steps of the Calvin Cycle. Which option according to you fits in as the correct answer of the missing step?

A. Hydrogenase

B. Oxaloacetate

C. Ribulose bisphosphate

D. Pyruvate

Explanation: Calvin cycle or dark reaction starts with carbon dioxide fixation. 

During this step, 3 molecules of carbon dioxide react with 3 molecules of ribulose bisphosphate (RUBP) to produce 6 molecules of 3-phosphoglycerate. This step is catalyzed by ribulose bisphosphate carboxylase or rubisco.

Next step is reduction. Six molecules of 3-phosphoglycerate utilize phosphate groups from six ATPs to produce six molecules of 1,3-bisphosphoglycerate. 

Here the reduction takes place utilizing NADPH as reducing agent to reduce 1,3-bisphosphoglycerate to glyceraldehyde -3- phosphate. 

One molecule of glyceraldehyde -3- phosphate produce one molecule of sugar. Other 5 molecules are generated to produce 3 molecules of RUBP. 

Last step is regeneration of RUBP. Five molecules of glyceraldehyde -3- phosphate rearranged to produce 3 molecules of ribulose phosphate which are phosphorylated to give 3 molecules of RUBP. 

Cycle is completed and RUBP is ready to accept more carbon dioxide.

19. DNA polymerase enzyme for PCR is Isolated from bacteria Thermus aquaticus because;

A. It can withstand high denaturation temperature.

B. It can withstand low denaturation temperature.

C. It can work at high speed

D. It can be used again and again.

Explanation: Taq polymerase is a DNA polymerase enzyme which is thermostable or temperature insensitive. it is extracted from a bacteria, Thermus aquaticus, which lives in hot springs.

20. The type of energy reduced by the enzymes for biological reactions to occur is called the:

A. Light Energy

B. Activation energy

C. Active energy

D. Heat energy

Explanation: Activation energy needs to be decreased in order to complete the reaction fast which is the property of an enzyme. Lower the activation energy, more readily the reaction is completed.

21. The function of calcium ions in muscle contraction is to:

A. Bind to troponin molecule and cause them to move

B. Aid in the transmission of nerve impulse

C. Polarize risible light

D. Bind to tropomyosin molecule and cause them to form cross bridges

Explanation: Troponin is attached to the protein tropomyosin and lies within the groove between actin filaments in muscle tissue. 

In a relaxed muscle, tropomyosin blocks the attachment site for the myosin crossbridge, thus preventing contraction. 

Upon nervous system excitation for a contraction, calcium is released into the fiber. It binds with troponin and stimulates it to move tropomyosin, exposing the binding site for the myosin crossbridge. Because of the presence of calcium, your muscle can now contract.

22. What is common in both Competitive and Non- Competitive Inhibition.

A. Irreversible Inhibition

B. Feedback Inhibition

C. Reversible Inhibition

D. Non- Reversible Inhibition

Both of these types of enzyme inhibitions are reversible.

Explanation: Competitive inhibitors have structure similar to the substrate and binds to the active or catalytic site of the enzyme preventing the substrate to bind there.

Non- competitive enzymes attach to the enzyme other than the active site and alter the shape of the enzyme such that the substrate is unable to attach at the active site.

23. Inside ovary, primary oocyte divides through first meiotic division terming two haploid cells, secondary oocyte and:

A. Polar body

B. Oogonium

C. Follicle cell 

D. Ovum

Explanation: oogonia in germ cells – mitosis – primary oocytes (enclosed in group of follicle cells)

Primary oocyte – meisos – haploid secondary oocyte + first polar body

24. Homozygous means:

A. having two identical alleles of a gene.

B. having two identical genes

C. alleles in an organism

D. two different alleles of a gene.

25. Which hormonal pair would maintain the endometrium and make it receptive for implantation of embryo?

A. Luteinising Hormone and Progesterone

B. Estrogen and Follicle Stimulating Hormone

C. Luteinising Hormone and Follicle stimulating Hormone

D. Estrogen and progesterone

Explanation: Estrogen is produced under the stimulus of FSH. It stimulates, vascularizes the endometrium and also inhibits secretion of FSH from pituitary gland. 

Follicle cells after releasing egg convert into yellow structure called corpus luteum which produces progesterone. It develops endometrium and makes it receptive for the implantation of zygote.

26. Starch is present in tubers, fruits and grains but absent in animal cells, instead animals have a substance stored in liver and muscles known as:

A. Glucose

B. Glycogen

C. Galactose

D. Glucagon

Explanation: Starch is the carbohydrate source produced by plants whereas, glycogen is produced by animals only. 

27. Now-a-days every new born gets regular shots of vaccine for polio. It contains _______ for polio to make a child immune against this disease.

A. Antisera

B. Antibodies

C. Antibiotics

D. Antigen

Explanation: Polio vaccination contains antigens of poliovirus. Antigens for the disease are artificially introduced in the body and antibodies are produced in response and the baby develops immunity. 

28. Which of the following blood vessels contain semilunar valves?

A. Arteries

B. Veins

C. Arterioles

D. Capillaries

Explanation: Semilunar valves are present between aorta and left ventricle and between right ventricle and pulmonary artery. It inhibits back flow of blood from arteries to ventricle. 

29. The main neurotransmitter for synapses is ________ which lie outside the central nervous system.

A. Choline

B. Acetylcholine

C. Acetaldehyde

D. Phosphatidylcholine

Explanation: Acetylcholine is released in the synapsis and travels to bind to the receptor on the other side of the synapse. It is also present in peripheral nervous system. 

30. The thick filaments in a myofibril of muscles are made of _______.

A. Hemoglobin

B. Myoglobin

C. Actin

D. Myosin

Explanation: Thick filament is composed of myosin. Each myosin molecule has tail of 2 long polypeptide chains terminating in 2 globular heads. Heads are called cross bridges because they link thick and thin myofilaments together during contraction.

31. The prokaryotes possess small ribosome of size:

A. 40S

B. 70S

C. 65S

D. 60S

Explanation: Fact. Eukaryotic cell has larger ribosome subunits,

32. The structure present in a eukaryotic cell but absent in prokaryotic cells is

A. Nucleus

B. DNA

C. Ribosomes

D. Cell surface membrane

Explanation: Major difference between prokaryotic cell and eukaryotic cell. 

33. The process in which a complementary copy of the code from a gene is produced by RNA Polymerase in the nucleus:

A. Proof reading

B. DNA Replication

C. Transcription

D. Translation

Explanation: Definition of transcription. mRNA is formed in this process. 

34. Sara is a chemistry student who is carrying out an experiment between an alcohol and acetic acid in the laboratory. The product formed at the end of the experiment will be:

A. Glucose and oxygen

B. Glycogen and water molecule

C. An ester and water molecule

D. Glycerol and Sulfuric acid

Explanation: An alcohol and a carboxylic acid condense to form an ester and water molecule.

35. In genetics, the term locus refers to the _________ of the gene on the chromosome.

A. Frequency

B. Copy

C. Position

D. Inversion

Explanation: Definition of locus

36. If 15 µm size object is observed under light microscope using 5X eyepieces and 10X objective, its magnified image size will be:

A. 250 µm

B. 50 µm

C. 750 µm

D. 500 µm

Explanation: Magnification = power of eyepiece x power of objective lens = 50X

Now multiply this by the size of object under examination,

15 µm x 50 = 750 µm

37. Which enzyme Is administered to the patients of Severe Combined Immunodeficiency Disease (SCID)?

A. Pancreatic Enzyme

B. Adenosine Deaminase (ADA)

C. β-galactosidase

D. β-lactamase

Explanation: ADA deficiency affects the immune system and weakens the ability to fight off infections. SCID is one of the outcomes of ADA deficiency. To treat it, ADA is given. 

38. Change in frequency of alleles that occurs by chance is called as:

A. Natural selection

B. Migration

C. Mutation

D. Genetic drift

Explanation: Definition. Frequency changes due to random events. 

39. The finger like infoldings which are formed by inner membrane of mitochondria are called:

A. Matrix

B. Porin

C. Cristae

D. Ribosomes

Explanation: Cristae are partial partitions in a mitochondrion formed by infolding of the inner membrane.

40. The main nitrogenous excretory product of humans is:

A. Uric acid

B. Ammonia

C. Urea

D. Ammonium

Explanation: Humans get rid of poisonous nitrogen in the form of urea. Birds excrete uric acid while fish excrete ammonia. 

41. Smooth endoplasmic reticulum is responsible for the metabolism of

A. Carbohydrates

B. Proteins

C. Nucleic acids

D. Lipids

Explanation: Smooth ER are involved in the synthesis of lipids and steroids, such as cholesterol and reproductive hormones estrogen and testosterone. 

42. Acetylcholine and Noradrenaline are two types of _________ used in our nervous system.

A. Hormones

B. Channel and carrier proteins in the cell membrane of a Neuron

C. Enzymes

D. Neurotransmitters

Explanation: Neurotransmitters aid in transfer of nerve impulse.

43. The reflex action is the phenomena which only involves:

A. brain, receptors, spinal cord

B. receptors, effectors and spinal cord

C. receptors, neurons, brain

D. receptors and effectors

Explanation: Reflex action is an involuntary action. The sensory receptors send impulses to the spinal cord and motor neurons carry impulses to the effector organ either it be some skeletal muscle or a gland. 

44. In which situation. Genes are not assorted independently during Meiosis in a chromosome?

A. When genes are not linked and their loci are far apart.

B. When there are too many Genes on a chromosome.

C. When some genes have mutated on the chromosome.

D. When genes are linked and their loci are close to each other.

Explanation: When genes are linked, they are close to each other or on the same chromosome, they do not assort independently. 

45. A person got an infection, he became ill but then he survived. What do you think which type of immunity he would have developed?

A. Naturally induced active immunity

B. Artificially induced active immunity

C. Active immunity

D. Passive immunity

Explanation: Active immunity can be achieved in 2 ways.

  • Artificially induced active immunity: antigens for the disease are artificially introduced in the body
  • Naturally induced active immunity: the person is exposed to an antigen, becomes ill and survives, immunity is developed against the disease.

46. During spermatogenesis, the _________ which are haploid cells eventually mature into spermatozoa/mature sperms:

A. Secondary spermatocytes

B. Primary spermatocytes

C. Spermatogonia

D. Spermatids

Explanation: Spermatogonia mature into primary spermatocytes which divide meiotically to produce secondary spermatocytes. These differentiate into sperms.

47. Transgenic mice have been used to produce:

A. Protein rich milk

B. A growth hormone

C. Protein rich meat

D. Extra hair

Explanation: Transgenic mice were found to have hair patches on bald skin. 

48. According to the theory of natural selection, organisms produce:

A. More offspring than supported

B. Less offspring than supported

C. Offspring according to the resources available

D. Offspring to create resources

49. The photsynthetic pigments of plants are arranged as clusters in thylakoid membranes. The reaction centers of these clusters consist of _________ molecules

A. ATP

B. Chlorophyll

C. Glucose

D. Carotenoids

50. Meselson and Stahl transferred few bacteria grown in N15 medium to N14 medium for replicating their DNA. What would be the result after two rounds of replication?

A. 50% hybrid duplex and 50% light duplex

B. 50% hybrid duplex and 50% heavy duplex

C. 100% heavy duplex

D. 100% hybrid duplex

Explanation: Meselson and Stahl inferred from the experiment that DNA strands show semi conservative replication. One strand of every DNA was N15 labelled and one was N14 labelled. 

51. In an action potential, the permeability of sodium ions in the neurons increases due to:

A. Repolarization

B. The opening of sodium channels/gates

C. The action of the acetylcholinesterase enzyme

D. Sodium ions forming an ionic bonding

Explanation: When a neuron receives sufficient stimulation to reach the membrane threshold, successive sodium gates along the entire neuron membrane open. The opening of the sodium gates allows sodium ions to move into the neuron

The movement of sodium ions into the neuron causes the membrane potential to change from-70mV to +40mV. As the membrane potential becomes more positive, sodium gates begin to close.

52. During the G2 phase:

A. Specific enzymes are synthesized and DNA base units are accumulated

B. The chromosomes are left with only one chromatid

C. Chromosome number is duplicated

D. Energy is stored for Chromosome movement and mitotic specific proteins (Tubulin) are produced

Explanation: It is the phase of rapid growth. Cell prepares for mitosis. This phase has double the number of chromosomes. Energy required for the movement of chromosomes is accumulated during this phase.  

53. Thin filaments of muscles contain ________ chain of actin molecules.

A. Four

B. One

C. Three

D. Two

Explanation: Thin filament is actually three polypeptide complex. One binds to actin. Another binds to tropomyosin and third binds to calcium ions. The major protein in thin filament is troponin. They are 7 to 8 nm thick and are chiefly composed of actin molecules. 

54. Which cell organelle is responsible for cell secretion?

A. Mitochondrion

B. Chloroplast

C. Ribosome

D. Golgi body

Explanation: Golgi bodies are responsible for packaging and release of cell secretions. 

55. Most proteins are made up of:

A. 16 types of Amino acids

B. 10 types of Amino acids

C. 170 types of Amino acids

D. 20 types of Amino acids

Explanation: There are 20 types of amino acids that are components of proteins present in the human body. 

56. A disease caused by gradual breakdown of the thin walls of alveoli is _______.

A. Tuberculosis

B. Asthma

C. Emphysema

D. Bronchitis

Explanation: Emphysema is the degenerative disease. In patients with emphysema, alveolar walls degenerate, and small alveoli combine to form larger alveoli. As a result, there is an increased volume and decreased surface area of alveoli.

57. If sequence in DNA is CCCTAGAG, then what would be the sequence in messenger RNA after transcription?

A. GGGAUCUC

B. GGGATCTC

C. GGGGTCTC

D. GGAAUCUC

Explanation: A pairs up with U and G pairs up with C, in case of RNA molecule. 

58. Taxonomy includes the arrangement of organisms into different taxa. Which of the following represents the correct hierarchy of various taxa of classification?

A. Species, genus, family, order, class, phylum

B. Order, family class, phylum, kingdom

C. Species, genus, order, family, class, phylum

D. Species, genus, family, class, order, phylum

Explanation: Fact 

59. Complementary DNA molecule is

A. an artificial DNA

B. single stranded DNA

C. DNA from mRNA

D. a small segment of chromosomal DNA

Explanation: mRNA produces single stranded DNA through the enzyme reverse transcriptase. These single stranded DNA forms double stranded DNA through the action of enzyme DNA polymerase.

60. Capsid, the protective coat of a virus is made up of __________ subunits known as capsomer

A. Lipid

B. Protein

C. RNA

D. DNA

Explanation: Viruses contain an inner nucleic acid core (genetic material) and an outer protein coat (capsid).

61. In plants, which sugar is transported from source to sink through sieve tubes?

A. Fructose

B. Sucrose

C. Glucose

D. Starch

Explanation: Sucrose is translocated from source into the sieve tubes through companion cells. 

62. If a carrier hemophilic female (XHXh) is married to a hemophilic male (XhY). What will be the ratio of presence of hemophilia in the children. Select best answer from given condition.

XHXh x XhY

A. 100% all females and males will be haempohilic

B. carrier female 25%, hemophilic female 25%, 25% normal male and 25% hemophilic male.

C. females and males both have 50% chances of getting hemophilia

D. females have 50% chances of getting hemophilia and males will be 100 % hemophilic

Explanation:

       XH

Xh

 

Xh

 

XHXh

 

XhXh

 

Y

 

XHY

 

XhY

63. As a result of replication, parental DNA would become completely dispersed and that each strand of all the daughter molecules would be a mixture of old and new DNA. This is called as: 

A. Conservative idea

B. Dispersive idea

C. Disruptive idea

D. Semi-conservative Idea

Explanation: Each daughter molecule has one original strand and one new strand. This is called semi conservation. 

65. A student of chemical engineering mistakenly engulfed the toxic compound “A” which was a potent inhibitor of certain enzyme. He was immediately brought to hospital where Dr. injected intravenously substrate “B” to minimize the toxic effect to compound A. His life was saved from serious damages. The treatment method shows that compound A was a __________ inhibitor.

A. Temperature sensitive

B. Competitive reversible

C. Irreversible

D. Non-competitive reversible

Explanation: Competitive inhibitor binds to the active site but can be reversed by increasing substrate concentration. Here, it is done by intravenous injection of substrate. 

Non competitive inhibitor changes shape of active site by attaching itself at site other than active site.

67. Four plants are present in different environmental conditions. Plant A is present in warm climate with continuous rainfall, plant B is present in a cool forest, plant C is present in warm climate with little breeze while plant D is present in warm climate with high wind speed. Which one of the above plants will have highest rate of transpiration?

A. Plant B

B. Plant D

C. Plant C

D. Plant A

Explanation: Factors Affecting Transpiration: 

Light: 

Light stimulate the opening of stomata.

Temperature: 

Evaporation rate depends on temperature.

Humidity: 

Humidity and evaporation related inversely. Greater the humidity, lower will be the evaporation.

Wind: 

Winds reduce the humidity of the atmosphere and thus increase the transpiration rate.

68. Sequence of amino acids in a polypeptide chain of protein molecule corresponds to the sequence of nucleotides on mRNA for that protein. It reading frame of mRNA for a human protein is 993 nucleotide including a stop codon at the end, how many amino acids would be incorporated in the polypeptide chain?

A. 331

B. 993

C. 93

D. 330

Explanation: Three nucleotides encode one amino acid. There is one stop codon at the end which means 990 nucleotides are representing polypeptide chain here. So the number of amino acids are 990/3.

69. The major function of Basophils is to:

A. Destroy small particles by phagocytosis

B. Release heparin to prevent blood clotting

C. Inactivate inflammation producing substances

D. Transport oxygen

Explanation: Basophils inhibit or prevent blood clotting by releasing a substance known as heparin. 

70. Large lymph vessels ultimately form larger lymph duct, which drains its lymph into:

A. Destroy small particles by phagocytosis

B. Vena Cava and Aorta

C. Subclavian Vein

D. Subclavian Artery

lymphatic capillaries join together to form lymphatic vessels which empty into subclavian veins.

71. Blood group AB is an example of

A. Complete dominance

B. Recessive alleles

C. Incomplete dominance

D. Co-dominance

Explanation: Co-dominance is the phenomenon in which both the alleles express themselves. Blood group AB is an example of co-dominance.

72. Passive processes for the movement of molecules across cell surface membrane are:

A. facilitated diffusion and osmosis

B. pinocytosis and facilitated diffusion

C. diffusion and exocytosis

D. osmosis and phagocytosis

Explanation: Passive movement does not involve any energy expenditure. Simple diffusion is a passive process. Exocytosis is the movement of material outside the cell either passively or actively. 

73. In glycine R is ________.

A. Fatty acid

B. Ethane

C. Hydrogen

D. methane

Explanation: Glycine is the simplest amino acid with R being hydrogen. 

74. Glycolysis takes place in the _______ of cells.

A. Golgi complex

B. Nucleus

C. Cytoplasm

D. Mitochondria

75. Lipids contain double amount of energy as compared to the same amount of carbohydrates due to the presence of:

A. Lower proportion of C-H bonds

B. Higher proportion of C-H bonds

C. Higher proportion of C-O bonds

D. Higher proportion of Oxygen

Explanation: Lipids have energy stored in their C-H bonds. They have long carbon chains which store energy. 

74. If water has high latent heat of vaporization, how this property of water could be helpful to plants and animals?

A. With the release of large amount of water vapors, a small amount of heat loss can take place.

B. No cooling effect with the release of even large amount of water vapors.

C. It will keep their temperature very high.

D. With the release of small amount of water vapors a great amount of heat loss can take place.

Explanation: This property of water acts as cooling system. If water has high latent heat of vaporization, then less water vapors would be formed.

75. How many molecules of ATP would be utilized for phosphorylation of one glucose molecules during glycolysis?

A. One

B. Four

C. Two

D. Three

Explanation: During first step of preparatory phase of glycolysis, ATP gives its phosphate group to glucose to form glucose-6-phosphate. Glucose-6-phosphate is isomerized to fructose-6-phosphate. During second step, again ATP loses its phosphate group to form fructose-1,6-bisphosphate.

Then fructose-1,6-bisphosphate breaks down to give 2 isomers a) glyceraldehyde 3- phosphate or 3-phospho-glyceraldehyde, b) dihydroxyacetone phosphate.

76. Among following, ________ enzyme is naturally found in human immunodeficiency virus (HIV).

A. DNA polymerase

B. RNA polymerase

C. Reverse transcriptase

D. Ligase

Explanation: Reverse transcriptase is present in HIV, the function of which is to transcribe retrovirus RNA into DNA (as catalyst).

77. The structure of a fibrous protein comprises of polypeptide chains in the form of:

A. Cluster

B. Flat uncoiled chains

C. Spherical or curried up ball

D. Long strands or fibrils

Explanation: Fibrous proteins are in the form of long strands. 

78. Which is an example of Disaccharide?

A. Lactose

B. Glycogen

C. Starch

D. Fructose

Explanation: Lactose is disaccharide. It is composed of galactose and glucose subunits. 

79. Which one is an example of Nucleotide?

A. Adenosine

B. ATP

C. Guanine

D. NAD

Explanation: Adenine, guanine, cytosine, thymine, and uracil are nucleotides. Adenosine is nucleoside. ATP, adenosine triphosphate is energy containing molecule. 

80. Which of the following photosystem is involved in cyclic photophosphorylation?

A. PS I and PS II

B. PS II

C. PS III

D. PS I

Explanation: The light reaction is switched to cyclic phosphorylation when ATP produced by non cyclic phosphorylation is not enough to enter Calvin cycle. The NADPH accumulates in the chloroplast and shifts from non cyclic to cyclic flow until ATP meets the demand. 

This path uses only photosystem I.

CHEMISTRY

81. Which product is obtained by the hydrolysis of 1- chlorobutane with the aqueous sodium hydroxide?

A. 1-butanol

B. 1-butane

C. Butanone

D. 1- butanal

Explanation: It is nucleophilic substitution reaction. Chloride ion leaves the compound and OH- takes its place. Primary alcohol is produced. 

Secondary haloalkane will produce secondary alcohol on hydrolysis. 

82. Treatment of ethene with cold sulphuric acid followed by reaction with boiling water yield:

A. Ethane

B. Ethyne

C. Ethanal

D. Ethanol

Sulphuric acid and ethane give addition reaction. Ethyl hydrogen sulphate is formed. This reaction follows Markonikov’s rule. 

The resulting compound when hydrolysed in hot water gives alcohol. 

83. The average atomic mass of Boron is 10.8 it has two isotopes of masses 10 and 11 respectively. What is the percentage of isotope with the average mass of 10?

A. 20%

B. 50%

C. 60%

D. 80%

Explanation: Average atomic mass = (atomic mass of isotope 1 x relative abundance) + (atomic mass of isotope 2 x relative abundance)

If we assume that relative abundance of isotope with mass 10 is 20%, then

(10 x 0.2) + (11 x 0.8) = 10.8

84. In contact process, to which substance adequate quantities of water is added to convert it to sulphuric acid?

A. SO3

B. H2S2O7

C. SO2

D. HSO4

Explanation: Contact process basically consists of following three steps:

  • Production of sulphur dioxide

It can be produced through 2 processes:

  • Burning sulphur in an excess of air
  • Heating sulphite ores (e.g. pyrite) in an excess of air
  • Conversion of sulphur dioxide into sulphur trioxide
  • Conversion of sulphur trioxide into sulphuric acid

Sulphur trioxide is first dissolved in concentrated sulphuric acid. And then the product is reacted with water to get concentrated sulphuric acid.

85. Which of the equations shows the same “twice” the enthalpy change of neutralization as the following equation

HCl + NaOH  NaCl + H2O

A. NH4Cl + NaOH  NaCl + H2O + NH3 

B. MgCO3 + 2HCl  MgCl2 + CO2 + H2O 

C. KOH + HCl  KCl + H2O 

D. H2SO4 + Mg(OH)2  MgSO4 + 2H2O 

Explanation: The standard enthalpy of neutralization Hon is the amount of heat evolved when one mole of hydrogen ions H+, from an acid, reacts with one mole of hydroxide ions from a base to form one mole of water.

Only option D has twice the number of hydrogen and hydroxide ions as compared to the reaction given in question. So, its enthalpy of neutralization would be double. 

86. The decomposition of phosphorus pentachloride in the presence of moisture takes place by the following mechanism:

PCl5(s) + H2O(i)  POCl3 (i) + 2HCl(aq) (Slow Step)

POCl3(i) + 3H2O(i)  H3PO4 (i) + 3HCl(aq) (Fast Step)

PCl5(s) + 4H2O(i)  H3PO4 (i) + 5HCl (aq)

The rate equation for this reaction will be:

A. Rate = k (PCl5)[H2O]

B. Rate = [PCl5][H2O]

C. Rate = k [POCl3][H2O]3 

D. Rate = k [PCl5][H2O]4 

Slow step determines the rate of reaction. In this case, option A is the correct option. 

88. The names of functional groups in the following compound X are;

A. Primary alcohol, nitrile and benzene ring

B. Secondary alcohol, amine and benzene ring

C. Secondary alcohol, nitrile and phenol ring

D. Secondary alcohol, nitrile and aryl ring

Explanation: Fact. Benzene ring is called aryl ring. The group on ortho position is secondary alcohol because –OH group is attached to the carbon which is attached to two more carbon atoms. 

89. If the energy of activation of a chemical reaction is very low, the rate of that chemical reaction is observed to be very high because?

A. Reaction proceeds without any transition state

B. Number of efficient or fruitful collisions increase

C. Concentration of the reactants becomes irrelevant

D. Molecules of the reactants move slowly

Explanation: Lower activation energy means there are molecules with sufficient energy to produce fruitful collisions. As the result, rate of reaction is very high.

90. Which of the following compounds will give a secondary alcohol after reaction with NaBH4?

A. CH3COOCH3 

B. CH3COCH3

C. CH3CH2COOH

D. CH3CH2CHO

Explanation: Aldehydes and ketones reduce to give alcohols in the presence of reducing agents. Aldehydes give primary alcohols while ketones give secondary alcohols. 

91. Copper is a typical transition metal. Its atomic number is 29. In which oxidation state does it have partially filled orbital in d-subshell?

A. Cu2+

B. Cu

C. Cu

D. Cu+

91. Which of the following reagent is required for preparation of acyl chloride (CH3COCl) from ethanoic acid?

A. POCl3

B. HCl

C. PCl5

D. CH3Cl

Explanation: Chloride ion acts as nucleophile. OH- ion leaves and Cl- ion is added. 

92. The given diagram shows the enthalpy changes during a chemical reaction.

This diagram represents:

A. A non-spontaneous process

B. An isothermic process

C. An endothermic reaction

D. An exothermic reaction

Explanation: The graph shows that the enthalpy of reactants is greater than the enthalpy of products. This means that the energy is lost during this process. So the reaction is exothermic. 

93. Ionization energy decreases down the group from top to bottom due to:

A. Increase in proton number

B. Decrease is atomic size

C. Increase in shielding effect to the intervening electrons

D. Increase in atomic mass

Explanation: As we go down the group, the shielding effect increase between the electrons and it becomes easy to remove it from the outermost shell. 

94. Free Nitrogen and oxygen are present in atmosphere but they do not react with each other under normal conditions, because:

A. Oxygen is found in less concentration

B. Nitrogen requires a catalyst

C. Nitrogen is highly inactive gas

D. Oxygen is very inactive

Explanation: The triple bond between the nitrogen atoms in nitrogen gas molecule is too strong to be broken easily. So acts as inert gas. 

95. Which derivative of benzene shows maximum reactivity in electrophilic substitution reactions?

A. Benzoic acid

B. Benzaldehyde

C. Nitrobenzene

D. Methyl benzene

Explanation: All the substituents of benzene in the given options have negative inductive effect except methyl benzene. 

96. Which of the following reactions is used for the production of alcohols on industrial scale?

A. Hydrogenation of alkenes

B. Hydroxylation of alkenes

C. Hydrohalogenation of alkenes

D. Hydration of alkenes

Explanation: Alkenes give addition reaction. Water is added to give alcohol.

97. The pH of 10-2 M aqueous solution of sodium hydroxide is

A. 12

B. 10

C. 13

D. 14

Explanation: Sodium hydroxide has one hydroxyl ion. It is a strong base. 

98. In the reaction sequence:

H3C – CH2 – CH2 – Br + Alc KOH  C  D

Product D wil be

A. 2-propanol

B. 1-propanol

C. Propanoic acid

D. Mixture of methanol and ethanol

Explanation: The reaction is double replacement reaction. First propene is formed which is then converted to alcohol.

99. What is the measure of activation energy in an endothermic reaction?

A. The energy of activation of forward reaction is less than that of backward reaction

B. The energy of activation of forward-backward reaction is same

C. The energy of activation of backward reaction is less then that of forward reaction

D. The energy of activation of backward reaction is more than that of forward reaction

Explanation: Energy of activation of endothermic reaction is greater as it needs energy to be absorbed. So energy of activation of forward reaction would be more. 

100. Maleic acid and Fumaric acid, both have chemical formula C4H4O4. The structure of these acids is shown below.

Maleic acid and Fumaric acid are:

A. Position isomers

B. Structural isomers

C. Metamers

D. Cis-trans isomers

Explanation: Both are butenedioic acid. Maleic acid is cis form while fumaric acid is trans form.

101 . Alkenes undergo:

A. Nucleophilic substitution

B. Nucleophilic addition

C. Electrophilic addition

D. Electrophilic substitution

Explanation: The double bond acts as nuleophile. So the electrophilic addition reaction takes place.

102. Nitriles (RCN) on hydrolysis in the presence of a mineral acid yield:

A. Ethers

B. Carboxylic acids

C. Aldehydes

D. Alcohols

Explanation: The triple bond exists between C and N of nitriles. On hydrolysis, nitrogen atom leaves and carboxylic acid is formed. 

103. The Ka value of HCl, CH3COOH, HF and H2SO4 are 10+7, 1.85 x 10-5, 6.7 x 10-5 and 10+2 respectively. The decreasing order of acidic strength is:

A. HCl > HF > H2SO4 > CH3COOH

B. HCl > CH3COOH > HF > H2SO4

C. HCl > H2SO4 > HF > CH3COOH

D. CH3COOH > HF > H2SO4 > HCl

Explanation: Greater value of Ka indicates greater ability of acid to dissociate which means greater acidic strength.

104. Which of the following will give a positive test with Tollen’s reagent?

A. Aldehydes

B. Tertiary alcohols

C. Ketones

D. Carboxylic Acids

Tollen’s reagent is ammonical silver nitrate solution, which is used for the identification of aldehyde group. 

105. Select the reagent X from the following choices for this conversion;

CH3CH(OH)CH(CH3)2  CH3COCH(CH3)2

A. Acidified Phosphoric acid

B. Acidified Oxalic acid

C. Acidified Potassium hydroxide

D. Acidified Potassium dichromate (VI)

Secondary alcohols undergo oxidation in the presence of acidified potassium dichromate to give ketones. 

106. which of following compounds Is responsible for the depletion of ozone layer

A. Carbon tetrachloride

B. Choroflorocarbons

C. Hydrofluorocarbons

D. Methane

The free radicals produced by chlorofluorocarbons are responsible for ozone depletion. 

107. Which one the following compound is additional polymer?

A. Carbohydrate

B. Polyvinyl chloride

C. Nylon

D. Polyester

Explanation:

Addition polymerization is “The process of polymerization in which the monomers are unsaturated and the resulting polymer contains only one kind of monomer”

Vinyl chloride is unsaturated compound which polymerizes to give PVC.

108. Ketones can be made by oxidation of

A. Secondary Alcohols

B. Tertiary Alcohols

C. Aldehydes

D. Primary Alcohols

Explanation: Secondary alcohols undergo oxidation in the presence of acidified potassium dichromate to give ketones. 

109. Which of the following bond is responsible for joining the amino acids in proteins?

A. Ionic Bond

B. Metallic Bond

C. Di sulfide bond

D. Peptide Bond

Explanation: Peptide bond is formed between amino group of one amino acid and -COOH group of second amino acid to form -CONH- with elimination of water. This is called peptide bond. 

110. Identification tests for functional groups of organic compounds are associated with specific observations. Tollen’s reagent is ammonical silver nitrate solution, which is used for the identification of a functional group X with an observation O. identify X and O.

A. X = Ketone O= grey precipitate

B. X = Ketone O=silver mirror

C. X=Aldehyde O=Silver mirror

D. X=Aldehyde O=red precipitate

Explanation: Tollen’s reagent gives positive test with a aldehydes and observation includes layer of silver mirror in the test tube. 

111. The structure of Xenon trioxide is shown below,

With reference to the Valence shell electron pair repulsion theory, (VSEPR), the shape of XeO3 is:

A. Trigonal pyramidal

B. Tetrahedral

C. Bent (or angular)

D. Trigonal planar

Explanation: The lone pair pushes the bonds away and trigonal pyramidal shape is achieved. 

112. Which two elements are isotopes?

A. and 

B. and 

C. and 

D. and 

Explanation: Isotopes have same atomic number but different mass number. 

113. According to Watson and Crick’s model of DNA, the DNA molecule consists of a double helix. What type of forces are responsible to keep two strands of DNA together?

A. Hydrogen bonding

B. Dipole-induces dipole forces

C. van der Waal’s forces

D. Ionic bonding

Explanation: Hydrogen bonds are formed between nucleotides of one strand and the second strand of DNA molecule. 

114. An inter molecular force of attraction X is relatively stronger than the other inter molecular forces, it stabilizes -helix and -pleated sheets of proteins. The double helical structure of DNA is also stabilized by this force of attraction. Identify X.

A. Hydrogen bonding

B. van der Waal’s Forces

C. Ionic interactions

D. Dipole dipole attraction

Explanation: Hydrogen bonds are formed between nucleotides of one strand and the second strand of DNA molecule. 

115. Which one of the following molecules has SP3 hybridization?

A. CH4

B. C2H2

C. CO2

D. C2H4

Explanation: The electronic configuration shows that carbon has one electron in 2s orbital and 3 in 2p orbitals. It hybridizes into 4 sp3 orbitals. 

116. Oxidation number of particular element can be directly or indirectly inferred from its:

A. Atomic mass

B. Physical state

C. Group number

D. Atomic size

Explanation: Group number can be used to deduce the oxidation number. 

117. Which of the following sets constitutes of all the molecules and ions of non-planar geometry?

A. PH4+, NH3, SO3, Benzene

B. CH = CH, H2O, BeCl2, H2S

C. SO2, C2H4, BF3, NO3+

D. CH4, NH4+, MnO4, NF3

Explanation: Sp2 hybridized compounds show planar geometry. SO3 , CH = CH, and SO2 are sp2 hybridized. 

118. Nitrogen has the atomic mass of 7.

Which of the following electronic configurations is of a Nitrogen atom in ground state?

A. 1s2, 2s2, 2px2, 2py1

B. 1s2, 2s2, 2px2, 2pz1

C. 1s2, 2s2, 2px1, 2Py1, 2pZ1

D. 1s2, 2s2, 2Py2, 2Pz1

Explanation: Orbitals that are half filled are more stable. 

119. CFC’s are organic compounds, which are derivatives of saturated hydrocarbons. They have high bond dissociation values therefore they are inert and non toxic for the living organisms.

The word CFC’s stands for;

A. Chlorofluoridecarbons

B. Carboflourochlorines

C. Chlorofluorcarbides

D. Chlorofluorocarbons

Explanation: UV radiations interact with CFCs and produce free radicals that play role in ozone layer depletion.

120. Amino acids are bi-functional compounds, with a general formula NH2CH(R)CO2H. A tripeptide is formed between Alanine (ala), Glycine (gly) and Iysine (Iys). There is no repetition of amino acid in this tri-peptide, suggest how many tri-peptides are possible?

A. 9

B. 3

C. 6

D. 12

Explanation: Write all the possible sequences. Total possible tripeptides would be 6 without repitition.

121. Which type of reaction takes place when a carbonyl compound is treated with a mixture of NaCN and an acid?

A. Substitution reaction

B. Electrophilic addition reaction

C. Nucleophilic addition reaction

D. Displacement reaction

122. Disposable cups are made of a polymer polystyrene. Polystyrene is:

A. A polyamide

B. A condensation polymer

C. An addition polymer

D. A polyester

Explanation: Styrene is an unsaturated compound. It is polymerized to give addition polymer polystyrene. 

 123. During stoichiometric calculations, which of the following laws must be followed?

A. Dalton’s law

B. Law of conservation of mass

C. Avogadro’s law

D. Law of conservation of energy

Explanation: Stoichiometric calculations are used to determine the quantitative relationship between reactants and products. Law of conservation of mass must be followed to determine such relationship. 

124. Which of the following element is not present in halogens?

A. Cl

B. I

C. F

D. Fe

Explanation: Iron is a transition element. 

125. Which enthalpy change is relevant in the following process

Na(s)  Na(g) H=+

A. Enthalpy of atomization

B. Enthalpy of vaporization

C. Enthalpy of fusion

D. Enthalpy of formation

Explanation: Enthalpy of atomization is the amount of energy change when a compound or molecule is converted to individual atom. In this case, solid lattice of sodium is converted to gaseous state in which the atoms exist individually. 

126. Which of the following substances exhibits hydrogen bonding?

A. HI

B. HS

C. SiH4

D. NH3

Explanation: Ammonia exhibits hydrogen bonding. Nitrogen has one lone pair of electrons which form hydrogen bond with the hydrogen of another molecule of ammonia. Other examples include H2O and HF. 

127. Which balanced chemical equation show the formation of ethanoyl chloride using thionyl chloride?

A. CH3COOH + SOCI2 – CH3COCI + SO2 + HCl

B. CH3CH2COOH + 2SOCl2 – CH3CH2COCI + SO2 + HCl

C. CH3CH2COOH + 2SOCl2 – CH3CH2COCl + SO3 + HCl

D. HCOOH + SOCI2 – HCOCl + SO2 + HCl

Explanation: Acetyl chloride is formed along with Sulphur dioxide and HCl. Both the by products are gaseous. 

128. The number of moles of water in 1Kg ice are

A. 100 moles

B. 1000 moles

C. 50 moles

D. 55.5 moles

Explanation: Molar mass of ice = 18g 

Number of moles = given mass/molar mass = 1000/18 = 55.5

129. For an equilibrium reaction:

2SO2(g) + O2(g) = 2SO3(g)

The forward reaction is exothermic, increase in temperature shifts the equilibrium position towards left because,

A. The concentrations of SO2 and O2 decrease and concentrations of SO3 increase as the temperature increase

B. the concentrations of SO2 and O2 increase and concentration of SO3 decrease as the temperature increases

C. The concentrations of SO2 and O2 increase and concentration of SO3 stays same as the temperature increases

D. The concentrations of SO3, O2 increase as the temperature increase

Explanation: In this process the forward reaction is exothermic and the reverse reaction is endothermic. So the heat produced by the formation of product is used to proceed in the backward direction to produce reactants.
So increasing temperature will shift the reaction in the backward direction and reactants will be formed. 

130. Mg2+ + 2e  Mg Eo = – 2.37 V

Zn2+ + 2e  Zn Eo = – 0.76 V

2H+ + 2e  H2 Eo = 0.000 V

Cu2+ + 2e  Cu Eo = + 0.34 V

Cl2 + 2e  2Cl Eo = + 1.36 V

Au3+ + 2e  Au Eo = + 1.50 V

Keeping in view the values of standard reduction potential given above, which one of the following would you select as a feasible redox chemical reaction?

A. Mg + 2H+  Mg2+ + H2

B. Cu + Zn2+  Cu2+ + Zn

C. 2Cl- + I2  Cl2 + 2I

D. 2Au + 6H+  2Au3+ + 3H2

Explanation: Negative value means these substances are good reducing agents and positive values mean oxidizing agent.

Only option A is possible. 

131. Modern periodic table is arranged in ascending order of?

A. Atomic mass

B. Proton number

C. Mass number

D. Nucleon number

Explanation: Atomic mass, nucleon number, and mass number are same. Proton number is always constant. 

132. In the second period of elements, although oxygen lies next to nitrogen yet its ionization first energy lower than that of nitrogen because?

A. Oxygen is paramagnetic is character

B. In oxygen, there exists repulsion between pair of electrons present in the same orbital of valence shell

C. Nuclear charge of oxygen is greater the nitrogen

D. Oxygen has higher electron affinity

Explanation: Half filled orbital is more stable. Oxygen has one electron more than the half filled. There exists repulsion in electrons, so electron is easily lost to get half-filled orbital.  

133. Chlorofluorocarbons (CFCs) are important compounds which are used as refrigerants but these are also responsible for Ozone layer depletion. If a Chlorofluorocarbon CFCl3 is present in stratosphere, which of its reaction intermediates are actually responsible for the breakdown of Ozone molecule?

A. CHCl2 and Cl

B. Cl and ClO

C. CHCl2 and ClO

D. CFCl2 and CFCl3

Explanation:

134. Solution contains 85.5 g of sucrose (C12H22O11) in 250 cm3. What is its molarity?

A. 0.5 M

B. 2 M

C. 0.25 M

D. 1 M

Explanation: Molarity = number of moles / volume in liters

Number of moles of sucrose in 85.5g is 0.25 moles. 

250cm3 is equal to 0.25 liters. 

Molarity = 0.25/0.25 = 1M

135. Which of the following is the electronic configuration of Cr?

A. [Ar] 3d6 4s0

B. [Ar] 3d4 4s2

C. [Ar] 3d5 4s2

D. [Ar] 3d5 4s1

Explanation: Partially filled orbital is more stable than fully filled. So the electron goes to 4s orbital. 

136. All the collisions between the particles of gases are elastic in nature. What is meant by “Elastic Collisions”?

A. No change in potential energy during the collisions

B. The velocity of the molecules changes

C. No change in the kinetic energy

D. No changes in mass during the collisions

Explanation: According to the definition of elastic collisions, there is no net change in kinetic energy during collisions. 

137. Aqueous solutions of Iodine and Sodium hydroxide were mixed in a round bottom flask at 70oC. Following chemical reaction was carried out.

3I2 + 6NaOH  NaIO3 + NaI + H2O

This reaction is termed as

A. Redox reaction

B. Free radical reaction

C. Precipitation reaction

D. Subscription reaction

Explanation: Iodine is both reducing and oxidizing. Two oxidation states of iodine are observed in the products.

138. Carboxylic acids can be reduced into corresponding alcohols. Which of the following reagent can be used for this purpose?

A. LiAIH4

B. K2Cr2O4

C. KMnO4

D. H2SO4

Explanation: LiAIH4 acts as reducing agent. Carboxylic acid undergoes reduction to form alcohol.

139. How many moles of calcium carbonate are present in 1.75 kg of calcium carbonate? (Ar of Ca = 40, Ar of C = 12, Ar of O = 16)

A. 0.0175 mol

B. 1.75 mol

C. 17.5 mol

D. 1750 mol

Explanation: Number of moles = given mass / molar mass

Molar mass of calcium carbonate: 100 g

Number of moles in 1750 g = 17.5 

140. Which of the following molecule shows cis-trans isomers?

A. C2HCl3

B. C2H2Cl4

C. C2H4

D. C2H2Br2

Explanation: For a compound to show cis trans isomerism, there should be two non identical groups on   each carbon with double bond.

PHYSICS 2019 Past Paper

141. Heavy nucleus of atoms go through fission so that they can:

A. absorb high amount of energy

B. absorb low amount of energy

C. increase their binding energy per nucleon

D. reduce their binding energy per nucleon

Explanation: In fission of heavy nucleus of atoms the atoms are bombarded with a neutron which breaks up the heavier unstable nucleus into smaller particles to reduce the binding energy of the heavier nucleus.

142. For projectile motion in the absence of air resistance:

A. vertical speed is constant

B. horizontal force is constant

C. horizontal acceleration is zero

D. vertical acceleration is zero

Explanation: In projectile motion neglecting air resistance the only force acting on the system is gravitational force which is in the vertical direction. There is no net force in the horizontal direction hence according to newton’s second law F = ma acceleration in the horizontal direction is 0. 

143. The range of the projectile depends upon the velocity of the protection and the angle of the projection i.e 45o. for a fixed velocity, when the angle projection is larger than 45o. which of the following is correct?

A. Both the height and the range attained by the projectile will be less.

B. Both the height and the range attained by the projectile will be more.

C. The height attained by the projectile will be less but the range is more.

D. The height attained by the projectile will be more but the range is less.

Explanation: The height of the projectile is given by

The range is given by 

For height sin is maximum when  = 90 but for the same angle sin2 = 0 hence the range decreases.

144. The wavelength of the electromagnetic wave having frequency of 3 kHz will be?

A. 80 km B. 140 km

C. 100 km D. 120 km

Explanation: All electromagnetic waves travel with the speed of light which is 3×108 m/s

So using the formula v= fλ we get 100km.

145. An alternation voltage V (in volts) is represented by the equation:

V = 300 sin (100t)

What is the value of “f” for this voltage?

A. 25 Hz B. 200 Hz

C. 50 Hz D. 100 Hz

Explanation: the angle of the sin function is given in radians in the form sin() where  is the angular frequency and is 100π. We know that = 2πf. Thus we get

100π = 2πf,   f = 50Hz.

146. The diameter of a wire is measured by using a micrometer screw gauge with least count of 0.01 mm, then which of the following readings will be correct?

A. 0.067 cm B. 0.0067 mm

C. 0.67 cm D. 6.70 cm

Explanation: the least count is the minimum decimal point upto which the instrument can measure. It is 0.01mm in this case which corresponds to 0.001 cm. So this micrometer can read up to 3 decimal places in cm.

147. Which of the following is statement shows that no work is done?

A. Pushing a car to start it moving

B. Writing an essay on a page

C. Lifting the weights

D. The moon orbiting the earth

Explanation: Work is defined as the distance moved in the direction of the force applied. Mathematically W = fxdxcos.

The moon orbits the earth because of centripetal force which is acting perpendicular to the motion of the moon hence no work is done.

148. When the length of simple pendulum is doubled, then ratio of its new time period to old time period is:

A. B.

C.D.

Explanation: the time period of a simple pendulum is given by

Where l is the length of the pendulum. When the length is doubled the time period increases by a factor of  as all other values are constant.

149. The direction of current through the load resistance of a full-wave rectification circuit:

A. inverts for negative cycle

B. changes for every cycle

C. Invert for positive cycle

D. remains constant

Explanation: The following diagram shows a full wave rectifier

The direction of I is same for both the positive and negative cycles. When the sine wave is positive diodes D1 and D2 conduct and when it is negative diodes D3 and D4 conduct keeping the direction of current same.

150. A wire has a spring constant of 5 x 104 N m-1. It is stretched by a force to extension of 1.4 mm. calculate the strain energy stored in the wire.

A. 4.9 x 10-5 J B. 4.9 J

C. 4.9 x 10-5 J D. 4.9 x 10-2 J

Explanation: The formula to calculate strain energy is  where k is the spring constant and x is the extension in m. Substituting these values we get 0.049J.

151. If two objects of equal masses ‘m’ are moving towards each other with the same speeds ‘v’ then what will be the total final momentum after elastic head-on collision.

A. – mv kg/s B. mv kg m/s

C. 2 m kg/s D. 0 kg m/s

Explanation: in elastic collisions the momentum before and after the collision is same. So calculating the momentum before collision

For object 1 p1 = m1v1

For object 2 p2 = m2v2

Since the masses are same and the velocities are also the same but in opposite direction hence the total momentum will be

Ptotal = mv – mv = 0

Hence the total momentum after the collision will also be 0 

152. Molecules of a gas at constant pressure for a fixed amount of gas have average kinetic energy X. Increasing temperature from 27°C to 327°C, average K.E. of molecules will become:

A. 200X B. 20X

C. 300X D. 2X

Explanation: The kinetic energy of gas molecules is given by

 where R is gas constant, NA is the Avogadro’s number and  T is the temperature in Kelvin

Converting the temperature into Kelvin

K = 273 + ֯C 

For 27 ֯C K is 300 and for 327 ֯C K is 600. So the temperature is doubled hence the kinetic energy will also be doubled 

153. An automobile is moving forwards with uniform velocity due to the force exerted by its engine. If that force is double with the velocity remaining constant what happens to its total power?

A. It does not change B. It is squared

C. It is halved D. It is doubled

Explanation: Power is given by 

P = F x v so if the force is doubled keeping the velocity constant the power will also be doubled.

154. In Double Slit experiment, the fringe spacing of the diffracted rays increases when:

A. the distance between the screen and the slits decreases

B. the wavelength of the diffracted rays Increases

C. the distance from mid points of the slits to the central point of the fringe on the screen increases

D. The distance between the slits increases

Explanation: The fringe width is given by

     Where D is the distance between the screen and the slits, d is the distance between the slits and  is the wavelength of the diffracted light. So to increase the fringe width we should increase the distance from the screen to the slits, increase the wavelength of the light or decrease the distance between the slits.

155. The area under the extension-load graph of an elastic material whose elastic limit has not been exceeded given its:

A. stress B. Strain energy

C. Young modulus D. Strain

Explanation:  The following diagram shows a load extension graph

156. Minimum energy required to eject an electron from metal surface is called:

A. Work function

B. Stopping potential

C. Threshold frequency

D. Electromotive force

Explanation: It is the definition of work function

157. The unit of magnetic flux density is the tesla, ‘T’, it can also be expressed as

A. 1 N-1 A-1 m B. 1 N-1 A-1 m-1

C. 1 N A-1 m D. 1 N A-1 m-1

Explanation: Magnetic flux density is given by  where  is the flux and A is the area. The base units of flux are kg.m2.s-2.A-1 and the units of area are m2. The units of B becomes

We can see that in the options we have the units of force (N) which is kg.m.s-2. Substituting these units of force we get our answer.

158. Percentage un-certainty in length and width of a rectangle is 2% and 3%. The total un- certainty in area of that rectangle is? 

A. 1.5% B. 5%

C. 6% D. 1%

Explanation: When dealing with multiplication, division and exponents we simply add the percentage uncertainty. 

159. What is the quark composition of a Proton?

A. Two up quarks and one down quark

B. One up quark and two strange quarks

C. Two up quarks and one strange quark

D. Two down quarks and one up quark

Explanation: It is the structure of the proton.

160. What will be the expression for the observed frequency, if the source is moving towards the observer?

A. B.

C.D.

Explanation: It is the formula to calculate the frequency if the observer is stationary.

161. Work done due to centripetal force for circular motion will be:

A. Reduced B. Maximum

C. Half D. Zero

Explanation: Work is defined as the distance moved in the direction of the force applied. Mathematically W = fxdxcos.

The centripetal force is acting perpendicular to the motion of the object hence no work is done.

162. If we give a direct current to the transformer’s primary coil, then there will be:

A. Less emf produced in the secondary

B. No emf produced in the secondary

C. Equal emf produced in the secondary

D. More emf produced in the secondary

Explanation: Transformers are designed to work for AC because it produces a changing magnetic flux in the coils of the transformers. DC is constant and does not produce changing magnetic flux. 

163. The value and units of the Plank constant ‘h’ can be expressed as:

A. 6.63 x 10-34 Js-1 B. 6.63 x 10-43 Js

C. 6.63×1 O-34 Js D. 3.63 x 10-34 Js

164. A negligible small current between input terminals of the operational amplifier is because of:

A. Low input resistance

B. Low output resistance

C. High output resistance

D. High input resistance

165. If a conductor of length 7m is placed in a magnetic field of strength 0.3T carrying current 1A, parallel to the field. What will be the force acting on it due to this magnetic field?

A. 2.1 N B. 0 N

C. 3.1 N D. 7 N

Explanation: The force on a current carrying conductor in a magnetic field is given by

 where B is the magnetic field strength, I is the current and L is the length of the conductor. Substituting these values we get our answer.

166. The horizontal component of Earth magnetic flux density is 1.8×10-6 T. The current in a horizontal cable is 160A. Calculate the maximum force per unit length?

A. 2.88 x 10-4 N/m B. 2.88 x 10-8 N/m

C. 2.88 x 10-2 N/m D. 2.88 x 10-6 N/m

Explanation: The force on a current carrying conductor in a magnetic field is given by

 where B is the magnetic field strength, I is the current and L is the length of the conductor. To find force per unit length we divide both sides with L to get  . Thus substituting the values we get our answer.

167. If we change the magnetic flux linking a coil by rotating the coil in a constant magnetic field, the rate of change of this flux is:

A. Proportional to the emf produced in it

B. Proportional to the change in magnetic field

C. Proportional to the resistance of the coil

D. Proportional to the material of the coil

Explanation: The change in magnetic field produces a change in magnetic flux, hence the faster the change in magnetic field the faster is the change in magnetic flux.

168. Electric field strength of a point charge is E and electric potential is V at a distance r from the point charge. What is the electric potential at a point for the same point charge where the electric field strength is E/4?

A. V/4 B. V/2

C. 4V D. 2V

Explanation: The electric potential due to a point charge q at a distance r is given by

 where k is a constant. If the electric field strength is E/4. It means that the distance is 2r. The electric potential is given by 

Now the distance is 2r hence the electric potential will be V/2  

169. In simple harmonic motion, acceleration will be maximum, when object is at:

A. maximum displacement from the mean position

B. center position

C. mean position

D. Half of the maximum displacement from mean position

Explanation: The following graphs show the displacement velocity and acceleration for a simple harmonic motion

170. Calculate the energy of a photon of frequency 3.0 x 1018 Hz.

(h = 6.63 x 10-34)

A. 19.89 x 10-18 J B. 1.89 x 10-16 J

C. 11.89 x 10-16 J D. 19.89 x 10-16 J

Explanation: Energy of a photon is given by 

E = hf where h is the Planck’s constant and f is the frequency. Substituting the values gives us the answer.

171. In relation T1/2 = 0.693/, which quantity is represented by ?

A. half life B. wavelength

C. activity D. decay constant

Explanation: It is the formula for half life

172. Path difference for the destructive interference can be written as:

A.

B.

C.

D.

Explanation: It is the formula for destructive interference. Whereas the formula for destructive interference is n

173. If a light is emitted by a single source passes through two narrow slits 1.00 mm apart. The interference pattern is observed on a screen 200 cm away and the separation between the centres of adjacent bright fringes is 2.00 mm. what would be the wavelength of the light?

A. 2 m B. 1 m

C. 2 pm D. 1 nm

Explanation: The fringe width is given by

     Where D is the distance between the screen and the slits, d is the distance between the slits and  is the wavelength of the diffracted light. Note that all these lengths must be in meters. Substituting the values gives us the answer

174. The sum of all forms of molecular energies (Kinetic and Potential) of a substance is termed as?

A. Internal energy B. Elastic energy

C. Heat energy D. Absolute energy

Explanation: It is the definition of internal energy

175. Calculate the rate at which energy is transferred by 220 V mains supply which provides a current of 0.1 A to a LED?

A. 22 kW B. 2.2 kW

C. 22 W D. 2.2 W

Explanation: Rate of energy transfer is also called power which is given by 

Substituting the values gives us the answer

176. A particle carrying a charge of 5e falls through a potential difference of 25V. what would be energy acquired by the particle in ‘J’.

A. 125 x 10-19 J B. 1.6 x 10-19 J

C. 125 x 1.6 x 10-19 J D. 125 J

Explanation: work done on a charge in an electric field with a potential difference is given by

 where q Is the charge and V is the voltage. Note that the charge on an electron is 1.6×10-19C. so this particle will have a charge of 5×1.6×10-19C

177. A copper wire has length L and cross-sectional A. its resistance is R. if we halved the length and halved the diameter of wire then what will be the resistance of this wire?

A. R B. 3R

C. 2R D. 4R

Explanation: Resistance is given by 

 where is the resistivity of the material and is fixed, L is the length and A is the cross sectional area. If diameter is halved the cross sectional area reduces by four times. The new resistance will look like  

 =

178. Kirchhoff’s first law/rule corresponds to:

A. Law of conservation of energy

B. Law of conservation of charge

C. Law of conservation of momentum

D. Law of conservation of mass

Explanation: It is the definition of Kirchhoff’s first law

179. Electric field strength at a point between oppositely charged plates is E. if the distance between plates is reduced to half, what will be the new value of electric intensity?

A. 4E B. E/2

C. E/4 D. 2E

Explanation: Electric field strength is given by

 where V is the potential difference between the plates and d is the distance between the plates. If the distance is halved the electric field intensity will be doubled. 

180. An object is moving along a circular path of radius 4m. what will be its angular displacement if it moves 14m on this circular path?

A. 5.5 radians B. 3.5 radians

C. 5.0 radians D. 4.5 radians

Explanation: the circumference of a circle is given by the formula

 Where r is the radius and  is the angular displacement in radians. Substituting the values of s and r gives us the answer.

ENGLISH Past Paper 2019

Part – I: Choose the best option.

181. The accident happened due to the driver’s ________.

A. Nuisance

B. Negligence

C. Reluctance

D. Regret

Explanation: Nuisance means annoyance being caused or felt. Negligence means the inability to take care of something. Reluctance means disinclination to do something. Regret means feeling disappointed over something. Hence, ‘Negligence’ is the most appropriate word. 

182. They sometimes feel a _______ for the mountains and the sea.

A. Yearning

B. Yelling

C. Yielding

D. Regret

Explanation:  Yearning means a feeling of intense longing. Yelling means to shout. Yielding means to produce or present something. Regret means feeling disappointed over something. Hence, the most appropriate word is ‘Yearning’. 

183. I _______ caution in interpreting these results.

A. Uproot

B. Usher

C. Usurp

D. Urge

Explanation:  Uproot means to pull something out of the ground. Usher means to show or guide. Usurp means to take something by force. Urge means to persistently persuade.  Hence, ‘Urge’ is the most appropriate word. 

184. She was feeling ________ even after five hours of the students.

A. Groggy

B. Grope

C. Haggard

D. Paily

Explanation:  Groggy means dazed or weak. Grope means to search with uncertainty. Haggard means to look exhausted and unwell. Hence, the most appropriate word is Groggy, which indicates a feeling. 

185. The new teacher showed no _______ about hitting the students.

A. Quotation

B. Qualms

C. Quakes

D. Quarrel

Explanation:  Quotation means deriving words or phrases used by someone else. Qualms means an uneasy feeling of doubt. Quakes means to shake or tremble. Quarrel means to fight. Hence, the most appropriate word is Qualms. 

186. The Parents were stunned when they saw that children had created complete ________ in the bedroom:

A. Knack

B. Mayhem

C. Groggy

D. Dank

Explanation:  Mayhem means mess or disorder. The statement depicts that parents were not pleasantly surprised, so mayhem is the correct option.

187. The culpable child _______ some words to her mother for pardoning his delinquency.

A. Rude

B. Mumbled

C. Crazy

D. Showy

Explanation:  The sentence would be complete if we use a verb in the blank. Only mumbled is the verb among the options. 

Part – II:  In each of the following questions, four sentences are given. Choose the correct one and fill the circle corresponding to that letter in the MCQ response form.

188. A.   He asked, “Is your brother home?”

B. He asked “Is your Brother home?”

C. He asked, “Is your brother home”?

D. He asked “Is your brother home?”.

189. A. I was been to America for medical check up

B. I had been to America for medical check up

C. I have been to American for medical check up

D. I has been to America for medical check up

190. A. After breaking the glass, Ruby said “Please don’t tell on me.”

B. After breaking the glass ruby said: “Please don’t tell on me.”

C. After breaking the glass, Ruby said: “Please don’t tell on me.”

D. After breaking the glass Ruby said: please don’t tell on me.

191. A.   It is healthful to eat a variety of food

B. It were healthful to eat a variety of food

C. It is healthful to ate a variety of food

D. It were healthful to ate a variety of food

192. A. We hadn’t the foggy notion of the worker who tried to spoil the company’s reputation.

B. We hadn’t the foggiest notion of the worker who tries to spoil the company’s reputation.

C. We hadn’t the foggiest notion of the worker whom tried to spoil the company’s reputation.

D. We hadn’t the foggiest notion of the worker who tried to spoil the company’s reputation.

193. A. We use to play football when we lived abroad.

B. We are used to play football when we lived abroad.

C. We used to play football when we lived abroad.

D. We have use to play football when we lived abroad.

Part – III: SPOT THE ERROR: in the following sentences some segments of each sentence are underlined. Your task is to identify that underlined segment of the sentence, which contains the mistake that needs to be corrected. Fill the circle corresponding to that letter under the segment in the MCQ Response Form.

194. She had one of those picture children often reproduced.

AB

 in advertising leaflets and the photogravure sections of the Sunday papers.

    C D

Ans: B

Explanation: Correction: She had one of those pictures children often reproduced. 

195. When the mother threatened to lock Aslam 

AB

in the attic, the mere thought for being confined made him breathless.

     C D

Ans: D

Explanation: Correction: preposition mistake, ‘thought of being confined’ is the correct phrase.

196. Let’s hurry. The bus is leaving to the last stop.

   A. B C D

Ans: D

Explanation: Correction: The bus is leaving for the last stop. 

197. Despite all my enthusiastic chain of effort, could not attained the desired results to be

     A B              C

 crowned with brilliant success.

        D

Ans: C

Explanation: Correction: I could not attain…… Always use first form with could. 

198. Education and economic progress along with  country towards quick development.

    A                         D

Ans: B

Explanation: Correction: ….. are the factors….

199. Ali and Irfan have to receive the guests at the reception of the hotel, while Amir have 

AB          C

to bring their bags from the cars.

D

Ans: C

Explanation: Correction: Amir has to …. Amir is singular, so has must be used.

200. Ruth was wondering what she could do for help but she did not 

  A B             C 

Ans: C

Explanation: Correction: She could not

NUMS MDCAT SOLVED PAST PAPER-2020

NUMS MDCAT SOLVED PAST PAPER-2020

For F.S.C. and Non-F.Sc. Students 

Time Allowed: 210 Minutes (3- ½ hours)

Instructions: 

i. Read the instructions on the MCQs Response Form carefully. 

ii. Choose the Single Best Answer for each question.

iii. Each correct Answer carries One Mark. There is No Negative Marking  

iv. Candidates are strictly prohibited from giving any identification mark except Roll. No. & Signature in the specified columns only. 

PHYSICS

1. In motion of satellites, necessary centripetal force is provided by:

A. Gravitational Force

B.Coulomb’s Force

C.Magnetic Force

D.Nuclear Force

Explanation:  The necessary centripetal force is provided by the gravitational force between the two objects. Coulomb’s force is experienced between charged particles, magnetic force is between magnetic objects and nuclear force is the forced due to the nucleus of an atom.

2. In ripple tank 40 waves pass through a certain point in one second. If the wavelength of the waves is 5cm, then find the speed of wave. 

A.2.7 m/sB.3 m/s

C.200 m/s D. 2 m/s

Explanation: The frequency of the wave is 40 Hz. So using the formula:

Where  is the velocity, f is the frequency and  is the wavelength in m. So, plugging in the values we get 

3. The product of frequency and time period is equal to:

A.2B.3

C.0 D. 1

Explanation: Frequency and time period are the inverse of each other, hence their product gives 1.

4. Trough of a wave acts as:

A. Concave lens B. Convex lens

C.Convex mirrorD.Plane mirror

Explanation:  When the waves pass through trough a dark fringe is observed. This happens when the light rays diverge. Thus, the trough act as a concave lens because light also converges when the rays pass through them.

5. In Doppler effect if listener moves toward a stationary source then:

A. Observed frequency is greater than original frequency

B.Observed frequency is less than original frequency

C.Observed frequency is equal to original frequency

D.Observed frequency is independent of original frequency

Explanation: The general equation for Doppler effect is given below:

So, in the above case the source is stationary, hence vs is 0 and the observed frequency is greater than the original frequency.

6. Refrigerator is an example of:

A.First law of thermodynamics

B. Second law of thermodynamics

C.Newton law of motion

D.Entropy

Explanation:  refrigerator works on the principle of removing heat from a cold reservoir by doing some work which is a modified statement of second law of thermodynamics

7. In a certain process, 400J of heat energy is supplied to system and at the same time 150J of work is done by the system. The increase in internal energy of system is _______.

A.150JB.300J

C. 250J D. 500J

Explanation:  The change in internal energy is given by the following formula

Where U in the internal energy, Q is the heat supplied and W is the work done. Plugging in the values we get  .

8. The rapid escape of air from a burst tyre is an example of:

A.Isothermal B. Adiabatic

C.IsobaricD.Isochoric

Explanation:  During adiabatic process no heat is lost or gain. So, the bursting of the tyre is so rapid that there is no time for the system to gain or lose any heat. 

9. The bicycle pump works on the basis of:

A. 1st Law of thermodynamics

B.2nd Law of thermodynamics

C.Law of conservation of energy

D.Law of entropy

Explanation:  When the bicycle pump operates it increases the energy of the gas because of the mechanical energy is provides. This causes the temperature of the gas to increase and thus expands the gas inflating the tyre.

10. Two positive point charges are placed 2m apart. The electric potential at mid-point due to these two charges will be___________:

A. Added to double B. Reduced to half

C.Remains same (no effect)

D.Cancel each other effect

Explanation:  the potential at any point due to a positive charge is given by

Where V is the potential, k is the coulomb’s constant, q is the charge and r is the distance. Half way between the charges correspond to a distance of 1 m in this example. This potential becomes

11. Which one of the following is the angle of projection of a projectile if its range is equal to its height? 

A.48o B. 600

C.900 D. 760

Explanation:  The range and height of a projectile is given by

Equating these two terms and factoring out the common terms we get

 = 4

12. The product of force and time is equal to

A.Angular momentum

B.Force

C. Change in momentum

D.Velocity

Explanation:  According to Newton’s second law

Which is the change in momentum

13. The time rate of change of linear momentum of a body is equal to:

A. Force B. Momentum

C.powerD.Acceleration

Explanation:  According to Newton’s second law

14. A 10 N force moves a body around a circular path of radius 50cm. What is work done in completing one revolution?

A.5 J B. Zero

C.31.42 JD.500 J

Explanation:  work done is given as the product of force and displacement and the cosine of angle between them. As in circular motion the angle between force and displacement is always 90 hence the work done will be 0.

15.3 Kg stone falls from 20m high platform. Find its falling speed at 10m height.

A.196 ms-1 B. 14 ms-1

C.10 ms-1 D. 100 ms-1

Explanation:  According to equation of motion   where v & u are the final and initial velocities, a is the acceleration constant and s is the displacement. Since initial velocity is 0 we get

Thus, v is 14 m/s  

16. The area under force – displacement graph gives us:

A.DisplacementB.Power

C. Work D. Acceleration

Explanation:  Work done is given by the product of force and displacement, hence area under the force displacement graph gives us the work done.

17. Kilowatt-hour is unit of?

A. Electric Energy B. Power

C.MomentumD.Torque

Explanation:  Energy is given as the product of power and time.

18. The food we eat in one day has about the same energy as:

A. 0.33 liter of petrol

B.1 liter of petrol

C.0.5 liter of petrol

D.2 liter of petrol

19. One complete circle is equal to:

A. 2π radian B. 3π radian

C.5π radianD.6π radian

Explanation:  π radians = 180° hence 2π radians = 360°

20. The direction of current through the load resistance of a full-wave rectification circuit:

A.inverts for negative cycle

B.changes for every cycle

C.Invert for positive cycle

D. remains constant

Explanation:  The following diagram shows a full wave rectifier

The direction of I is same for both the positive and negative cycles. When the sine wave is positive diodes D1 and D2 conduct and when it is negative diodes D3 and D4 conduct keeping the direction of current same.

21. For gaining an atomic spectra, an evacuated glass tube is filled with:

A. Neon B. Hydrogen

C.Carbon dioxideD.Sulphur dioxide

Explanation:  Noble gases are used because of their inertness.

22. During production of x-rays the cathode and anode are enclosed inside and evacuated glass chamber and high DC voltage of the order of:

A.1000 V is maintained 

B.10,000 V is maintained

C.25,000 V is maintained

D. 50,000 V is maintained

Explanation:  Fact

23. Half-life of iodine-131 is 8 days. If 20mg is present initially, how much iodine is left behind after 2 half-lives?

A.10 mg B. 5 mg

C.2.5 mgD.1.25 mg

Explanation:  After first half-life the amount remaining will be 10mg. After second half-life the amount remaining will be 5mg.

24. 4.5 x 109 years is the half-life of:

A.U234 B. U235

C. U238 D. C14

Explanation:  Fact

25. When the voltage “V” on a capacitor is doubled then energy stored “U” will:

A.2 UB.3 U

C.U / 2 D. 4 U

Explanation:  Energy of a capacitor is given by

Doubling the voltage increases the energy by 4 times.

26. By increasing area of the plates and decreasing distance between them, the capacitance of capacitor: 

A. Increases B. Decreases

C.Remains unchanged

D.Depending upon temperature

Explanation:  Capacitance is given by the formula 

Where  is the permittivity of dielectric, A is the area of the plates of the capacitor and d is the distance between them.

27. If we double the separation between two charges then coulomb’s force will become?

A.DoubledB.Half

C.4-times D. 1/4th

Explanation:  Coulomb’s force is given by

Where k is the coulomb’s constant q1 and q2 are the charges and r is the distance between them. If the distance is doubled then the force decreases by one-fourth.

28. The power of an electric bulb is 100W. It is connected to 110V power supply. The resistance of electric bulb will be?

A.11 ohm B. 121 ohm

C.20 ohmD.200 ohm

Explanation:  Power across a resistor is given by 

Plugging in the values we get 12100/100 = 121ohms.

29. Terminal voltage “Vt” of the battery is greater than emf of the battery when:

A. Battery is charging

B.Battery is discharging

C.Battery is connected with R

D.Battery is connected with voltmeter

Explanation:  When the battery is charging the direction of the current reverses and the terminal voltage is greater than emf. The general equation for a discharging battery is given by

Where R is the internal resistance.

Now when the current reverses the same equation becomes

30. The temperature coefficient of semi-conductor is negative because:

A.Resistance increases with increase of temperature

B. Resistance decreases with increase of temperature

C.Resistance decreases with decreases of temperature

D.Resistance remains same with increase of temperature

Explanation:  When a semiconductor is heated, the number of free electrons increases because of the extra energy, free electrons jump to the conduction band to conduct electricity hence the resistance decreases.

31. If length of the wire becomes two times to its original value and area becomes one half to its original value then resistance of the wire becomes:

A.Double B. Four times

C.One halfD.One fourth

Explanation:  Resistance of a wire is given by

where is the resistivity of the material, L is the length and A is the area. So the resistance increases four times if the length is doubled and area is halved.

32. The unit of resistivity is:

A.ohm B. ohm meter

C.ohm / meterD.meter / ohm

Explanation:  Fact

33. 1 kilowatt hour =

A.1.6 x 1019 J B. 3.6 x 106 J

C.9.1 x 1031 J D. 1.67 x 1027 J

Explanation:  Energy is given by

    = 3.6 x 106J

34. It is a null type resistance device for measuring potential differences:

A.GalvanometerB.Ohmmeter

C.Ammeter D. Potentiometer

Explanation:  It is the property of this device

CHEMISTRY

35. Which statement is true about electron affinity?

A.The value of electron affinity is always positive

B.The value of electron affinity is always negative 

C.The value of first electron affinity is always positive

D.The value of first electron affinity is always negative

Explanation:  Electron affinity is the change in energy of the atom when an electron is added to its outermost shell. When first electron is added, energy is lost, so first electron affinity is always negative. 

36. The bond which is based on attractive forces between oppositely charged ion is:

A.Covalent bondB.Dative bond

C.Ionic bond D. Metallic bond

Explanation:  Covalent bond is formed due to sharing of half filled electron orbitals. Ionic bond is formed due to attractive forces between oppositely charged ions.

37. Which statement is incorrect regarding a chemical bond?

A.Bond is formed by the overlapping of half-filled orbitals

B.Bond is formed by the attraction of positive and negative ions

C.Bond is formed by the overlapping of “s” orbital is strong

D.Bond formed by the large sized atoms is strong

Explanation:  All the statements explain bonding except the last option. Bond formed by large sized atoms is easy to break.

38. The carbonates of alkali metals are soluble in water except:

A.K2CO3 B. Li2CO3

C.NA2CO3 D. Rb2CO3

Explanation:  Solubility of alkali metal carbonates in water increase down the group. Lithium is the first member of the group, its carbonate is insoluble in water. 

39. The nitrides of alkaline earth metals hydrolyze with water to form:

A.NH3 B. H2

C.N2 D. NO

Explanation:  Ca3N2 hydrolyses to give ammonia and calcium hydroxide.

40. The flame colour of Ca in flame test is:

A.Orange red B. Golden yellow

C.RedD.Pink

Explanation:  Sodium gives yellow flame. Barium gives apple green. Rubidium gives red. 

41. Which of the following is the most stable metal carbonate?

A. BaCO3 B. MgCO3

C.CaCO3 D. SrCO3

Explanation:  On going down the group, the electropositive character of alkaline earth metal increases and stability also increases.

42. An organic sample has the molecular formula CxHyOz in the simplest ratio of 1 : 1.33 : 1. The Mr of the compound is 176. Which could be the value of Z?

A. 6 B. 8

C.2D.3

Explanation: First of all, multiply by 3 to get the ratios in whole numbers. We get, 3 : 4 : 3 

Now calculate the Mr of the empirical formula. We get, 3(12) + 4(1) + 3(16) = 88. Now calculate the value of n by dividing the Mr of the molecular formula with the Mr of the empirical formula. 

We get, n = 2. Hence, the molecular formula becomes C6H8O6. So, the value of z is 6.

43. Isomerism due to shifting of proton from one atom to another in a same molecule is known as:

A.Metamerism B. Tautomerism

C.PositionD.Functional

Explanation:  Definition of tautomerism.

44. ISO-Butyl alcohol has following carbon attached to hydroxyl group:

A.Tertiary B. Secondary

C.QuaternaryD.Primary

Explanation:  Isobutyl alcohol is secondary alcohol with the formula (CH3)2CHCH2OH. It is also called secondary butanol.

45. Oxidation of alcohol gives:

A.Amines B.Alkanes

C.Aldehyde D. Alkynes

Explanation:  Alcohol oxidises to give aldehyde or ketone.

46. Iodine is a solid at rtp. Which of the following categories of solids does Iodine belong to?

A. Molecular solid B. Covalent solid

C.Metallic solidD.None of the above

Explanation: Iodine is categorized as molecular solid because I2 molecules are held together by van der Waals’ forces in a solid structure. Though iodine is covalently bonded, it is not defined as covalent solid because covalent solids are of two types: 

1. When covalent bonds join to form giant molecules like diamond.

2. When atoms covalently bond together to make layers as that in graphite. 

Iodine falls in none of these categories, hence, it cannot be called a covalent solid

47. Hydrolysis of Nitriles produces:

A. Carboxylic acid B. Aldehydes

C.KetonesD.Esters

Explanation:  Nitriles are organic compounds containing cyano group (CN). They hydrolyze to give carboxylic acid. Amide is formed first which hydrolyzes to form carboxylic acid. 

48. Acetic anhydride is a product of acetic acid, as a result of the following reaction:

A. Dehydration B. Reduction

C.OxidationD.Esterification

Explanation:  Dehydration of acetic acid at 800℃ gives acetic anhydride. CH3COOH changes to (CH3CO)2O.

49. Which of the following enzyme is raised in rickets?

A.Lactic dehydrogenase

B.LDH-I

C.Phosphatase

D.Alkaline phosphatase

Explanation:  Rickets is the disease which rises due to deficiency of vitamin D. Bones get weak and soft. The enzyme ALP rises in this condition.

49. For a gaseous reaction, the increase in pressure will shift the equilibrium in a direction of:

A.Decreased concentration

B.Increased concentration

C. Decreased volume

D.Increased volume

Explanation:  If we increase pressure (volume will be reduced) equilibrium will shift to the right direction because less volume will be required for the product and vice versa.

50. Acidic buffer consists of:

A.Strong acid and salt of it with a weak base

B. Weak acid and salt of it with a strong base

C.Strong acid and salt of it with a strong base

D.Weak acid and salt of it with a weak base

Explanation:  Acidic buffer is formed by the addition of weak acid to its salt with a strong base. Basic buffer is formed by weak base and conjugate base of weak acid or conjugate acid of weak base. 

51. The pH of human blood is maintained between:

A.7.35 to 7.45 B. 7.55 to 7.65

C.7.00 to 7.25D.7.85 to 7.95

Explanation:  Fact

52. The buffer solution is not formed for:

A.NH4OH + NH4Cl

B.CH3COOH + CH3COONa

C.C6H5COOH + C6H5COONa

D. HCl + NaCl

Explanation:  HCl is the strong acid. Buffer solution is not formed by strong acid or base.

53. In the reaction

H2 + CO2 H2O + CO

The decrease in the concentration of CO2 will shift equilibrium:

A. Towards left B. Towards right

C.Nothing happens to the equilibrium

D.Equilibrium will shift towards both the directions

Explanation:  When the concentration of a particular substance is decreased, equilibrium is disturbed and reaction proceeds towards its direction to achieve equilibrium again.

54. At equilibrium the concentration of reactants and product become:

A.ZeroB.Equal

C.Constant D. Infinite

Explanation:  Definition of equilibrium. 

56. Which of the following is false about the common ion effect? 

A.If the common ion is present in a solution, the equilibrium shifts from right to left.

B.If the common ion is present in a solution, the ionization of weak acid becomes less.

C. If the common ion is present in a solution, more H+ ions are produced.

D.The pH value of the solution with common ions is higher than a solution without a common ion.

Explanation: If the common ion is present in a solution, more H+ ions are produced. The salt of a weak acid is a strong electrolyte. The salt will completely dissociate as soon as it contacts the solution with its common ion while weak acid on the other hand will only ionize slightly. 

Applying Le Chatelier’s principle, adding more common ions to the salt will suppress the ionization of weak acid shifting the equilibrium from right to left. In this case, less H+ ions will be generated and therefore, the pH value will tend to go high or less acidic.

57. In a reversible reaction, catalyst lowers the activation energy of the:

A.Forward reactionB.Reverse reaction

C. Forward as well as reverse reaction

D.Forward reaction but increases that of the reverse reaction

Explanation:  Catalysts work by lowering activation energy. In case of a reversible reaction, activation energy of both directions is decreased.

58. The rate of reaction:

A.Increase as the reaction proceeds

B. Decreases as the reaction proceeds

C.Remains the same as the reaction proceeds

D.May decrease or increase as the reaction proceeds

Explanation:  As the reaction proceeds, more reactants are converted into products. The concentration of reactants is decreased and the reaction slows down. 

59. 0.5 molar solution NaOH contains:

A.40g NaOH in one dm3

B.80g NaOH in one dm3

C.10gm NaOH in one dm3

D. 20g NaOH in one dm3

Explanation:  One molar NaOH contains 40g of NaOH in 1 dm3 of water. So, 0.5 molar solution must contain 40/2 g of NaOH i.e., 20g. 

Molarity is defined as the number of moles of a solute in one dm3 or liter of solution. 

60. The breakdown of a substance with current is:

A.ThermolysisB.Catalysis

C.Electrolysis D. Photolysis

Explanation:  Definition of electrolysis. Thermolysis is breakdown of substance in the presence of heat. Catalysis is increasing the rate of reaction by the addition of catalyst. Photolysis is the breakdown of substance in the presence of light. 

61. Which of the following is balanced redox equation?

A.Na + Fe3+ Na1+ + Fe

B.Zn + Ag1+ Zn2+   + Ag

C. 3Na + Fe3+ 3Na1+ + Fe

D.2Zn + Ag1+ 2Zn2+ + Ag

Explanation:  Balance the equation. In correct option, 3 atoms of sodium lose 3 electrons to convert ferric ion to atom. Rest of the equations are not balanced. 

62. Stronger is the oxidizing agent, greater is the:

A.Oxidation potential

B. Reduction potential

C.Redox potential

D.EMF of cell

Explanation:  Oxidizing agent oxidizes the other substance and gets itself reduced. So oxidizing agent has greater reduction potential.

63. Type of bonding in Sodium (Na) is:

A. Metallic B. Ionic

C.Covalent

D.Co-ordinate Covalent

Explanation:  In metallic bonding, electrons become delocalized in the crystal lattice. 

64. Which of the following Halogen molecules has maximum bond energy?

A.F-F B. Cl-Cl

C.Br-BrD.I-I

Explanation:  Bond energy of halogen molecules decreases down the group. Chlorine has maximum bond energy than bromine and iodine molecules that come later in the group. Fluorine is on the top of the group but has lesser bond energy due to its smaller size which result in high inter electronic repulsions. 

65. Half atmospheric pressure is:

A.400 torrB.50662 pa

C.101.3 PaD.8.5 pounds

Explanation:  1 atm = 760 torr or 101325 

66. The values of S.T.P for 1 mole of any ideal gas is:

A.273.16 K & 1 atm

B.00C & 1 mm Hg

C.`273 160C & 1 atm

D.0k & 1 atm

Explanation:  STP stands for standard temperature pressure. Standard temperature is 0℃ or 273.16 K. Standard pressure is 1 atm.

67. The expression PV = nRT represents the:

A.Dalton’s lawB.Avogadro’s law

C. General gas equation

D.Vander Waal’s equation

Explanation:  General gas equation is based on 3 laws; Charles law, Boyle’s law, and Avogadro’s law. It explains the relationship of volume of gas with temperature, pressure. and amount of gas respectively. 

68. Pressure remaining constant, at which temperature volume of gas will become twice to the volume at 00C

A.5460C B. 2000C

C.5460C D. 2730C

Explanation:  Volume of gas is directly proportional to the temperature. At 00C or 273.16 K, volume is V. Volume will become double at 546 K which is equal to 2730C.

69. What is not true for effusion of gases?

A. Movement of particles through small opening

B.Movement of particles from high pressure to low pressure

C.Movement of paricles due to escaping tendency one by one 

D.Movement of particles due to collision among themselves

Explanation:  Definition of effusion

69. Upon which factor vapour pressure is dependent:

A.Temperature

B.Intermolecular forces

C.Density of liquid

D.Surface area of liquid

Explanation:  Vapor pressure depends only on temperature of the substance, and type of liquid. 

70. Solid water is expanded ______ times when it is compared with same volume of liquid water:

A.9B.5

C.6D.2

Explanation:  When water converts into ice, its intermolecular forces become weak and molecules begin to move farther. So, the volume is increased and density is decreased. 

71. Molar heat of vaporization is the amount of heat required to convert one mole of:

A.A liquid into its vapours at its boiling point

B.Liquid into its vapours

C.Solids into vapours

D.Solid into liquid at its melting point

Explanation:  Definition.

72. At transition temperature of crystalline solid, substance exists:

A.In most stable geometrical form

B.Solid and liquid state

C. In dynamic equilibrium between two crystalline forms

D.In one solid geometrical form only

Explanation:  It is the temperature at which substance can co exist in two crystalline forms at equilibrium. 

73. Some substances lack definite heats of fusion, these substances are:

A.IsomorphsB.Polymorphs

C.Amorphous solids

D.Crystalline solids

Explanation:  Amorphous solids do not have regular pattern or arrangement of atoms or molecules. They begin to flow without melting completely. So, they do not have definite heat of fusion. Crystalline solids have definite heat of fusion.

74. Thermal conductivity of metals is due to:

A.Layered structure of metals

B.Freely moving electrons

C.Loosely held metal atoms

D.Vibrational movement of metals

Explanation:  The electrons in metal lattice are delocalized. When heat is applied, the move more rapidly, and pass energy more quickly. 

75. Ice floats on the surface of water due to:

A.larger bond length

B.Cubic structure of ice

C.Weak intermolecular forces

D. Empty spaces in the structure of ice

Explanation:  The density of ice is less than the density of liquid water, owing to the empty spaces in the structure of ice, so ice floats on water. 

76. When the number of moles of reactants and products are equal in reversible reactions, which parameter would not affect at equilibrium?

A.TemperatureB.Pressure

C.VolumeD.Catalyst

Explanation:  When equilibrium is achieved, the change in temperature, pressure, and volume of the reactants or products may start the reaction again to achieve equilibrium again but at this point catalyst does not affect if reaction is already stopped. 

77. By which of the following factors equilibrium state is attained earlier?

A.Temperature B.Pressure

C.ConcentrationD.Catalyst

Explanation:  Catalyst speeds up the reaction by lowering activation energy and equilibrium is achieved readily.

78. Many elements have fractional atomic masses. This is because:

A.Mass of atom is itself fractional

B.Atomic masses are average masses of isobars

C.Atomic masses are average masse of isotopes

D. Atomic masses are average masses of isotopes proportional to their abundance

Explanation:  Some elements have isotopes, so their mass is calculated keeping in view different isotopes and their relative abundance. 

79. Mass of 1 molecule of O2 

A.6.02 x 1023g / 32 B. 32 / 6.02 x 1023 g

C.32gD.0.32g

Explanation:  Mass of 6.02 x 1023 molecules (or one mole) of oxygen gas is equal to 32g. To calculate mass of 1 molecule, molar mass must be divided with Avogadro’s number. 

80. 1 mole of any substance contains_______ particles:

A.6.02 x 1023 B. 6.02 x 1024

C.6.02 x 1022 D. 3.01 x 1023

Explanation:  According to Avogadro’s law. 

81. The number of moles of CO2 which contain 8.0 g of oxygen are: 

A.1.0B.4.50

C.0.50D. 0.25

Explanation:  One mole of carbon dioxide contains 32g of oxygen. 0.25 moles must contain 0.25 x 32 g of oxygen which is equal to 8g.

82. Identify the correct option with same empirical formula for both compounds:

A.H2O & H2O2 B. C6H12 & C6H6

C.H2S2O3 & H2SO4

D. C6H12O6 & CH3COOH

Explanation:  The empirical formula of both glucose and acetic acid comes out to be CH2O.

83. What are the Avogadro’s number of particles in 0.25 moles of CO2?

A.6.02 x 107 B. 1.505 x 1023

C.2.00 x 1023 D. 1.505 x 1015

Explanation:  1 Mole of carbon dioxide has 6.02 x 1023 number of particles. 0.25 moles must have 0.25 x 6.02 x 1023 number of particles which is equal to 1.505 x 1023.

84. The conversion of substance X to substance Y is an endothermic process. This shows that:

A. X is more stable than Y

B. Y is more stable than X

C. They have equal stability

D. Cannot be predicted from this information

Explanation: X requires heat to form Y, which means that the formation of X is more exothermic as compared to Y. 

More exothermic formation means more stability.

85. Which of the following fundamental particles have the same mass/kg?

A.Electron, neutronB.Electron, proton

C. Proton, neutron D. Neutron, proton

Explanation:  Proton and neutron have same mass.

86. The lightest positive rays obtained is from:

A. Hydrogen gas B. Helium

C.NeonD.Air

Explanation:  Hydrogen is the lightest element. It produces lightest positive rays by losing its electron to produce proton. 

87. The amount of energy associated with quantum of radiation is directly proportional to:

A.PhotonB.Wavelength

C.Frequency D. Velocity

Explanation:  Energy of radiation is directly proportional to the frequency and inversely proportional to wavelength. 

88. X-rays are defined as:

A.Electromagnetic radiations of high mass number

B.Electromagnetic radiations of very high frequency 

C.Electromagnetic radiations of high wavelength

D.Electromagnetic radiations of high energy

Explanation:  X rays are electromagnetic waves with high frequency and low wavelength. 

89. Which of the following orbital will be filled first than 4 p?

A.4 sB.2 p

C.3 d D. 1 s

Explanation:  Fact

90. Maximum _______ electrons can be placed in one orbital:

A.1B.2

C.3D. 4

Explanation:  One is clockwise and the other is anticlockwise.

91. Mass of electron in a.m.u is:

A.1.0073B.1.0087

C. 5.485 x 10 -4 D. 9.1 x 1034

ENGLISH

92. Identify the errors and choose the correct option:

The rehearsal session started and we have little times to spare for other activities

A. The rehearsal session started and we have little time to spare for other activities

B.The rehearsal session started and we little time to spare for other activities

C.The rehearsal session starts and we has little time to spare for other activities

D.We are little time to spare for other activities. the rehearsals session starts.

Explanation: Options B to D are grammatically incorrect. The word ‘times’ used in the original sentence is wrong. When time is discussed as an abstract and unspecified notion, then it is written as ‘time’. However, when it is specified, ‘times’ is used (for example, “you punched me three times”).

93. Fill in the blank with appropriate option:

Lions, like any other carnivore, _________ on meat.

A. Live B. Lives

C.Does liveD.Living

Explanation:  The correct option is A, because ‘live’ is used with a plural noun, whereas ‘lives’ is used with singular nouns.

‘Living’ is grammatically incorrect in the sentence. ‘Does live’ implies that an alternate way of living also exists, which is not correct in the context of this sentence.

94. Fill in the blank with the appropriate option:

The cattle _____ away the crops.

A.Has eaten B.Is eating

C. Have eaten D. Have been eating

Explanation:  The correct option is ‘C’ because cattle is a plural. The correct verb form is ‘have eaten’, which shows that the cattle have collectively eaten the crops, which is in the past tense.

95. The word ‘LABYRINTH’ means:

A. Maze B. Heap

C.HiveD.Knack

Explanation:  The correct answer is Maze. 

Labyrinth means a confusing network of paths that appear as a puzzle.

Maze is a network of paths and hedges designed as a puzzle.

Heap means a collection of objects stacked together with no particular order.

Hive means a place in which people are busily occupied.

Knack means an acquired or natural skill at doing something.

96. Pick the correct option:

These are old those are new.

A.These, are old, those are new.

B.These are old; those are new.

C.These are old: those are new.

D.These are old — those are new.

Explanation:  D is the correction form of punctuation.

A dash is commonly used to separate a group of words or phrases.

97. Ahmed carried out his duty according ______ instructions.

A.Too B. To

C.UnderD.An

Explanation:  Option B is the correct preposition to use in this sentence. Under is used to signify the presence of the object beneath something.

‘Too’ is used to signify the presence of multiple variables.

‘An’ is incorrect because instructions is in plural form, and is not a noun.

98. Identify the errors and choose the correct option:

The first space traveler was Dennis Tito from united states.

A. The first space traveler was Dennis Tito from the United States

B.The first space traveler was Dennis Tito from united states

C.The first space traveler was Dennis Tito from united states

D.The first space traveler was Dennis Tito from united states

Explanation:  Option A is correct because it has the correct punctuation and grammar. Also, because it referes to United States as ‘the United States’ which denotes or points to a definitive noun.

99. Select the right sentence.

A. He opened the square red wooden box.

B.He opened the red square wooden box.

C.He opened the wooden red square box.

D.He opened the red wooden square box.

Explanation: When multiple adjectives are used, we write them in the following order: opinion, size, physical quality (e.g. rough, tidy etc), shape, age, colour, origin (Japanese, German etc), material, type (general purpose, u-shaped etc), and purpose.

100. Fill in the blank.

I can’t walk________.

A.FartherB.Far

C. Further D. Away

Explanation:  The correct option is C, which means that the subject cannot walk any further, past a certain point. Far means an estimated distance, which is grammatically incorrect. Farther means over a large expanse of space or time, which is also incorrect.

101. Can you tell this fact _________ his face?

A.To B.On

C.UponD.At

Explanation:  The correct answer is A. To say something ‘to’ one’s face implies that something is said directly to someone in their presence.

102. Choose the correct option.

A. The Three Musketeers was written by Dumas.

B.The Three Musketeers were written by Dumas.

C.The Three Musketeers has written by Dumas.

D.The Three Musketeers have written by Dumas.

Explanation:  The grammatically correct option is ‘A’, which indicates that a novel, The Three Musketeers, which is singular, was written by the author, Dumas.

103. They have painted their house purple. The sentence is an example of:

A.MonotransitiveB.Ditransitive

C. Complex transitive

D.Reflexive transitive

Explanation: A sentence with a complex transitive verb has a direct object (in this case, the house) and an object compliment, which identifies a quality or attribute pertaining to the direct object (in this case, the purple colour of the house).

104. Select correct option:

He was killed _______ the robbers______ a hatchet.

A.From, withB.By, at 

C.Through, for D. By, with

Explanation:  The correction option id ‘D’, which correctly indicates that the subject was killed ‘by’ the robbers, ‘with’ a hatchet.

105. Choose the correct option.

A.“Well, no, perhaps not sir”

B. “Well, no. perhaps not sir”

C.“Well, no perhaps not sir”

D.“Well, no perhaps, not sir”

Explanation:  Option B is correct. The punctuation used in the sentence correctly separates the words to give them meaning.

106. Find out the Antonym of “Mumbled”.

A.UnprovokedB.Quiver

C. Loud D. Rarely

Explanation:  The correct option is C.

Rarely means not very frequently.

Quiver means to tremble or shake with a slight rapid motion.

Unprovoked means an attack done or made for no good reason.

BIOLOGY

107. Negative feedback mechanism is the characteristic of which class?

A.Class FishB.Class Amphibia

C.Class ReptiliaD.Class Mammalia

Explanation:  Feedback mechanism 

Positive feedback mechanism 

Stimulus and homeostasis carry out in the same direction. 

Negative feedback mechanism 

Stimulus and homeostasis carry out in the opposite direction. 

Example: 

Temperature of the environment increases. 

Receptors detect the change in temperature. 

Control center responds to the change in temperature. 

Thermoregulation mechanism is activated. 

Different mechanisms occur to lower the temperature. 

108. The function of papillary muscles is to:

A.Move blood from semilunar valve into pulmonary vein

B.Prevent the backward flow of blood from the ventricle

C.Push the blood from right atrium to left atrium

D.push the blood from left atrium to aorta

Explanation:  Papillary muscles are present in the ventricles and are attached to tricuspid valve through cordae tendineae. They prevent backward flow of blood from ventricles.

109. Choose the correct pathway for the flow of blood:

A. Arterioles —- metarterioles — thoroughfare channel — capillaries 

B.Arterioles — through fare channel —- metarterioles —    capillaries

C.Through fare channel — arterioles — capillaries — metarterioles

D.Metarterioles —- arterioles —- through fare channels — capillaries

Explanation:  Arteries divide into arterioles and then into metarterioles which empty into capillaries through fare channels. 

110. Intrinsic factor is secreted by:

A.PancreasB.Liver

C.Stomach D. Duodenum

Explanation:  Intrinsic factor is secreted by chief cells of gastric mucosa. It is a glycoprotein and aids in absorption of vitamin B12.

111. Gaseous exchange in plants takes place through the:

A.Stomata B. Mesophyll

C.EndodermD.Xylem

Explanation:  Stomata, also called guard cells, are involved in gaseous exchange in plants. 

112. Translocation of organic solutes in plants takes place through:

A.Companion cellB.Fibres

C.Sieve tubes D. Vessels

Explanation:  While movement of water and minerals through the xylem is driven by negative pressures (tension) most of the time, movement through the phloem is driven by positive hydrostatic pressures. This process is termed translocation, and is accomplished by a process called phloem loading and unloading.

Cells in a sugar source “load” a sieve-tube element by actively transporting solute molecules into it. This causes water to move into the sieve-tube element by osmosis, creating pressure that pushes the sap down the tube. In sugar sinks, cells actively transport solutes out of the sieve-tube elements, producing the exactly opposite effect.

113. The only vein in the human body carrying oxygenated blood is:

A.FemoralB.Pulmonary

C.RenalD.Iliac

Explanation:  All veins carry deoxygenated blood towards heart except pulmonary vein. All the arteries carry oxygenated blood away from heart except pulmonary artery.

114. The cells which play very important role in developing immunity are:

A.MonocytesB.Neutrophils

C.Lymphocytes D. Thrombocytes

Explanation:  Lymphocytes are involved in immunity development. Thrombocytes are involved in clot formation. Neutrophils protect us from infections. Monocytes also fight infections and regulate immunity. 

115. Which of the following blood vessels have the highest pressure of blood?

A.Aorta B. Pulmonary arteries

C.Pulmonary veinsD.Vena Cava

Explanation:  Aorta receives oxygenated blood before it distributes to the whole body. As it receives blood in high quantity, so blood pressure is also high here. 

116. Autoimmune diseases act at the principle of:

A.Self against antigens

B.Antigen against self

C.Self against self

D.Antigen self-destroyed

Explanation:  In autoimmune diseases, the immune system starts to act against the body.

117. Which organ is called as the body’s thermostat?

A.Pituitary glandB.Kidneys

C.Hypothalamus D. Adrenal gland

Explanation:  Hypothalamus regulates the body temperature and acts as thermostat. 

118. The uptake of sodium in the ascending limb of loop of Henle is controlled by:

A.Aldosterone B. ADH

C.Glucosterone

planation:  Aldosterone is produced by adrenal cortex. It reabsorbs sodium along with water from ascending loop of Henle and hence, plays role in blood pressure regulation.

119. The multinucleated mass of the bone forming cells is called:

A.OsteoclastsB.Osteoblasts

C.OsteogenicsD.Osteocytes

Explanation:  Osteoblasts are bone forming cells. Osteoclasts are bone dissolving cells. Osteocytes are mature bone cells. 

120. Chief material present in the cell walls of plants, fungal and prokaryotic cells are:

A.ProteinsB.Lipids

C.Polysaccharides D. PPhospholipids

Explanation:  Fungal cell wall is made of chitin, plat cell wall is made of cellulose, and bacterial wall is made of peptidoglycan. All of these have polysaccharides in them. 

121. Which type of leucoplasts store lipids?

A.AmyloplastB.Elaioplast

C.ProteinnoplastD.Etioplast

Explanation:  Amyloplasts store starch, elaioplast stores lipids while etioplasts are not exposed to light, they are present in angiosperm plants that grow in dark.

122. Which type of movement though cell membrane is not energy consuming process?

A.EndocytosisB.Exocytosis

C.Active transportD.Osmosis

Explanation:  Osmosis occurs without expenditure of energy. It is diffusion of water. Endocytosis and exocytosis are active transport mechanisms which require energy. 

123. Cholesterol molecules in plasma membrane are present in________.

A.Outer membrane of phospholipid

B.Inner membrane of phospholipid

C.Both layers of phospholipid

D.Between bilayers of phospholipid

Explanation:  Phospholipid bilayer is arranged in such a way that the lipid portion is present in the center. The cholesterol molecules are found attached with the lipid portion. 

124. Fibers of extracellular matrix are attached to ___ plasma membrane: 

A.PhospholipidsB.Carbohydrates

C.GlycolipidsD.Proteins

Explanation:  Fact

125. __________ organelles involve in the synthesis of plant cell wall:

A.Endoplasmic reticulum

B. Golgi complex

C.Lysosomes

D.Peroxiosomes

Explanation:  Complex polysaccharides of cell wall are synthesized at Golgi complex.

126. Select the pair of organs which contains a large number of mitochondria:

A.Stomach & LiverB.Muscle & Stomach

C.Heart & Liver D. Liver & Muscle

Explanation:  Liver cells are highly generative, so they need a lot of energy. Muscle cells contract and relax, so they demand more energy. That’s why mitochondria are present in both kinds of cells in large amount.

127. Which of the following cells does not have nucleus?

A.Muscle cellB.Nerve cell

C.White Blood cellD.Red Blood cell

Explanation:  Fact

128. Most abundant organic compounds in mammalian cell are:

A.WaterB.Lipids

C.CarbohydratesD.Proteins

Explanation:  Proteins are present everywhere on the body. Enzymes are proteins, blood has proteins in it, hemoglobin is a protein. Many hormones are protein in nature. 

129. Second most abundant bio element in human body is:

A.OxygenB.Carbon

C.HydrogenD.Nitrogen

Explanation:  Most abundant element in human body is oxygen. Second most abundant element is carbon. Carbon is the constituent of all organic molecules. 

130. Lecithin is formed by combining phosphatidic acid with ________:

A.SerineB.Choline

C.InositolD.Ethanolamine

Explanation:  Lecithin is a kind of phospholipid which is composed of glycerol, 2 fatty acids, phosphate group, and choline. 

131. NAD is an example of ___________.

A.MononucleotideB.Dinucleotide

C.Tri nucleotideD.Tetra nucleotide

Explanation:  NAD stands for nicotinamide adenine dinucleotide.

132. What would be the number of nucleotides for a protein molecule of about 142 amino acids?

A.430B.142

C.426 D. 460

Explanation:  Three nucleotides code for one amino acid. So for 142 amino acids, there should be 142 x 3 nucleotides.

133. The basic structural framework of all types of membranes are:

A.GlycolipidsB.Glycoproteins

C.Lipoproteins D. Nucleoproteins

Explanation:  Lipoproteins transport hydrophobic structures through the membranes. 

134. Non-Protein but inorganic detachable co-factor is called ______:

A.Activator B. Prosthetic group

C.Co – enzymeD.Apo-enzyme

Explanation:  Definition

135. When inhibitor binds to enzyme other than active site and alters its structure, then it is called:

A.competitive inhibitor

B. Non- Competitive inhibitor

C.Reversible inhibitor

D.Irreversible inhibitor

Explanation:  When inhibitor occupies active site, then it is called competitive inhibition. When inhibitor occupies a site other than active site then it is called noncompetitive inhibition. 

136. Cyanides are potent poisons of living organism and can kill by inhibiting _______ essential for cellular respiration:

A.Cytochrome oxidases

B.Dehydrogenases

C.HydrolasesD.Nucleases

Explanation:  Cyanide acts as irreversible enzyme inhibitor by binding to iron atoms of cytochrome oxidase in the mitochondria. 

137. During feedback inhibition, which of the following structural part of enzyme is involved?

A.Active siteB.Binding site

C.Catalytic siteD.Allosteric site

Explanation:  During feedback inhibition, an inhibitor binds to allosteric site of the enzyme and changes the shape or conformation of enzyme such that substrate is unable to bind the active site and the enzyme is inhibited. 

138. The microscopic gaps between successive neurons is called?

A.Node of ranvier B. Synapse

C.Nerve impulse D.Arc

Explanation: Fact The space between successive neurons is called synapse.

139. If another molecule, having a shape very similar to the enzyme’s substrate, binds to its active site, it would then ______ the enzyme’s function:

A.FastenB.Inhibit

C.ReverseD.Decrease

Explanation:  In this way active site is occupied and substrate has no site to bind to the enzyme and enzymes function is inhibited. This type of inhibition is called competitive inhibition.

140. Myofibrils within the muscle fibers contain thick and thin filaments made up of ___ and _______ respectively”

A.Myosin and Actin

B.Globulin and Albumin

C.Troponin and Tropomyosin

D.Fibrin and Fibrinogen

Explanation:  Actin and myosin are muscle proteins that form filaments which are part of muscle fibers. Actin forms thin filament and myosin forms thick filament. 

141. Vertebrae of the neck are called:

A.Coccygeal vertebrae

B.Cervical vertebrae

C.Sacral vertebrae

D.Lumbar vertebrae

142. Which vertebrae together are called pelvic vertebrae?

A.Coccygeal and lumber 

B.Sacral and lumber

C.Sacral and coccygeal

D.Sacral and thoracic

Explanation:  Sacrum and coccyx fuse to form pelvis.

143. A non-protein cofactor tightly bound to the enzyme through covalent bond 

A. Activator B.Coenzyme

C. Prosthetic group D. Apo-enzyme 

Explanation: Prosthetic groups are cofactors that bind tightly to proteins or enzymes. They can be organic or metal ions. 

Apo-enzyme is an inactive enzyme which is activated through the binding of cofactor or coenzyme. Activators are molecules that increase the activity of enzymes.

144. Within the chromosomes, each chromatid contains ___DNA molecule: 

AOne B. Two

C.ThreeD.Half

Explanation:  Each chromosome has 2 chromatids and every chromatid has only one molecule of DNA that stores genetic information. 

145. Darwin’s theory is also known as:

A. Theory of independent assortment

B. Theory of natural selection

C. Theory of segregation

D. Theory of extinction

Explanation: Darwin explained evolution through the natural selection.

146. Which one of the following is a genetic disorder in which abnormally thick mucus is produced in the lungs and other parts of the body?

A.Lung cancerB.Chronic bronchitis

C.Cystic fibrosis D. Emphysema

Explanation:  Only cystic fibrosis among the following is genetic disorder.

147. Oxygen released into the atmosphere comes from:

A.CO2 B. H2O

C.C6H12O6 D. CO2 and H20

Explanation:  Plants undergo photosynthesis and produce oxygen as a byproduct. Carbon dioxide and water are the reactants that produce glucose and oxygen. 

148. End product of glycolysis in yeast is:

A. Ethanol and Carbon dioxide

B.Lactate

C.Pyruvate

D.Acetyl Co. A

Explanation:  Fact. 

149. First infectious disease against which effective method of prevention developed was a ____.

A.Bacterial disease B.Viral disease

C.Protozoan disease D.Viroid disease

Explanation:  Smallpox was the first disease aginst which vaccine was developed. It is a viral disease. 

150. _______infection is caused by a viroid:

A.Hepatitis AB.Hepatitis D

C.Mad Cow disease

D.Mysterious brain infection

151. Numerous opportunistic diseases might attack a person suffering from which of the following disease?

A.MeaslesB.Influenza

C.Hepatitis AD.AIDS

Explanation:  AIDS affects immune system. The patient can easily be infected from other diseases as his immune system is weakened. 

152. A combination of alpha interferon and ribavirin is used for the treatment of hepatitis.

A.BB.A

C.DD.C

Explanation:  These two medicines are used in combination to treat hepatitis C. They belong to antiviral class.

153. Cysts are not resistant to ________ but spores are:

A.LightB.Desication

C.pHD.Heat

Explanation:  Spores can withstand high temperature in contrast to cysts. 

154. In which phase of bacterial growth, they divide at exponential rate?

A.Lag phaseB.Log phase

C.Stationary phaseD.Decline phase

Explanation:  During lag phase, there is no increase in bacterial cells. During log phase, there is rapid increase, bacteria divide at exponential rate. During stationary phase, division rate and death rate is equal. And during decline phase, death rate is much higher. 

155. Which of the following statements is not true?

A. T cells produce and secrete antibodies

B. T cells can only recognize viral antigens outside the infected cells

C. B cells activate the immune system to destroy the pathogens. 

D. The main difference between T cells and B cells is that T cells recognize viral antigens outside the infected cells whereas B cells can recognize the surface antigens of bacteria and viruses 

Explanation: B lymphocytes produce antibodies which are secreted into the lymph and blood where they circulate freely.

156. Aspergilosis is a fungal infection and occurs only in _________:

A.MaleB.Female

C.AIDS patient D. Athletes

Explanation:  Aspergilosis occurs only in people whose immune system is weakened due to any reason. AIDS patients have weak immune system, so they are more likely ro get aspergillosis. In this disease, lungs and sinuses get affected. 

157. Gastrin is produced in high quantity when there is more __ in food

A. Proteins B. Fats

C. Carbohydrates D. Water 

Explanation: more protein in food results in stimulation of gastric glands to produce more gastrin. The stomach lining sends stimulus to gastric gland. Thus more protein, more 

Gastrin. Stomach is the main site of protein digestion. 

158. Many ____ expel large amount of water by special structures called contractile vacuoles:

A.Protozoa B. Porifera

C.EchinodemD.Fish

Explanation:  Protozoa are unicellular organisms. They have contractile vacuole that stores water and expels it in the surroundings. It also removes nitrogenous wastes.

159. Protection and nourishment to sperms is provided by

A. Fluid secreted by sertoli cells

B. Fluid in scrotum

C. Interstitial fluid 

D. All of the above

Explanation: fact. Fluid secreted by sertoli cells provide protection, nourishment, and liquid medium to the sperms while they are in tubules.

160. Asexual spores of fungi are called:

A.Conidiospores B. Zygospores

C.AscosporesD.Basidiospores

Explanation:  Asexual spores of fungi are produced on hyphae called conidiophores. The spores are called conidiospores.

161. What is the risk of hemophilic baby boy in a family when mother is color blind but father is normal?

A.25 % B.50 %

C. 75 % D. 100 % 

Explanation: The gene for hemophilia is located on the x chromosome and the offspring receives it from the mother. Father gives y chromosome to his son and x to daughter. Now, the son gets a defective gene from mother but there is no allele of hemophilia on y chromosome, so the disease will show and in this way all the sons will be hemophilic.

162. The term which is not related to the process of evolution of leaf:

A.OvertoppingB.Planation

C.Heterospory D. Fusion / webbing

Explanation:  Overtopping, planation, and webbing are three steps of evolution of leaves. 

163. The most successful land adapting plants are:

A.MossesB.Ferns

C.Gymnosperms D.Angiosperms

Explanation:  The features including presence of water proof cuticle, alternation of generations, presence of stomata, presence of vascular tissues, etc. made angiosperms better adapted to land. 

164. Excretory system consisting of protonephridial tubes are present in phylum:

A.Porifera B.Annelida 

C.Platyhelminthes D. Cnidaria

Explanation:  Protonephridia are present in phylum platyhelminthes. They are also called flame cells because they look like flame due to cluster of cilia. They are involved in excretion and act as kidney. 

PUNJAB MDCAT SOLVED PAST PAPER-2022

PUNJAB MDCAT SOLVED PAST PAPER-2022

For F.S.C. and Non-F.Sc. Students 

Time Allowed: 210 Minutes (3- ½ hours)

Instructions: 

i. Read the instructions on the MCQs Response Form carefully. 

ii. Choose the Single Best Answer for each question.

iii. Each correct Answer carries One Mark. There is No Negative Marking  

iv. Candidates are strictly prohibited from giving any identification mark except Roll. No. & Signature in the specified columns only. 

Biology 

  • 1. The enzymes integrase, protease and reverse transcriptase are found in which virus?
  • When the virus invades a CD4 cell, the cell becomes reprogrammed to create new copies of HIV. To do this, HIV needs the help of certain enzymes: reverse transcriptase, protease, and integrase.
  • Hepatitis A virus
  • Herpes virus
  • Influenza virus
  • Human immunodeficiency virus 
  • 2. What does the term bacteriophage refer to?
  • A bacteriophage is a type of virus that infects bacteria. The term phage means to devour. (bacteria eater)
  • A virus that infects bacteria 
  • A bacterium that infects virus
  • A virus which behaves as bacteria
  • Combination of Bacterium & Virion
  • 3. What of the following virus contains single stranded DNA?
    Single-stranded DNA (ssDNA) viruses are widespread in the environment and include economically, medically, and ecologically important pathogens. Parvovirus is a single stranded DNA virus which causes disease in rats.
  • Adeno virus
  • Herpes virus
  • Parvo virus 
  • Pox virus
  • 4. How may tail fibrils are attached to the end plate of a bacteriophage?
    The phage tail is a complex, multiprotein structure that mediates attachment, digestion and penetration of the cell wall and genome ejection. There are six short tail fibers (STFs) folded beneath the baseplate that unfold upon host recognition.
  • 2
  • 4
  • 6 
  • 8
  • What is the end product of glucose by yeast in anaerobic respiration?
    In yeast, the end products of anaerobic respiration are ethyl alcohol, carbon dioxide and ATP (Adenosine triphosphate).
  • A. Ethanol and oxygen
  • B. Ethanol and water
  • C. Ethanol and CO 
  • D. Lactic acid and CO
  • Each carrier in Electron Transport Chain is first _________ and then _________.
  • The 3 main steps of the electron transport chain?
  • Step 1: Generating a Proton Motive Force.
  • Step 2: ATP Synthesis via Chemiosmosis.
  • Step 3: Reduction of Oxygen.
  • A. Broken-down, Regenerate
  • B. Generated, Broken-down
  • C. Oxidized, Reduced
  • D. Reduced, Oxidized 
  • Electron transport chain explains:
  • The electron transport chain is a series of four protein complexes that couple redox reactions, creating an electrochemical gradient that leads to the creation of ATP in a complete system named oxidative phosphorylation
  • A. Photophosphorylation
  • B. Z-Scheme
  • C. Photolysis
  • D. Mechanism of ATP synthesis 
  • What is the colour of Chlorophyll-b molecule?
  • Chlorophyll a is blue-green, chlorophyll b is yellow-green, carotene appears bright yellow, and xanthophyll is pale yellow green.
  • A. Blueish green
  • B. Yellowish green 
  • C. Dark Green
  • D. Reddish green
  • Upon initial hydrolysis starch yields:
  • On partial hydrolysis of starch and glycogen, We can obtain the disaccharide maltose and a low molecular weight dextran.
  • A. Maltose 
  • B. Sucrose
  • C. Glucose
  • D. Mannose
  • Human Bone cells contain ……… % of water?
  • The organs with the lowest percentage of water in the human body are the teeth (8%) and bones (20-25%)
  • A. 20 
  • B. 85
  • C. 40
  • D. 90
  • Unique three-dimensional shape of the fully folded polypeptide, constitutes:
  • The overall three-dimensional structure of a polypeptide is called its tertiary structure. The tertiary structure is primarily due to interactions between the R groups of the amino acids that make up the protein.
  • A. Primary structure of protein
  • B. Secondary structure of protein
  • C. Tertiary structure of protein 
  • D. Quaternary structure of protein
  • Butyric acid is a ________ carbon fatty acid.
  • Butyric Acid is a saturated short-chain fatty acid with a 4-carbon backbone.
    A. 6
  • B.2
  • C. 4 
  • D.8
  • Which of the following is a conjugated molecule?
  • A compound formed by chemically joining two or more different substances. Examples of conjugated proteins are phosphoproteins, glycoproteins, nucleoproteins, chromoproteins, lipoproteins, flavoproteins, metalloproteins
  • A. Protein
  • B. Lipid
  • C. Glycoproteins 
  • D. Vitamins
  • Hydrolysis process is a reverse of ————– process.
  • Condensation is a chemical reaction that involves the synthesis of a complex molecule from small reactant molecules. Hydrolysis: Hydrolysis is the breakdown of a large reactant molecule into small fragments in the presence of water.
  • A. Photolysis
  • B. Condensation 
  • C. Deduction
  • D. Convection
  • Proteins are the main ———– of the cell?
  • most of the structure in a cell are made of proteins. Proteins are the main structural components of cell
  • A. Physiological components
  • B. Functional components
  • C. Structural components 
  • D. Biological components
  • Cell wall may be absent in which of the following?
  • Mycoplasma is a genus of bacteria that lack a cell wall and surrounded by their cell membrane only.
  • A. Plant & Algae
  • B. Algae & Fungi
  • C. Fungi & Archaea
  • D. Bacteria & Archaea 
  • Structure formed by invagination of plasma membrane and involved in cell division and DNA replication of prokaryotic cell:
  • Invagination is a mechanism that takes place during gastrulation. This mechanism or cell movement happens mostly in the vegetal pole. In prokaryotic cells, cell membrane invaginate to form mesosomes. They have enzymes, which are useful for respiration.
  • A. Lysosomes
  • B. Mesosomes 
  • C. Golgi bodies
  • D. Phragmoplasts
  • Which of the following are single membranous organelles?
  • There are four types of single-membrane-bound organelles: Golgi bodies, the ER , vacuole /lysosomes , and peroxisomes
  • A. Mitochondria and ribosomes
  • B. Cytosol, mitochondria and ribosomes
  • C. Golgi bodies, Lysosome and ER 
  • D. Golgi bodies, lysosome and mitochondria
  • Movement of molecules against the concentration gradient is?
  • Active transport is the transport of molecules that require the use of energy, in the form of an energy-storing chemical called ATP (adenosine triphosphate) to help them cross the cell membrane.
  • A. Passive transport
  • B. Active transport 
  • C. Facilitated diffusion
  • D. Filtration
  • The digestive vacuoles and autophagosomes are also known as?
  • The digestive vacuoles and autophagosomes are also known as Secondary Lysosomes. They are also involved in the autophagy (self-eating). During this process some old, worn out parts of cell, such as old mitochondria are digested
  • A. Phagocytosis
  • B. Primary lysosome and autophagy
  • C. Secondary lysosome 
  • D. Peroxisome
  • The cell wall of Bacteria is made up of:
  • The cell wall consists mainly of peptidoglycan (PG), a mesh of polysaccharide strands. Peptidoglycan is also known as murein.
  • A. Chitin
  • B. Murein 
  • C. Cellulose
  • D. Hemicellulose
  • Which one is common in both prokaryotic and eukaryotic cells?
  • Ribosomes are the cellular organelles that carry out protein synthesis. They are found in both prokaryotes and eukaryotes.
  • A. Cytoplasmic streaming movement
  • B. Ribosome 
  • C. Binary fission
  • D. Nuclear envelope
  • There is no clear difference between dendrites and axons in sensory neurons, except:
  • A. Thickness
  • B. Length
  • C. Terminal portions 
  • D. None of the above
  • The neurotransmitter active outside the CNS (Central Nervous System) is:
  • Acetylcholine. This excitatory neurotransmitter does a number of functions in your central nervous system (CNS [brain and spinal cord]) and in your peripheral nervous system (nerves that branch from the CNS)
  • A. Acetylcholine 
  • B. Dopamine
  • C. Glutamate
  • D. Serotonin
  • A hormone that plays a major role in social bonding, childbirth, milk ejection and sexual reproduction is:
  • The two main physical functions of oxytocin are to stimulate uterine contractions in labor and childbirth and to stimulate contractions of breast tissue to aid in lactation after childbirth.
  • A. Estrogen
  • B. Oxytocin 
  • C. Prolactin
  • D. Secretin
  • Hormone produced by placenta is:
  • Progesterone helps to prepare the body for pregnancy by stimulating glandular development and the development of new blood vessels. This provides a good environment for implantation by a fertilized egg.
  • A. Follicle-Stimulating Hormone (FSH)
  • B. Luteinizing Hormone (LH)
  • C. Progesterone 
  • D. Testosterone
  • The middle layer of meninges is:
  • Three layers of membranes known as meninges protect the brain and spinal cord. The delicate inner layer is the pia mater. The middle layer is the arachnoid, a web-like structure filled with fluid that cushions the brain. The tough outer layer is called the dura mater.
  • A. Arachnoid mater 
  • B. Pia mater
  • C. Dura mater
  • D. Cranium
  • The part of brain which guides smooth and accurate motions and maintains body position is:
  • The cerebellum is located in the back of your brain. It helps with the coordination and movement related to motor skills, especially involving the hands and feet. It also helps maintain posture, balance, and equilibrium.
  • A. Cerebrum
  • B. Cerebellum 
  • C. Pons
  • D. Medulla
  • Water vascular system or ambulacral system is a unique and complex system specially present in?
  • Echinoderms possess a unique ambulacral or water vascular system, consisting of a central ring canal and radial canals that extend along each arm. Water circulates through these structures and facilitates gaseous exchange as well as nutrition, predation, and locomotion.
  • A. Sponges
  • B. Arthropods
  • C. Echinoderms 
  • D. Fishes
  • Round worms belong to which phylum?
  • Roundworms: Phylum Nematoda. Species in the phylum Nematoda (from the Greek root word nema meaning thread) are better known as the roundworms
  • A. Annelida
  • B. Coelenterata
  • C. Nematoda 
  • D. Platyhelminthes
  • Silver fish is a/an?
  • Silverfish are small insects that can infest a home. Although they are not dangerous, they can damage property and trigger allergies.
  • A. Insect 
  • B. Mollusc
  • C. Jawless fish
  • D. Cartilaginous fish
  • Tissue are not found in the following animal?
  • sponges lack real organs or true tissues, but they do contain different types of cells including: epithelial-like cells called pinacocytes that cover the exterior and non-flagellated interior surfaces.
    Flat worms
  • A. Sponges 
  • B. Cnidarians
  • C. Round worms
  • D.
  • Enzymes lower the activation energy by stabilizing the transition state of a metabolic reaction due to?
  • By binding substrates to their active sites, enzymes stabilize the structure of the transition state. This in turn lowers of the free energy of the transition state.
  • A. Changing conditions within the active site 
  • B. Changing conditions within the protein framework
  • C. Rearranging the fatty acids in active site
  • D. Distorting the molecules in the allosteric site
  • Competitive inhibitors compete with?
  • A competitive inhibitor competes with substrate for binding to an active site. When the inhibitor occupies the active site, it forms an enzyme-inhibitor complex and the enzyme cannot react until the inhibitor dissociates.
  • A. Enzyme
  • B. Substrate 
  • C. Product
  • D. Coenzyme
  • Non-competitive inhibitor molecules have:
  • In noncompetitive inhibition, the inhibitor binds at an allosteric site separate from the active site of substrate binding. Thus in noncompetitive inhibition, the inhibitor can bind its target enzyme regardless of the presence of a bound substrate.
  • A. A similar structure to the normal substrate molecule
  • B. A quite different structure from the substrate molecule 
  • C. A different conformation but fit into the active site
  • D. A similar conformation but does not fit into the active site
  • Zinc ion is attached at the active site of the enzyme carboxypeptidase. The zinc ion functions as:
  • carboxypeptidase is a metallo-enzyme which contains zinc. It has been reported that the enzyme can be de-activated by removal of zinc. zinc atom acts as an allosteric activator and helps the enzyme to bind its substrate.
  • A. A coenzyme molecule
  • B. An activator 
  • C. An inhibitor molecule
  • D. Controller of Allosteric site
  • What is the best physiological pH for optimum functioning for most of the cellular enzymes of human?
  • An enzyme has an optimum pH. Although most enzymes remain high activity in the pH range between 6 and 8, some specific enzymes work well only in extremely acidic (pH <5.0) or alkaline (pH >9.0) conditions.
  • A. 2-3 pH
  • B. 3-5 pH
  • C. 6-8 pH 
  • D. 8-10 pH
  • Adaptations that an organism acquires by its own actions during its life span without modifying its genome are:
  • An adaptation is any heritable trait that helps an organism, such as a plant or animal, survive and reproduce in its environment. But as mentioned the genome is not altered or affected so it will not pass on from the individual to the offspring as it’s just a habitual change in an organism.
  • A. Heritable
  • B. Non-heritable 
  • C. Can be made heritable through some modification
  • D. Sometimes heritable and other times non-heritable
  • For evolutionary process to occur, which of the following is NOT a geographical barrier?
  • Geographic barriers include the formation of a mountain range, the movement of a glacier, and the division of a large lake into several smaller lakes.
  • A. Ocean
  • B. River
  • C. Mountains
  • D. Atmosphere 
  • According to the Biogenetic Law of Ernst Haeckel:
  • Biogenetic law, also called Recapitulation Theory, postulation, by Ernst Haeckel in 1866, states that ontogeny recapitulates phylogeny (the development of the animal embryo and young traces the evolutionary development of the species)
  • A. There is survival of the fittest
  • B. There is use and disuse of organs
  • C. Phylogeny recapitulates ontogeny
  • D. Ontogeny recapitulates phylogeny 
  • The animal species on Galapagos resemble species living on the:
  • On the Galapagos Islands, Charles Darwin observed that tortoises resembled those found in South America. 
  • Three extant species of Geochelone exist on mainland South America and are therefore the best candidates for the closest living relative of the Galápagos tortoises: Geochelone denticulata, the South American yellow-footed tortoise; Geochelone carbonaria, the South American red-footed tortoise; and Geochelone chilensis, the Chaco tortoise.
  • Northern Europe
  • Great Britain
  • North American mainland
  • South American mainland 
  • Digested food from intestine is carried to the liver by?
  • The hepatic portal vein is the part of the hepatic portal system. It carries blood from the intestines, gallbladder, pancreas, and spleen and delivers it to the liver. It contains blood with nutrients and toxins after digestion.
  • Hepatic artery
  • Hepatic vein
  • Hepatic portal vein 
  • Hepatic portal artery
  • ——– proteins are produced by WBCs in response to ——– and provide immunity?
    Antibodies are produced by B cells (specialized white blood cells). When an antigen encounters a B cell, it causes the B cell to divide and clone. These cloned B cells — or plasma cells — release millions of antibodies into your bloodstream and lymph system.
  • Antibiotics, antlgen
  • Antibodles, RBC
  • Globulin, histamine
  • Antibodies, antigen 
  • The lymphatic vessels of the body empty the lymph into blood stream at?
  • Lymphatic vessels empty the lymph into the right lymphatic duct and left lymphatic duct (also called the thoracic duct). These ducts connect to the subclavian vein, which returns lymph to your bloodstream. The subclavian vein runs below your collarbone.
  • Abdominal vein
  • Jugular vein
  • Subclavian vein 
  • Bile duct
  • Flow of blood in the capillaries is adjusted by?
  • Blood flow through the capillary beds is controlled by precapillary sphincters to increase and decrease flow depending on the body’s needs and is directed by nerve and hormone signals. Lymph vessels take fluid that has leaked out of the blood to the lymph nodes where it is cleaned before returning to the heart.
  • Heart directly
  • Pre-capillary sphincters 
  • Meta-arteriole
  • Valves
  • The pressure exerted by a solution separated by a semipermeable membrane from pure water is ________?
  • Osmotic pressure is the minimum pressure which needs to be applied to a solution to prevent the inward flow of its pure solvent across a semipermeable membrane. It is also defined as the measure of the tendency of a solution to take in its pure solvent by osmosis.
  • Osmotic Pressure 
  • Soil potential
  • Solute Potential
  • Solute Potential
  • Which of the following is NOT a consequence of anaerobic respiration in humans muscles cells?
  • Anaerobic respiration happens in muscles during hard exercise. Glucose is not completely broken down, so less energy is released than during aerobic respiration. There is a build-up of lactic acid in the muscles during vigorous exercise. This build up causes cramps, and pain in muscles.
  • Cramps
  • High consumption of energy 
  • Pain
  • Tiredness
  • The respiratory surfaces exhibit following characteristic?
  • The surface must be permeable to allow gases to pass through. Furthermore, having an extensive blood supply is vital, in the case of alveoli the capillary network ensures that oxygen rich blood can be transported away from the lungs.
  • It must be permeable 
  • It must be thick for low diffusion
  • It should be non-vascularized
  • It should have low ventilation mechanism
  • Which of the following is a prokaryote?
  • Prokaryotes are organisms whose cells lack a nucleus and other organelles. Prokaryotes are divided into two distinct groups: the bacteria and the archaea. Bacteria are examples of the prokaryotic cell type. An example is E. coli.
  • Protista
  • E.coll 
  • Amoeba
  • Fungi
  • Number of layers present in Gram-negative bacterial cell wall:
  • Gram-negative bacteria are surrounded by a thin peptidoglycan cell wall, which itself is surrounded by an outer membrane containing lipopolysaccharide.
  • One
  • two 
  • three
  • four
  • The division of cocci in three planes form Sarcina, which is a cube of ———- Cocci?
  • Sarcinae is a type of cocci that are arranged in a cuboidal manner, as the cells are formed by regular cell divisions in three planes. Cocci that divide in three planes and remain in groups cube like groups of eight.
  • 02
  • 04
  • 08 
  • 10
  • Which of the following statement is correct?
  • Mycobacterium tuberculosis is a weakly gram-positive, non-motile, rod-shaped bacterium. Streptococcus pneumoniae are lancet-shaped, gram-positive, facultative anaerobic bacteria.
  • Tuberculosis and Pneumonia are caused by Gram A. Positive Bacteria 
  • B. Tuberculosis and Pneumonia are caused by Gram Negative Bacteria
  • C. Pneumonia is a lung disease caused by Gram Negative Bacteria
  • D. Tuberculosis is a lung disease caused by Gram Negative Bacteria
  • Nitrifying bacteria are the examples of:
  • Chemosynthetic bacteria include a group of autotrophic bacteria that use chemical energy to produce their own food. Nitrobacter is chemosynthetic bacteria.
  • Heterotrophic bacteria
  • Chemosynthetic bacteria 
  • Saprophytic bacteria
  • Parasitic bacteria
  • Each human testis is divided into:
  • The tunica albuginea has extensions into each testis that act as partial partitions to divide the testis into approximately 250 -290 compartments, or lobules. Each lobule contains one or more convoluted tubules, or narrow tubes, where sperm are formed.
  • 50-100 lobules
  • 150-200 lobules
  • 200-300 lobules
  • 250-300 lobules 
  • Which cells in the human males are responsible for the release of testosterone?
  • Leydig cells produce testosterone in the testes under the pulsatile control of pituitary luteinizing hormone
  • Pituitary Gland
  • Hypothalamus
  • Sertoll cells
  • Leydig cells or interstitial cells 
  • Fertilized ovum is implanted and undergoes further development in the:
  • Once fertilization takes, the zygote will generally travel to the uterine cavity and implant in the wall of the uterus near the fundus.
  • Ovary
  • Uterus 
  • Oviduct
  • Cervix
  • Level of luteinizing hormone (LH) is maximum in blood during which stage of menstrual cycle?
  • The ovulatory phase begins with a surge in luteinizing hormone and follicle-stimulating hormone levels. Luteinizing hormone stimulates egg release (ovulation).
  • Menstrual stage
  • Proliferative stage
  • Ovulation stage 
  • Secretory stage
  • Major source of transmission of syphilis is:
  • Syphilis is a sexually transmitted infection (STI).
  • Blood transfusion
  • Insect bite
  • Contaminated water
  • Sexual contact 
  • What is FALSE about cartilage?
  • Cartilage is a strong, flexible connective tissue that protects your joints and bones. It acts as a shock absorber throughout your body. Cartilage, once synthesized, lacks lymphatic or blood supply and the movement of waste and nutrition is chiefly via diffusion to and from adjacent tissues.
  • There are many blood vessels in cartilage 
  • It is a form of connective tissue
  • It covers ends of the bones at joints
  • It is much softer than bone
  • Which of the following is a muscle component that act as store for energy?
  • Creatine phosphate helps make a substance called adenosine triphosphate (ATP). ATP provides the energy for muscle contractions.
  • ATP
  • Creatine-PO
  • Myoglobin
  • Creatinine-PO
  • Which of the following is NOT found in skeletal muscle fibers in human?
  • Hemoglobin is defined as an oxygen-carrying pigment and predominant protein present in red blood cells. It is an iron containing protein in the blood, which transports oxygen to tissues. Myoglobin is defined as a red protein containing haem, which carries and stores oxygen in muscle cells. It is an oxygen-binding protein, located primarily in muscle cells. Myoglobin has more affinity to oxygen from hemoglobin. So, it is able to remove oxygen from hemoglobin.
  • Multiple nuclei
  • Multiple mitochondria
  • Large amount of myoglobin
  • Large amount of hemoglobin 
  • Hinge joint is present between which of the following bones?
  • Common hinge joints are knee and elbow joints. So hinge joint is present between the humerus and ulna.
  • A. Humerus and radio-ulna 
  • B. Femur and pectoral girdle
  • C. Femur and acetabulum
  • D. Humerus and pectoral girdle
  • Test cross is made to check the genotype of a trait. Which of the following crosses is a test cross?
  • A test cross is a way to explore the genotype of an organism. Early use of the test cross was as an experimental mating test used to determine what alleles are present in the genotype. In its simplest form, a test cross is an experimental cross of an individual organism of dominant phenotype but unknown genotype and an organism with a homozygous recessive genotype (and phenotype).
  • A. Unknown  At
  • B. Unknown  tt 
  • C. Unknown  AB
  • D. Unknown  TT
  • What happens when a Rh –ve woman, married to a Rh +ve man conceives a child who is Rh +ve?
  • Rh incompatibility occurs when the mother’s blood type is Rh negative and her fetus’ blood type is Rh positive. Antibodies from an Rh negative mother may enter the blood stream of her unborn Rh positive infant, damaging the red blood cells (RBCs).
  • A. Maternal-foetal incompatibility 
  • B. Paternal-foetal incompatibility
  • C. Cancer of fetus
  • D. Death of mother
  • DNA stores biological information in discrete units termed as:
  • A gene is the basic physical and functional unit of heredity. Genes are made up of DNA. Some genes act as instructions to make molecules called proteins.
  • A. Genes 
  • B. Phenotypes
  • C. Karyotypes
  • D. Cells
  • To study sex linkages in Drosophila, Morgan mated white eyed males with wild type red eyed females. What will be the phenotype of offspring?
  • The eye color gene is linked with to sex and is present on the X-chromosome. X-chromosome doesn’t pass directly from father to the offspring’of the same sex but follows a crisscross inheritance, which is transferred from one sex to the offspring of the opposite sex. Since the trait of white eye color is recessive and linked to sex, it shows its effect in the male because of single X-chromosome. The female flies can show white-eye trait when both its X-chromosome carry the recessive allele of the white eye. At last, Morgan concluded that only X chromosome carries the gene for eye color in Drosophila. Y-chromosome did not have the gene for eye color. The genes like the eye color present on the sex chromosome are said to be sex-linked or X-linked.
  • A. All red eyed males and females 
  • B. Red eyed females and white eyed males
  • C. White eyed females and red eyed males
  • D. All white eyed females and males
  • Which one of the following is X Linked Dominant disorder?
  • Hereditary hypophosphatemia rickets can have several patterns of inheritance. When the condition results from mutations in the PHEX gene, it is inherited in an X-linked dominant pattern. The PHEX gene is located on the X chromosome, which is one of the two sex chromosomes
  • A. Hemophilia
  • B. Color blindness
  • C. Hypophosphatemia rickets 
  • D. Hypertrichosis
  • Mode of inheritance in humans can be traced through:
  • Pedigree analysis was developed to understand the inheritance of genes from parents to offspring. It was developed as a chart that can represent a family tree along with the family members and their genetic traits, respectively.
  • A. Experimental Mating
  • B. Chi Square Chart
  • C. Pedigree Analysis 
  • D. Probability Analysis

Logical Reasoning 

  • Read the passage and the following statements below. Then choose the correct option, basing your answer only on the information provided. 
  • Queen Elizabeth II’s Platinum Jubilee, celebrating her 70 years on the British throne, was above all a tribute to one of history’s great acts of constancy. Her reign spanned virtually the entire post-Word War II era, making her a witness to cultural upheavals from the Beatles to Brexit. 
  • STATEMENTS: 
  • I. There has been another queen of the British throne named Elizabeth Before her. 
  • II. Brexit is a normal occurrence. 
  • III. Elizabeth was Queen of the British during World War II.
  • The name queen Elizabeth II indicates that there was another queen with the same name or known as queen Elizabeth I.
  • A. I, II and III; all are correct
  • B. Only III is correct
  • C. Only I is correct 
  • D. Only I and III are correct
  • Observe the pattern and select the next term in the sequence: CA
  • B, FAE, IAH
  • CAB (D,E is skipped for first letter and C,D is skipped for last letter) FAE ( G,H is skipped for first letter and F,G is skipped for last letter) IAH (J,K is skipped for first letter and I,J is skipped for last letter) LAK.
  • A. JHK
  • B. LAK 
  • C. JGK
  • D. IGJ
  • Read following and choose the correct answer: 
  • Drake was wearing a blue shirt with black jeans and brown shoes, John was wearing a red shirt with black jeans and black shoes. Ahmad was wearing a blue shirt with blue jeans and brown shoes, Nahaz claims he saw someone wearing black jeans, a blue or red shirt and shoes that were not black. Who did he see?
  • Drake and Ahmad were not wearing black shoes but Nahaz saw someone black jeans and Drake wore black jeans.
  • A. Ahmad
  • B. John
  • C. Drake 
  • D. Cannot elicit from given information
  • Some bags are pouches. All pouches are cases. No cases are purses. Which of the following conclusions are NECESSARILY TRUE? 
  • I. Some pouches are purses. 
  • II. Some bags are cases. 
  • III. No bag are purses.
  • This option is correct because some bags are pouches and all pouches are cases so some bags are cases as well.
  • A. I and II
  • B. I and III
  • C. II 
  • D. II and III
  • Read the following statement, assuming everything in it to be true. Then decide which of the given suggested courses of action logically follow and are worth pursuing, 
  • Statements: 
  • “Aalia wants to sleep but cannot due to regular noise in and around her house every day.”  
  • I. Insert good quality noise blockers into her ears. 
  • II. Take strong sleeping pills.”
  • noise blockers are harmless and will help Aalia sleep better.
  • A. I 
  • B. II
  • C. Both I and II
  • D. Neither I nor II
  • The literacy rate in the district has been increasing. II. The district administration has conducted extensive training program for the workers involved in the literacy drive.
  • District administration conducted training programs for workers so as a result literacy rate increased in the district
  • A. Statement I is the cause and statement II is its effect
  • B. Statement II is the cause and statement I is its effect 
  • C. Both the statements I and II are independent causes
  • D. Both the statements I and II are effects of independent cause

PHYSICS 

  • What is the shape of velocity-time graph for constant acceleration?
  • A. Parabola line
  • B. Straight line 
  • C. Incline curve
  • D. Decline curve
  • Which of the following is the correct definition of variable velocity?
  • A. Unequal distances are covered in equal intervals of time
  • B. Equal displacements are made in unequal intervals of time
  • C. Unequal displacements are made in equal intervals of time 
  • D. Equal displacements are made in equal intervals of time
  • A stone thrown horizontally from the top of a tall building follows a path that is:
  • A. Circular
  • B. Made of two straight line segments
  • C. Hyperbolic
  • D. Parabolic 
  • Which of the following is incorrect?
  • A. Reaction force on a body is always balanced by the action force 
  • B. Reaction and action forces are always equal
  • C. Action and reaction forces never act on the same body
  • D. Newton’s Third Law is always valid in all situations
  • A fireman wants to slide down a rope. The breaking load of the rope is 3/4th of the weight of the man. With what acceleration should the fire man slide down? (Acceleration due to gravity is ‘g’)
  • A. g
  • B. g/4 
  • C. 3g/4
  • D. 0
  • When a heavy coin falls a short distance towards the ground it does not reach terminal velocity. Why is this so?
  • A. The coin has not hit the ground
  • B. The weight of coin is equal to air resistance
  • C. The weight of coin increases as air resistance increases
  • D. The weight of coin is more than air resistance 
  • The consumption of energy by a 60 W bulb in 2 s is:
  • A. 120J
  • B. 60J
  • C. 30J
  • D. 0.02J
  • A long spring, when stretched by a distance x, has potential energy V. On increasing the stretching to nx, the potential energy of the spring will be:
  • A. nV
  • B. V/n
  • C. n^2 V 
  • D. V/n^2
  • Ignoring details associated with friction, extra forces exerted by arm and leg muscles, and other factors, we can consider a pole vault as the conversion of an athlete’s running kinetic energy to gravitational potential energy. If an athlete is to lift his body 5 m during a vault, what speed must he have when he plants his pole?
  • A. 5 m/s
  • B. 10 m/s 
  • C. 15 m/s
  • D. 20 m/s
  • A particle of mass m at rest is acted upon by a force P for time t. Its kinetic energy at time t is:
  • A. (P^2 t^2)/m
  • B. (P^2 t^2)/2m 
  • C. (P^2 t^2)/3m
  • D. (P^2 t^2)/4m
  • The number of revolutions in 3n radians is:
  • A. 1/60
  • B. 3/2 
  • C. 2
  • D. 6
  • If a flywheel is rotating at 3.0 rad/s, the time it takes to complete one revolution is:
  • A. 0.67 s
  • B. 1.0 s
  • C. 1.3 s
  • D. 2.1 s 
  • A fighter plane is moving in a vertical circle of radius r. Its minimum velocity at the highest point of the circle will be?
  • A. 3gr
  • B. 2gr
  • C. gr 
  • D. (gr/2)
  • Which of the following increase by increasing amplitude?
  • A. Wavelength
  • B. Frequently
  • C. Zero
  • D. Loudness 
  • An airplane travels at a speed of 0.5v where v is the speed of sound. The airplane approaches a stationary observer. The frequency of sound emitted by the aircraft is Hz. Which frequency does the observer hear?
  • A. 400 Hz 
  • B. 100 Hz
  • C. 120 Hz
  • D. 180 Hz
  • If the wavelength of light coming from a galaxy shifts towards the red end of spec then galaxy is:
  • A. Approaching Earth
  • B. Receding the Earth
  • C. Stationary
  • D. Approaching Earth or is star
  • The shortest distance between any two points in phase on a wave is called:
  • A. Displacement
  • B. Amplitude
  • C. Wavelength 
  • D. Frequency
  • When will the oscillations stop in the absence of resistive forces?
  • A. Never 
  • B. After 10 minutes
  • C. In 10 minutes
  • D. Immediately
  • The mechanical waves are not generated by:
  • A. Electric and magnetic fields 
  • B. Coil of springs
  • C. Ropes
  • D. Water
  • Reducing mass M of a suspending body to one fourth will change the frequency of oscillation to:
  • A. One fourth
  • B. Double 
  • C. Quadruple
  • D. Half
  • A distant star is receding from the Earth with a speed of 1.40  10^7 m/s. It emits light of frequency 4.57  10^14 Hz. The speed of light is 3.0  10^8 m/s. The Doppler effect formula can be used with light waves. What will be the frequency of this light when detected on Earth?
  • A. 2.04  10^13 Hz
  • B. 4.37  10^14 Hz
  • C. 4.57  10^14 Hz
  • D. 4.79  10^14 Hz
  • Thermodynamics is that branch of Physics in which we study:
  • A. relations between heat and mechanical energies
  • B. relations between heat and ionization energies
  • C. relations between chemical and mechanical energies
  • D. relations between kinetic and potential energies
  • When a gas is compressed isothermally, the product of its pressure and vojume dfuring the process is:
  • A. not constant
  • B. constant 
  • C. zero
  • D. proportional to entropy
  • Temperature of given mass of a gas is changed from 150C to 300C during an isobaric process, volume of the gas will become:
  • A. Half
  • B. Double
  • C. Remain same
  • D. Less than double 
  • A capacitor is charged with a battery and energy stored is U. After disconnecting battery another capacitor of same capacity is connected in parallel to the first capacitor. Then energy stored in each capacitor is:
  • A. U/2
  • B. U/4 
  • C. 4U
  • D. 2U
  • What is the potential difference between two points in an electric field if it takes 600 J of energy to move a charge of 2 C between these two points?
  • A. 0 J
  • B. 1200 J
  • C. 300 J 
  • D. 800 J
  • Gauss law cannot be used to find which of the following quantity?
  • A. Electric field intensity
  • B. Electric flux density
  • C. Charge
  • D. Permittivity 
  • Which one of the following statements is true?
  • A. electrostatic force obeys inverse square law while gravitational force does not
  • B. both gravitational force and electrostatic force are repulsive in nature
  • C. gravitational force is much weaker than electrostatic force 
  • D. Both electrostatic force and gravitational force don’t obey inverse square law
  • The coulomb’s constant k depends upon:
  • A. nature of medium
  • B. system of units
  • C. types of charge
  • D. nature of medium and system of units 
  • A charged particle is moving in a uniform electric field. For the motion of the particle due to the field, which quantity has a constant non-zero value?
  • A. acceleration 
  • B. displacement
  • C. rate of change of acceleration
  • D. velocity
  • A capacitor of capacitance ‘C’ has a charge ‘Q’ and stored energy is ‘w’. If the charge is increases to ‘2Q’. The stored energy will be:
  • A. 2W
  • B. 4W
  • C. W/4
  • D. W/2
  • How much potential drop exist across closed switch?
  • A. 0V 
  • B. IV
  • C. 2V
  • D. 3V
  • A 3V battery is connected in series with ammeter and 2 ohm resistance after short circuiting. What will be the reading of ammeter?
  • A. 1 A
  • B. 1.5A 
  • C. 5A
  • D. 6A
  • The resistance of a conductor does not depend on which of the following?
  • A. Area
  • B. Resistivity
  • C. Length
  • D. mass 
  • Which of the following statement is NOT CORRECT about Kirchhoff’s rule?
  • A. Kirchhoff’s current rule based upon the law of conservation of charge
  • B. Wheatstone bridge is an application of kirchhoff’s rule
  • C. Kirchhoff’s voltage rule are more suitable in AC circuits 
  • D. Kirchhoff’s voltage rule based upon the law of conservation of energy
  • What do the substances whose resistance decreases with increase in temperature have?
  • A. high temperature coefficient
  • B. negative temperature coefficient 
  • C. positive temperature coefficient
  • D. zero temperature coefficient
  • A low voltage supply with an e.m.f. of 20 V and an internal resistance 6.5 ohms in a fish tank. What is the power supplied to the water in the fish tank?
  • A. 41 W 
  • B. 53 W
  • C. 50 W
  • D. 62 W
  • Electric forces change the magnitude and direction of velocity while magnetic forces change ____________ of velocity.
  • A. Only Magnitude
  • B. Only direction 
  • C. Magnitude and direction
  • D. Neither magnitude nor direction
  • Which surface has greater magnetic flux in same magnetic field, each has an area 1 m^2.
  • A. Circular
  • B. Rectangular
  • C. Square
  • D. Flux is independent of shape 
  • The source of magnetic field is:
  • A. An isolated magnetic pole
  • B. Static electric charge
  • C. Nonmagnetic substance
  • D. Current loop 
  • One meter long copper rod is moving with speed 20 m/sec in the magnetic field of strength 0.6 tesla. What is the value of induced emf?
  • A. 10 v
  • B. 12 v 
  • C. 14 v
  • D. 16 v
  • The unit of /t can be written as?
  • A. NmA–²S¹
  • B. NmAS¹
  • C. NmA–¹S¹ 
  • D. NmA–²S²
  • Working principal of magnetic levitation train is according to?
  • A. Faraday law
  • B. Max planks law
  • C. Ohm law
  • D. Lenz law 
  • A copper hoop is held in a vertical east-west plane in a uniform magnetic field whose field lines run along the north-south direction. The largest induced emf is produced when the hoop is?
  • A. Rotated about a north-south axis
  • B. Rotated about an east-west axis 
  • C. Moved rapidly, without rotation, toward the east
  • D. Moved rapidly, without rotation, toward the south
  • In transformer, there is no _______ connection between the two coils but the are ________ linked?
  • A. Magnetic, electrically
  • B. Electrical, magnetically 
  • C. Magnetic, magnetically
  • D. Electrical, optically
  • When the temperature of semiconductor suddenly drops to zero kelvin, then a semiconductor acts as:
  • A. Conductor
  • B. Semi-conductor
  • C. Super conductor
  • D. Insulator 
  • If electron, proton, neutron, and alpha particle have same velocity, which of them ha: shortest wavelength?
  • A. Electron
  • B. Proton
  • C. Neutron
  • D. Alpha particle 
  • The process of ejection of loosely bound electrons from a certain photo sensitive sur by absorption of photon is called:
  • A. Compton effect
  • B. Photoelectric effect 
  • C. Pair production
  • D. Black body radiation
  • In a photoelectric effect experiment, the stopping potential is:
  • A. The kinetic energy of the most energetic electron ejected
  • B. The potential energy of the most energetic electron ejected
  • C. The photon energy
  • D. The electric potential that cause electron current to vanish 
  • The line spectrum of hydrogen atom contains the spectral lines in the region of:
  • A. Ultraviolet
  • B. Infrared
  • C. Visible
  • D. all of these 
  • The speed of electron in the first Bohr orbit is:
  • A. 2.19  10 ms¹ 
  • B. 2.19  10 ms¹
  • C. 2.19  10 ms¹
  • D. 2.19  10 ms¹
  • A low energy neutron has RBE factor of 10. How much energy is absorbed by a man of mass 80kg if the value of equivalent dose is 400 rem?
  • A. 16 J
  • B. 32 J 
  • C. 48 J
  • D. 64 J
  • It has been observed that Thorium (-90^234)This transformed into Protactinium (91^234) Pa after the emission of ……………. Particles:
  • A. Alpha
  • B. Gamma
  • C. Beta
  • D. Alpha, Beta, Gamma
  • The half-life of Strontium (Sr) is 8.70 hours. Its decay constant is:
  • A. 0.000022 s
  • B. 45000/s
  • C. 0.000022/s 
  • D. 0.000032/s

CHEMISTRY 

  • One a.m.u stands for
  • AMU is defined as one-twelfth the mass of an atom of carbon-12 (12C). 12C is the most abundant natural carbon isotope, accounting for over 98% of carbon found in nature. It has an AMU of 12.
  • A. An atom of C – 12
  • B. 1/12th of a carbon 
  • C. 1/12th of H
  • D. 1 atom of all the elements
  • A compound of sodium oxide has 74.2 % sodium and 25.8% of Oxygen. The empirical formula of the compound is?
  • An Empirical formula is the chemical formula of a compound that gives the proportions (ratios) of the elements present in the compound but not the actual numbers or arrangement of atoms.
  • A. NaO
  • B. NaO
  • C. NaO 
  • D. NaO
  • 30 grams of 2-prapanol were mixed with excess acidified K2Cr207 and boiled under reflux for 20 minutes. The organic product was then collected by distillation. The yield of product was 75.0%. What is the mass of product produced?
  • H3​C−CHOH​−CH3​K2​Cr2​O7​(acidified)​H3​C−C∣∣O​−CH3​
  • Given weight of propan-2-ol =30g
  • Molecular weight of propan-2-ol =60g
  • Therefore, 
  • No. of moles of propan-2-ol reacted =30/60​=0.5 mol
  • Now the yield of product is 75%.
  • Therefore, 
  • No. of moles of propan-2-one =75/100​×0.5=0.375mol
  • Now, 
  • Molecular weight of propan-2-one =58g
  • Therefore, 
  • Mass of product collected =58×0.375=21.75g
  • A. 1.74g
  • B. 21.75g 
  • C. 2.74g
  • D. 29.g
  • According to which scientist, the probability of finding an electron at a certain position is possible?
  • The Schrödinger model assumes that the electron is a wave and tries to describe the regions in space, or orbitals, where electrons are most likely to be found.
  • A. Bohr’s
  • B. De-Broglie
  • C. Hund’s
  • D. Schrodinger 
  • Which gas in the discharge tube produces lightest canal ray particles?
  • A. Ar
  • B. He
  • C. H2 
  • D. Ne
  • Which element has the ground state electronic configuration of 1s2, 2s2, 2p6, 3s2, sp6?
  • Argon is a chemical element with the symbol Ar and atomic number 18. [Ne] 3s² 3p is its ground state electronic configuration.
  • A. Ar
  • B. Cl
  • C. Na
  • D. S
  • What is the proton (atomic number) of an element that has four unpaired electrons in its ground state?
  • A. 6
  • B. 14
  • C. 22
  • D. 26 
  • A gaseous mixture contains 9.6% NH, 22.6% N and 67.8% H gases.  If the total pressure is 50 atm, then the partial pressure of H is
  • A. 67.8  100 / 50
  • B. 50  100/100
  • C. 67.8  50/100 
  • D. 67.8 + 50/100
  • If we want to raise the temperature of one mole of an ideal gas by one kelvin, we have to provide how much amount of energy?
  • A. 0.0821 joules
  • B. 8.314 dm³-atm
  • C. 0.0821 Kj
  • D. 0.0821 dm³-atm 
  • The process of heat flow between hotter and colder gases remains continued until at the molecules have equal
  • When a particle moves it has kinetic energy and translational kinetic energy is responsible for the movement of particles in liquids and gases. When the molecules or particles attain a similar translational kinetic energy that indicates the heat has been distributed equally.
  • A. Average translational kinetic energy 
  • B. Average rotational kinetic energy
  • C. Average translational potential energy
  • D. Average vibrational kinetic energy
  • In liquid, with the change in dipole-dipole forces, there is a change in some physical properties. Select the property which is not affected by the strength of dipole-dipole forces?
  • The mole is the amount of substance of a system which contains as many elementary entities as there are atoms in 0.012 kilogram of carbon 12. Intermolecular forces are electrostatic in nature; that is, they arise from the interaction between positively and negatively charged species.
  • A. boiling point
  • B. heat of vaporization
  • C. heat of sublimation
  • D. moles 
  • Which of the following factor does not affect the magnitude of vapor pressure?
  • Vapor pressure is the pressure caused by the evaporation of liquids. Three common factors that influence vapor press are surface area, intermolecular forces, and temperature.
  • A. amount of liquid 
  • B. size of molecule
  • C. temperature of liquid
  • D. intermolecular forces
  • A small building block which belongs to whole information about crystal structure is called?
  • Unit Cell is the smallest part (portion) of a crystal lattice. It is the simplest repeating unit in a crystal structure. The entire lattice is generated by the repetition of the unit cell in different directions.
  • A. Cell
  • B. Unit Cell 
  • C. Crystal lattice
  • D. Crystal unit
  • Which type of solid is called as atomic solid?
  • Covalent-network (also called atomic) solids. Made up of atoms connected by covalent bonds; the intermolecular forces are covalent bonds as well. Characterized as being very hard with very high melting points and being poor conductors
  • A. Covalent solids 
  • B. Ionic solids
  • C. Metallic solids
  • D. Molecular solids
  • The decrease in solubility of the salt in a solution that already contains an ion common to that salt is known as:
  • The common ion effect describes the effect on ​equilibrium that occurs when a common ion (an ion that is already contained in the solution) is added to a solution. The common ion effect generally decreases ​solubility of a solute.
  • A. Le Chatelier’s principle
  • B. Solubility Product
  • C. Common ion effect 
  • D. Ksp
  • The precipitation occurs if the ionic concentration is:
  • Precipitation reactions occur when cations and anions in aqueous solution combine to form an insoluble ionic solid called a precipitate. 
  • Q
  • Q=Ksp. The solution is saturated and at equilibrium. 
  • Q>Ksp. The solution is supersaturated, and ionic solid will precipitate 
  • Q is ionic product concentration.
  • A. Less than ksp
  • B. More than ksp 
  • C. Equal to ksp
  • D. Present in any amount
  • One can estimate the direction in which equilibrium will shift with the help of:
  • Le Châtelier’s principle states that if a dynamic equilibrium is disturbed by changing the conditions, the position of equilibrium shifts to counteract the change to reestablish an equilibrium.
  • A. Le Chatelier’s principle 
  • B. Law of mass action
  • C. Mess’s law
  • D. Law of heat of formation
  • What is the overall order of this rate equation? Rate = k[H] [NO]²
  • The Order of the Reaction is the sum of powers of the reactant concentration. 1+2 = 3
  • A. 1
  • B. 2
  • C. 3 
  • D. 4
  • The catalysis in which the catalyst and the reactants are in the same phase is known?
  • When a catalyst is present in the same phase as the reactant and products it is called as a homogeneous catalyst. In these reactions, the catalyst, reactant, and products are in the same phase and are called homogeneous catalytic reactions or homogeneous catalysis.
  • A. Heterogeneous catalyst
  • B. Homogeneous catalyst 
  • C. Slow
  • D. Fast
  • Born-Haber cycle is used to determine the Lattice energy of ionic compounds. It is the application of: 
  • Hess’s law states that if a process can be expressed as the sum of two or more steps, the enthalpy change for the overall process is the sum of the ΔH values for each step.
  • A. Henry’s law
  • B. Le-Chatleir’s Principle
  • C. Hess’s law 
  • D. Common ion effect
  • Which of the following term is state function?
  • The properties which depends only on initial and final state, not on path or reaction intermediate are called state function.
  • A. Freezing
  • B. Decomposition
  • C. Sublimation
  • D. enthalpy 
  • An electrochemical cell is based upon which reaction?
  • An oxidation-reduction (redox) reaction involves the transfer of electrons from the reducing agent to the oxidizing agent. A voltaic electrochemical cell involves two half cells one containing an oxidizing agent and the other a reducing agent. salt bridge to complete the circuit and maintain electrical neutrality.
  • A. Acid-base reaction
  • B. Redox reaction 
  • C. Nuclear reaction
  • D. Neutralization reaction
  • In which of the following, oxygen shows fractional oxidation number?
  • We know potassium which is a member of group 1 and thus it can naturally react with oxygen. Again, we know KO2 is an super oxide and its is an neutral molecule so it has a net charge of zero. 
  • Let x be the oxidation state of Oxygen. Thus, we can write, 
  • 1+2x=0 where +1 is the oxidation state of potassium
  •               thus, x=-1/2
  •               Thus, the oxidation state of oxygen in KO2 is -0.5.
  • A. OF
  • B. NaO
  • C. KO 
  • D. ClO
  • Which of the following element has smaller size?
  • Atomic number of Na is 11, K is 19, Al is 13 and Li is 3. Lithium is the smallest in size.
  • A. Na
  • B. K
  • C. Al
  • D. Li
  • Among LiCl, BeCl, NaCl, CsCl, the compounds with the greatest and the least ionic character respectively are:
  • As the size of the cation increases ionic character increases, so among the given chlorides the Cs is having larger size and the element having least size is Be. So, the compound having more ionic character is CsCl.
  • A. LiCl and CsCl
  • B. NaCl and LiCl
  • C. CsCl and NaCl
  • D. CsCl and BeCl 
  • Which statement describes the conversion of magnesium atoms to magnesium ions for ionic bond formation with chlorine?
  • Mg has 12 electrons so it loses 2 electrons to stabilize itself and becomes an ion with 2+ charge. Loss of electrons is called oxidation.
  • A. The change is reduction, because there has been a gain of electrons
  • B. The change is oxidation, because there has been a loss of electrons 
  • C. The change is reduction, because there has been a loss of electrons
  • D. The change is oxidation, because there has been a gain of electrons
  • AB Type with no Lone Pairs geometry enables to form which shape of molecule?
  • The charge clouds due to four electron pairs avoid their electrostatic repulsions by drifting apart to maintain a mutual bond angle of 109.5°. Such geometry enables to a form a shape of regular tetrahedron
  • A. Trigonal
  • B. Regular tetrahedron 
  • C. Regular octahedron
  • D. Regular pyramidal
  • Why dimer of Aluminum chloride is formed
  • AlCl3 forms a dimer, as Al due to the presence of 3d−orbitals can expand its covalency from four to six. Also, dimerization enables Al atoms to complete their octets.
  • A. Aluminum is electron rich
  • B. Aluminum is having lone pair of electron
  • C. Aluminum donates lone pair to form bridge
  • D. Aluminum forms coordinate bonds with chlorine to complete its octet 
  • Which group of the periodic table contain non-metals, metalloids and metals?
  • Group 4A (or IVA) of the periodic table includes the nonmetal carbon (C), the metalloids silicon (Si) and germanium (Ge), the metals tin (Sn) and lead (Pb).
    IB
  • A. VII A
  • B. IV A 
  • C. VI A
  • Which of the following sulfate compound is insoluble in water?
  • All sulfates are soluble except barium, strontium, lead (II), calcium, silver, and mercury.
  • A. BeSO
  • B. BaSO 
  • C. MgSO
  • D. CaSO
  • Which of the following complex show a tetrahedral geometry?
  • A. [Fe(CO)5]
  • B. [Cu(CN)4]-2 
  • C. [Au(Cl)4]-
  • D. [Pt(NH3)4]+2
  • In which pair one has all unpaired d orbitals while other have all paired d orbitals?
  • A. Cu and Zn
  • B. Cr and Fe
  • C. Cr and Zn 
  • D. Mn and Co
  • In which of the following functional groups, the carbon atom is sp hybridized?
  • The carbon in the cyano group of a ketone has tone single bond to the other carbon atom and a triple bond with the nitrogen atom. This means that its hybridization is sp.
  • A. -CHO
  • B. -COOH
  • C. -CN 
  • D. -COOR
  • The compounds containing R-SH functional group are known as:
  • Alcohols: -OH, 
  • Thio-alcohols: R-S-H, 
  • Thio-ether: R-S-R, 
  • Nitrile: R −CN
  • A. Alcohols
  • B. Thio-alcohols 
  • C. Thio-ether
  • D. Nitrile
  • What is the number of isomers of a hydrocarbon having molecular formula, CH?
  • There are five constitutional isomers with the formula C4​H8​. They are
  • 1. But-1-ene
  • 2. But-2-ene
  • 3. 2-Methylpropane
  • 4. Cyclobutane
  • 5. methylcyclopropane
  • A. 2
  • B. 3
  • C. 4
  • D. 5 
  • Alkylbenzene is formed when benzene is treated with an alkyl halide in the presence of anhydrous aluminum chloride. Identify the type of reaction.
  • Friedel-Crafts Alkylation refers to the replacement of an aromatic proton with an alkyl group. This is done through an electrophilic attack on the aromatic ring with the help of a carbocation. The Friedel-Crafts alkylation reaction is a method of generating alkylbenzenes by using alkyl halides as reactants.
  • A. Halogenation
  • B. Friedel-Crafts acylation reaction
  • C. Friedel-Crafts alkylation reaction 
  • D. Sulphonation
  • Three alternate single and double bonds in benzene are called?
  • A conjugated system is a system of connected p-orbitals with delocalized electrons in compounds with alternating single and multiple bonds.
  • A. Conjugate bonds 
  • B. Coordinate covalent bonds
  • C. Fixed bonds
  • D. Ionic bonds
  • Which of the following compound is more acidic?
  • A hydrogen tends to be more acidic when attached to a more electronegative atom. Alkynes are more acidic than alkenes, which are in turn more acidic than alkanes, because sp carbon atoms are more electronegative than sp 2 carbon atoms, which are in turn more electronegative than sp 3 carbon atoms. cycloalkane has no electron delocalization, and thus is a much weaker acid.
  • A. Alkane
  • B. Alkene
  • C. Alkyne 
  • D. Cycloalkane
  • Consider the chlorination of methane, the attack of chlorine free radical on methane form methyl free radical occurs in?
  • The over-all process is known as free radical substitution, or as a free radical chain reaction. 
  • 1) Chain initiation: The chain is initiated (started) by UV light breaking a chlorine molecule into free radicals
  • Cl22Cl. 
  • 2) Chain propagation reactions: These are the reactions which keeps the chain going. 
  • CH4+Cl.CH.3+HCl
  • CH.3+Cl2CH3Cl+Cl. 
  • 3) Chain termination reactions: These are reactions which remove free radicals from the system without replacing them by new ones
  • 2Cl.Cl2
  • CH.3+Cl.CH3Cl
  • CH.3+CH.3CH3CH3
  • A. Initiation step
  • B. Propagation step 
  • C. Termination step
  • D. Last step
  • The ratio of sigma to Pi electrons in benzene is?
  • Sigma bonds are present in benzene- six sigma bonds between carbon-carbon atoms and six sigma bonds between carbon and hydrogen atoms, a total of 12 sigma bonds. The ratio of sigma and pi bonds is = 12 3 = 4 1 or 4 : 1
  • A. 1:3
  • B. 3:1
  • C. 4:1 
  • D. 1:4
  • When halogen is removed from an alkyl halide a carbocation is formed, identify the most reactive carbocation
  • Reactivity of SN1 reaction depends on the stability of the carbocation formed. Higher the stability of carbocation, higher will be its reactivity. 
  • Stability of carbocation follows the order, 
  • Tertiary > Secondary > Primary. 
  • Methyl carbocation is more stable as more alkyl groups are attached to the carbocation more inductive electron donation occurs and the carbocation becomes more stable.
  • A. Primary carbocation
  • B. Secondary carbocation
  • C. Tertiary carbocation
  • D. Methyl carbocation 
  • Freon is commonly known as?
  • Freon is a non-combustible gas that is used as a refrigerant in air conditioning applications. This freon undergoes an evaporation process over and over again to help produce cool air that can be circulated throughout your AC system.
  • A. Refrigerant 
  • B. A solvent
  • C. Insecticides
  • D. A fire extinguisher
  • Neopentylchloride belongs to which class of alkyl halides?
    Neopentyl chloride is a primary alkyl halide because the Cl-atom is linked to a primary carbon.
  • A. Primary alkyl halides 
  • B. Secondary alkyl halides
  • C. Tertiary alkyl halides
  • D. Quaternary alkyl halides
  • What is the common name of 1,2, 3,-propanetriol?
  • A. Glycerin or Glycerol.
  • B. Butyl alcohol
  • C. Glycol
  • D. Glycerol 
  • E. Propyl alcohol
  • Benzene is formed when Na reacts with which of the following?
  • A. Alcohol
  • B. Butyl alcohol
  • C. Propanol
  • D. Phenol 
  • When Phenol reacts with formaldehyde, which of the following product is produced?
  • An adduct is a product of a direct addition of two or more distinct molecules, resulting in a single reaction product containing all atoms of all components.
  • A. Adduct 
  • B. Hycronium ion
  • C. Oxonium ion
  • D. Phenoxide ion
  • Which of the following is the correct name of CHCHCHCOCHCHO?
  • A. 3-oxo hexanal 
  • B. 3-one hexanal
  • C. 3-oxo hexanol
  • D. 3 keto hexanol
  • Which is the most suitable reagent for the conversion of R-CHOH-RCHO?
  • A. KMnO/NaOH
  • B. KCrO/HSO(Conc.)
  • C. CrO
  • D. CrO/HSO(Conc.) 
  • Which of the following is also called silver mirror test?
    Tollens’ test, also known as silver-mirror test, is a qualitative laboratory test used to distinguish between an aldehyde and a ketone. It exploits the fact that aldehydes are readily oxidized, whereas ketones are not. Tollens’ test uses a reagent known as Tollens’ reagent, which is a colorless, basic, aqueous solution containing silver ions coordinated to ammonia.
  • A. Benedict’s solution test
  • B. Fehling’s solution test
  • C. Iodoform test
  • D. Tollen’s reagent test 
  • Which among the following have least pH?
  • Electron donating group decreases the strength of the acid Electron withdrawing group increases the strength of the acid. 
  • Cl is an electron withdrawing group, it increases the acidity.
  • A. CHCHCOOH
  • B. CHClCHCOOH
  • C. CHCHClCOOH 
  • D. CHCHCOOH
  • If carboxylic acid and ketone groups C = O are present in a chain then final name will be given as:
  • In the case of molecules containing a carboxylic acid and aldehydes and/or ketones functional groups the carbonyl is named as an “Oxo” substituent. 
  • The –e ending is removed from the name of the parent chain and is replaced -anoic acid.
  • A. oxo, oic acid 
  • B. one, oic acid
  • C. Both 1 and 2
  • D. None of these
  • When carboxylic acids and dicarboxylic acids have similar molecular weights, how do their melting points compare?
  • Dicarboxylic acids are solids at room temperature and they have melting points that are higher than those of monocarboxylic acids containing the same number of carbon atoms, since stronger associations between molecules exist, mainly as a result of hydrogen bond formation.
  • A. Carboxylic acids have greater melting points
  • B. Both acids have similar melting points
  • C. Dicarboxylic acids have greater melting points 
  • D. No any consistent trends exits
  • When food reaches stomach, the action of which of the following come to an end due to acidic pH?
  • The optimal pH for salivary amylase enzymatic activity ranges from 6 to 7. Over and below this level, the response rate decreases as the enzymes become denatured. The salivary amylase enzyme is most active at pH 6.8
  • A. Amylase 
  • B. Lipases
  • C. Maltase
  • D. Hydrolases
  • Which of the following proteins acts as carrier of copper in blood plasma?
  • A. Ceruloplasmin is a protein   that is made in the liver. It stores and carries copper from the liver into the bloodstream and to the parts of your body that need it.
  • B. Hemoglobin
  • C. GlycoproteinCeruloplasmin 
  • D. Histone

ENGLISH

  • Synonym of the word “Capricious” is:
  • Capricious: given to sudden and unaccountable changes of mood or behaviour.
  • Fickle: changing frequently, especially as regards one’s loyalties or affections.
  • A. Fickle 
  • B. Predictable
  • C. Uniform
  • D. Invariable
  • Diseases like diabetes are supposed to be taken seriously or they can be ………… Which of the following words will fill in the blank most appropriately?
  • The word fatal means causing death.
  • A. Cursing
  • B. Healthy
  • C. Fatal 
  • D. Impersonating
  • Choose the most appropriate antonym for “abandonment’:
  • Cessation: the fact or process of ending or being brought to an end.
  • Stoppage: an instance of movement, activity, or supply stopping or being stopped. 
  • halt: bring or come to an abrupt stop. 
  • extension: a part that is added to something to enlarge or prolong it.
  • A. Cessation
  • B. Stoppage
  • C. Halt
  • D. extension 
  • Fill in the blank with the correct word. The shepherd ploughed this mountain with cattle the first time it ………. Ever ploughed.
  • (Clearing in the sky)
  • A. was 
  • B. was been
  • C. had
  • D. had been
  • To give one some idea of Rabies’ horrors, one ………… only read such descriptions as the following: spasms, restlessness, shudders at the least breath of air, an ardent thirst, convulsive movements, and fits of furious age.
  • We use “need” with I, you, we, they, and plural nouns.
  • We use “needs” with he, she, it, and singular nouns
  • A. Needs
  • B. need 
  • C. needed
  • D. has needed
  • By 2030, people ……….. Been reading the works of Charles Dickens for more than 190 years.
  • The future perfect continuous, also sometimes called the future perfect progressive, is a verb tense that describes actions that will continue up until a point in the future. The future perfect continuous consists of will + have + been + the verb’s present participle (verb root + -ing).
  • A. Had
  • B. Will
  • C. Have
  • D. will have 
  • Choose the most suitable/appropriate sentence out of the following:
  • This is a past tense with the use of ‘did’ first form of verb is used. Example: I didn’t have time to watch TV yesterday.
  • A. Penny did not let me to get my book.
  • B. Penny was not leaving me to get my book.
  • C. Penny did not let me get my book. 
  • D. Penny had not left me get my book.
  • Which one of the following is correct?
  • past perfect tense.
  • A. We visited, Istanbul, Turkey, and Kowloon, Hong Kong last summer.
  • B. We visited: Istanbul, Turkey, and Kowloon, Hong Kong last summer.
  • C. We visited Istanbul, Turkey, Kowloon, Hong Kong last summer.
  • D. We visited Istanbul, Turkey, and Kowloon, Hong Kong last summer. 
  • Which of the following sentences is correct?
  • Spelling of persuade and mum are correct in chosen option.
  • A. How could Sarah perswad her mum to stay out later?
  • B. How could Sarah persuade her mum to stay out later? 
  • C. How could Sarah persuad her mum to stay out later?
  • D. How could Sarah persuade her mun to stay out later?
  • Choose the sentence with the correct use of article.
  • we should use definite article “the” with musical instruments.
  • A. Natasha can play a plane and a violin.
  • B. Natasha can play the piano and the violin. 
  • C. Natasha can play the piano and a violin.
  • D. Natasha can play piano and violin.
  • Distribute the handouts _________ the candidates. The correct preposition to be filled in I
    Among means ‘in the middle or included in a larger group of people or things’
  • A. Into
  • B. among 
  • C. in
  • D. on
  • Choose the correct sentence:
  • some nouns, like scissors, only have a plural form
  • A. These scissors are very sharp 
  • B. This scissors is very sharp
  • C. This scissor is very sharp
  • D. These scissor are very sharp
  • Identify the sentence, out of the following, that is error free:
  • the verb ‘being’ is used to refer to actions that are going on in the present or in the continuous tense
  • A. I do not enjoy being laughed at by other people 
  • B. I did not enjoy laughing by other people
  • C. I am not enjoying laughing by other people
  • D. I do not enjoying being laughed at other people
  • Choose the sentence that is grammatically correct.
  • boring is an –ing adjective that describes a thing or person that causes this feeling. Bored is an –ed adjective that describes the person that feels the effect of this feeling.
  • A. We agreed that the play was rather boring so we felt bored 
  • B. We agreed that the play was rather bored so we felt boring
  • C. We agreed that the play was rather bore so we felt bores
  • D. We agreed that the play was rather bores so we felt bored
  • I decided to sell the piece of land when I was offered more ___________ price. The most appropriate word to be filled in here is:
  • realistic means sensible and practical approach or idea.
  • A. True
  • B. realistic 
  • C. exact
  • D. perfect
  • “To cut off the head”. Idiom means:
  • defrock:  to deprive of the right to exercise the functions of office
  • decapitate: to behead or cut off head. 
  • impaled: to pierce with or as if with something pointed. 
  • urbanite: a person who lives in a city
  • A. defrock
  • B. decapitate 
  • C. impaled
  • D. urbanite
  • Wasim was so good at mathematics that people considered him to be a ________. Fill in the blank with the correct response.
  • Prodigy: a highly talented child or youth. 
  • prodigal: characterized by profuse or wasteful expenditure. 
  • primeval: of or relating to the earliest ages. 
  • profligate: wildly extravagant.
  • A. prodigy 
  • B. prodigal
  • C. primeval
  • D. profligate
  • The newly elected president and CEO for the newly established branch of our company …….. arrived recently. Fill the blank with the appropriate choice:
  • present tense is used. Have and has indicate possession in the present tense (describing events that are currently happening). Have is used with the pronouns I, you, we, and they, while has is used with he, she, and it.
  • A. Have
  • B. Having
  • C. have been
  • D. has 

ETEA MDCAT SOLVED PAST PAPER-2023

ETEA MDCAT SOLVED PAST PAPER-2023

For F.S.C. and Non-F.Sc. Students 

Time Allowed: 210 Minutes (3- ½ hours)

Instructions: 

i. Read the instructions on the MCQs Response Form carefully. 

ii. Choose the Single Best Answer for each question.

iii. Each correct Answer carries One Mark. There is No Negative Marking  

iv. Candidates are strictly prohibited from giving any identification mark except Roll. No. & Signature in the specified columns only. 

1. If 5 moles each of hydrogen and oxygen are reacted to form water, which of the following statements is true?

2H2 + O2 2H2O

A. H2 is excess reagent

B. O2 is limiting reagent

C. H2 is limiting reagent

D. Reaction has no limiting reagent

Explanation: For the balanced equation, 2 moles of H2 react with 1 mole of O2. With 5 moles of each present, H2 will be used up first. Thus, hydrogen will limit the amount of water that can be produced.

2. Oxygen can be prepared by the decomposition of potassium chlorate (HClO2). How many moles of oxygen O2(g) can be formed by taking 12 moles of potassium chlorate (KClO2) according to the following equation?

2KClO3(l) + heat 2KCl(l) + 3O2(g)

A. 12 moles of oxygen

B. 15 moles of oxygen

C. 18 moles of oxygen

D. 21 moles of oxygen

Explanation: 2 moles of KClO3 produce 3 moles of O2. Therefore, 12 moles of KClO3 will produce 18 moles of O2.

3. Considering a sample of NaCl weighing 117.0g, how many formula units of NaCl are present in it?

A. 1.204 x 1024 formula units of NaCl

B. 12.04 x 1022 formula units of NaCl

C. 1.204 x 1023 formula units of NaCl

D. 6.023 x 1023 formula units of NaCl

Explanation: The molar mass of NaCl is 58.5g/mole and Avogadro’s number is 6.023×10236.023×10 23. Thus, 117g corresponds to 2 moles of NaCl, or 1.204×10241.204×10 24 units.

4. For a sub-shell where n = 4 and l = 3, how many orientations are possible?

A. 5

B. 3

C. 7

D. 1

Explanation: The number of orientations (ml values) for a given l value ranges from -l to +l. For l=3, the orientations are -3, -2, -1, 0, 1, 2, 3, making a total of 7 orientations.

5. Which of the following series of the hydrogen spectrum is associated with the photon of the lowest wavelength?

A. Balmer series

B. Plund series

C. Bracket series

D. Paschen series

Explanation: The Balmer series corresponds to transitions to n=2 and usually falls in the visible range. The series with the lowest wavelengths (highest energy) are transitions to n=1, called the Lyman series.

6. The maximum e/m ratio for positive rays is obtained when the discharge tube contains which of the following?

A. He

B. N2

C. Ne

D. H2

Explanation: The e/m ratio refers to the charge-to-mass ratio. Hydrogen (H2) has the lightest mass among the given options. Thus, for positive rays or ions, it will have the maximum charge-to-mass ratio.

7. Which of the following is not equivalent to 1 atm of pressure?

A. 760 cm Hg

B. 760 mm Hg

C. 760 torr

D. 101325 Pa

Explanation: 1 atm is equivalent to 760 mm Hg, 760 torr, and 101325 Pa. All are standard measures of atmospheric pressure, and they’re used interchangeably based on the context.

8. If a given volume of an ideal gas at a certain pressure is x, what will be its volume when the pressure is reduced to half at constant temperature?

A. x/2

B. 2x

C. 4x

D. x/4

Explanation: According to Boyle’s law, the volume of a gas is inversely proportional to its pressure if the temperature is kept constant. If the pressure is halved, the volume will double.

9. What is the volume occupied by 3.2 g of oxygen gas at S.T.P?

A. 1.12 dm3

B. 2.24 dm3

C. 22.4 dm3

D. 24 dm3

Explanation: At STP, 1 mole of any ideal gas occupies 22.4 dm^3. Given that the molar mass of O2 is 32 g/mol, 3.2 g of O2 is 0.1 mole. Therefore, its volume at STP is 2.24 dm^3.

10. Which of the following crystal systems does not have β = γ = 90o?

A. Cubic

B. Orthorhombic

C. Tetragonal

D. Rhombohedral

Explanation: In the rhombohedral crystal system, the angles between the crystal axes are not 90 degrees. In the other systems mentioned, the angles between axes are right angles.

11. Which compound has the lowest boiling point? 

A. Carbon tetrachloride

B. Ethyl alcohol

C. Benzene 

D. Acetic acid

Explanation: Benzene has a boiling point of around 80°C. Ethyl alcohol boils at 78°C, but it has hydrogen bonding which makes it boil higher than benzene. Carbon tetrachloride and acetic acid have higher boiling points than benzene.

12. Which of the following can form Hydrogen bonding with each other? 

A. Methanal & Ethanal 

B. Propanone & ethyl methyl ketone 

C. 3’ Amine & H2O 

D. Acetone & Acetaldehyde

Explanation: 3’ Amine has a nitrogen atom with a lone pair and H2O has hydrogen atoms bonded to oxygen, making it capable of forming hydrogen bonds with the nitrogen of the amine.

13. Identify the molecular formula of Furan: 

A. C4H4O

B. C5H3O 

C. C4H5O

D. C4H3O

Explanation: Furan is a five-membered ring with four carbons and one oxygen. Its molecular formula is C4H4O.

14. Haber’s process is used for the synthesis of ammonia. The optimum temperature for the Haber process is: 

A. 35 – 50oC 

B. 130 – 150oC

C. 400 – 450°C 

D. 500 – 600oC

Explanation: The Haber process, which is used for ammonia synthesis, operates at high temperatures, typically around 400-450°C.

15. If the ionic product is less than Ktp, then: 

A. Solution will be saturated 

B. No precipitation will occur 

C. Solution will be super saturated 

D. Precipitation will occur

Explanation: If the ionic product of a solution is less than the solubility product (Ktp), the solution is unsaturated, and no precipitation occurs.

16. For ∆n=0: 

A. Kp=Kn 

B. Kp≠Kn 

C. Kp>Kn

D. Kp<Kn

Explanation: When the change in the number of moles (∆n) is zero, the equilibrium constant in terms of pressure (Kp) is equal to the equilibrium constant in terms of concentration (Kn).

17. Which reaction follows third-order kinetics? 

A. Decomposition of nitrogen dioxide 

B. Decomposition of hydrogen iodide

C. Gas phase oxidation of nitric oxide

D. Formation of hydrogen iodide

Explanation: The decomposition of hydrogen iodide, represented by 2HI H2 + I2, follows third-order kinetics.

18. Compared to exothermic reactions, the formation of an activated complex in an endothermic reaction requires: 

A. Greater Ea

B. Smaller Ea 

C. Equal Ea 

D. No Ea

Explanation: Endothermic reactions require more energy to proceed, meaning a higher activation energy (Ea) to form the activated complex.

19. The SI unit of specific heat capacity is: 

A. Jg-1K-1

B. JK-1

C. JmoleK-1 

D. KJ/mol

Explanation: Specific heat capacity is defined as the amount of heat required to raise the temperature of 1 gram of a substance by 1 Kelvin. Therefore, its SI unit is Jg-1K1.

20. Processes involving solids and liquids have their: 

A. ∆H = ∆E + P∆V 

B. ∆H = P∆V

C. ∆H = ∆E

D. ∆H = 0

Explanation: For processes involving solids and liquids, the volume change is negligible, so P∆V is effectively zero. Therefore, ∆H is approximately equal to ∆E.

21. In the redox reaction of SO2 with KMnO4 in an acidic medium: 

A. SO2 oxidizes KMnO4

B. SO2 reduces KMnO4 

C. KMnO4 is inert in an acidic medium

D. SO2 can’t undergo a redox reaction

Explanation: In an acidic medium, KMnO4 acts as an oxidizing agent. Therefore, SO2 reduces KMnO4 (i.e., KMnO4 gets reduced by SO2).

22. Zinc displaces copper from its solution because: 

A. The atomic number of zinc is higher than that of copper 

B. Zinc has a higher reduction potential than copper 

C. Zinc has a smaller reduction potential than copper 

D. Zinc is more soluble

Explanation: In the reactivity series, zinc lies above copper. Zinc has a smaller reduction potential than copper, making it more reactive. Therefore, zinc can displace copper from its solution.

23. The molecular shape of a molecule with three bonded atoms and one lone pair electron on the central atom will be: 

A. Trigonal planar 

B. Tetrahedral 

C. Trigonal pyramidal

D. Linear

Explanation: For a molecule with three bonded atoms and one lone pair on the central atom, the shape will be trigonal pyramidal. The four regions of electron density arrange themselves in a tetrahedral shape, but since one of the regions is a lone pair, the molecular shape is trigonal pyramidal.

24. The molecule having the highest bond energy (Experimentally) is: 

A. HCl 

B. HF 

C. HBr

D. HI

Explanation: HF has the highest bond energy among the given options. This is because fluorine is the most electronegative element, leading to a strong bond with hydrogen.

25. The bond length between double bonded carbon atoms is:

A. 1.34Ao 

B. 1.10Ao 

C. 1.54Ao 

D. 1.82Ao

Explanation: The double bond between carbon atoms, as found in alkenes, has a bond length of approximately 1.34 Angstroms.

26. Choose the element that forms a normal oxide only: 

A. Li 

B. Na

C. K 

D. Rb

Explanation: Among the given elements, lithium (Li) is the only one that forms a normal oxide (Li2O). The other elements (Na, K, Rb) can form peroxides or superoxides besides their normal oxides.

Top of Form

27. Element whose salts do not impart color to flame test is: 

A. Be 

B. Cs 

C. Ca

D. K

Explanation: Beryllium salts do not impart any distinct color to a flame test, unlike the other elements which give a characteristic flame color.

28. Indicate superoxide in the following: 

A. CaO 

B. BaO2 

C. Na2O2 

D. RbO2

Explanation: Superoxides have the general formula RO2, where R is an alkali metal. Among the given options, RbO2 (rubidium superoxide) is the superoxide.

29. Which of the following is covalent in nature:

A. Mg3N2

B. Be3N2 

C. Ca3N2 

D. Ba3N2

Explanation: Beryllium nitride (Be3N2) is covalent in nature due to the small size of the beryllium ion leading to overlapping of orbitals and sharing of electrons.

30. Which one has one unpaired electron in the valence shell? 

A. Zn+2 

B. Cu+ 

C. Ti+3 

D. Fe+3

Explanation: The Cu+ ion has an electronic configuration of [Ar]3d^10, which means there is no unpaired electron.

31. The oxidation number of Cobalt in the given coordination complex is: [Co(H2NCH2CH2NH2)3]2(SO4)3 

A. III 

B. II 

C. IV 

D. VI

Explanation: Given the overall charge on the complex is 2+ and each sulfate anion (SO4^2-) contributes 2- charge, the oxidation state of cobalt must be +3 to balance the charge.

32. Pyrole belongs to which class of compounds: 

A. Hydrocarbons 

B. Homocyclic 

C. Alicyclic 

D. Heterocyclic

Explanation: Pyrrole is a heterocyclic compound because it contains atoms of at least two different elements in its ring structure, namely carbon and nitrogen.

33. Ethyl alcohol in the presence of H2SO4 at 170o C produces: 

A. Ether

B. Ethene 

C. Ester

D. Ethane

Explanation: When ethyl alcohol (ethanol) is heated in the presence of concentrated sulfuric acid (H2SO4) at around 170°C, it gets dehydrated to produce ethene.

34. Lindlar’s catalyst is used in hydrogenation of alkyne:

A. To increases activation energy 

B. To enhance hydrogenation of alkene 

C. To prevent hydrogenation of alkene 

D. To start polymerization reaction

Explanation: Lindlar’s catalyst is a poisoned palladium catalyst used to convert alkynes to cis-alkenes, preventing the further hydrogenation to alkanes.

35. Which one of the following is not Meta directing group?

A. -COR 

B. -NO2 

C. -CHO 

D. -NR2

Explanation: Among the options provided, -NO2 is a strong deactivating group and is meta-directing. However, in the context of the question, -NR2 (amines) show ortho-para direction due to their +M effect. So, they are not meta directing.

36. Benzene-1,3-diol is also known as: 

A. Catechol

B. Resorcinol 

C. Hydroquinone 

D. O-Gresol

Explanation: The compound Benzene-1,3-diol, where the two hydroxyl groups are on positions 1 and 3, is known as Resorcinol.

37. Identify the electrophile called as acylium ion: 

A. R3N+ 

B. RCO+ 

C. RCOO+

D. RNO+

Explanation: An acylium ion has the structure RCO+ where R can be an alkyl or aryl group.

38. Among the following hydrocarbons which one has acidic hydrogen? 

A. C2H6 

B. C2H4 

C. C2H2 

D. C3H6

Explanation: Among the given hydrocarbons, C2H2 (acetylene) has a hydrogen atom that is bonded to a sp-hybridized carbon, making it more acidic than hydrogens in other hydrocarbons.

39. Greater number of alkyl groups on substrate favours: 

A. Substitution reaction 

B. Elimination reaction 

C. Oxidation reaction 

D. Free radical reaction

Explanation: Having more alkyl groups (bulkier substituents) on a substrate increases steric hindrance. This favors the elimination reaction over the substitution.

40. SN1 reactions mostly result in parallel racemization, in partial racemization there is:

A. Inversion only 

B. Retention only 

C. Equal inversion and retention

D. More inversion than retention

Explanation: In SN1 reactions, when the carbocation intermediate forms, it is planar. Nucleophiles can attack from either side, leading to products that have both the original configuration (retention) and the opposite configuration (inversion). This results in racemization, or an equal mixture of the two configurations.

41. Which of the following involve the same steps? 

A. E1 and E2 

B. E1 and SN1 

C. SN1 and Sg2 

D. E2 and SN1

Explanation: Both E1 (elimination unimolecular) and SN1 (substitution nucleophilic unimolecular) involve the formation of a carbocation intermediate, which is a two-step process.

42. The correct sequence of electronic configuration is:

A. 4p 5s 4d 5p 6s 4f 5d 

B. 4p 4s 4d 5p 5s 5f 6d 

C. 4p 4s 4d 5p 5s 4f 5d

D. 4p 3s 4d 5p 6s 4f 5d

Explanation: The correct sequence of filling of electrons based on energy levels is given by the option A.

43. When phenol reacts with excess of bromine in aqueous solution it results in the formation of: 

A. Ortho/para bromophenol

B. Meta-bromophenol 

C. 2,4,6-Tribromophenol 

D. 3,5,0 bromophenol

Explanation: Phenol undergoes electrophilic aromatic substitution with bromine, and in aqueous conditions, the ortho and para positions are both brominated to give 2,4,6-Tribromophenol.

44. Propylene glycol and trimethylene glycol are:

A. functional group isomers

B. Metamers 

C. Position isomers

D. Tautomers

Explanation: Metamers are isomers which have the same molecular formula but different alkyl groups on either side of the functional group. Propylene glycol and trimethylene glycol fall into this category.

45. The reduction of Aldehydes and Ketones to alkanes in the presence of Zinc amalgam and HCl is called: 

A. Clemmenson reduction 

B. Williamson’s synthesis 

C. Wolf-Kishner reduction 

D. Dow process

Explanation: The Clemmenson reduction involves the reduction of carbonyl compounds to alkanes using zinc amalgam in the presence of HCl.

46. Aldehyde and Ketone on reaction with hydroxylamine form: 

A. Hydrazine 

B. Hydrazone 

C. Oxime 

D. Imine

Explanation: Aldehydes and ketones react with hydroxylamine to form oximes.

47. Fehling’s solution works on the principle of redox reaction which results in:

A. Reduction of Aldehydes

B. Oxidation of Copper (II)

C. Oxidation of Aldehyde 

D. Oxidation of Ketone

Explanation: In the Fehling’s test, aldehydes reduce the blue solution of copper (II) to a red precipitate of copper(I) oxide, hence oxidizing Copper (II) to Copper (I).

48. In conversion of Acid halides to ester, pyridine is used to: 

A. Stabilize acid halides 

B. Consume HCl formed in the reaction 

C. Dehydrate alcohol

D. Dehydrogenate acid halides

Explanation: In the conversion of acid halides to esters using alcohols, pyridine is used to neutralize the HCl formed.

49. Number of carbon atoms in valeric acid is:

A. 4 

B. 5

C. 6 

D. 7

Explanation: Valeric acid, or pentanoic acid, has five carbon atoms in its structure.

50. The IUPAC name of formyl chloride is: 

A. Chloro methanoic acid 

B. Chloro methane 

C. Methanoyl chloride 

D. Chloro methanoate

Explanation: Formyl chloride is the acyl chloride of formic acid. Its IUPAC name is Methanoyl chloride.

51. The inhibition, in which the inhibitor does not combine directly with the enzyme but binds to the enzyme substrate complex is called: 

A. Reversible inhibition

B. Competitive inhibition 

C. Non-competitive inhibition 

D. Uncompetitive inhibition

Explanation: In uncompetitive inhibition, the inhibitor binds only to the enzyme-substrate complex, preventing the complex from releasing products and hence inhibiting the reaction.

52. Trypsinogen can be activated by the action of: 

A. Amylase 

B. HCl 

C. Glucokinase 

D. Enterokinase

Explanation: Trypsinogen is a precursor of the enzyme trypsin. It is activated by enterokinase (or enteropeptidase) to produce active trypsin.

53. Crystal having orthorhombic crystal system is:

A. PbCrO4

B. BaSO4 

C. ZnO 

D. K2Cr2O7

Explanation: Barium sulfate (BaSO4) crystallizes in the orthorhombic crystal system.

54. Alkoxy carbonyl functional group is present in: 

A. Ether 

B. Aldehyde

C. Carboxylic acid

D. Ester

Explanation: Esters have an alkoxy carbonyl functional group represented as -COOR where R is an alkyl group.

55. Does Pakistan need so many plans for development? 

  • Yes, Nothing can be achieved without proper planning. 
  • No. Too much time, money, and energy is wasted on planning.
  • Only I is true
  • Only II is true 
  • Either I or II is true
  • Neither I nor II is true

56. Methyl tetrachloride (MTC) is a chemical found in some pesticides, glues, and sealants. Exposure to MTC can cause people to develop asthma. In order to halve the nation’s asthma rate, the government plans to ban all products containing MTC. The government’s plan to halve the nation’s asthma rate relies on which of the following assumptions? 

A. Exposure to MTC is responsible for no less than half of the nation’s asthma cases 

B. Products containing MTC are not necessary to the prosperity of the country’s economy 

C. Asthma has reached epidemic proportions 

D. Banning MTC will not have negative side effects 

57. Statement: I. Importance of yoga and exercise is being realized by all sections of the society. II. There is an increasing awareness about health in the society, particularly among the middle-aged group of people. 

A. Statement I is the cause and statement II is its effect 

B. Statement II is the cause and statement I is its effect

C. Both the statement I and II are independent causes.

D. Both the statement I and II are effects of independent causes 

58. Statement: The rates of interest on Post Office recurring deposit accounts have been increased with effect from 1st March. This has been done to attract more deposits. Course of action. I. Efforts should also be made to make the public aware of this increase in the rate of interest. II. If the deposits don’t increase in the next six months, the rate of interest should be further increased. 

A. Only conclusion I follows 

B. Only conclusion II follows 

C. Both I and II follow

D. Neither I nor II follows 

59. Equal forces act on isolated bodies A and B. The mass of B is 1551​ times that of A. The magnitude of the acceleration of A is:

A. 1/5 times that of B

B. 1/3 times that of B

C. the same as B

D. Nine times that of B

Explanation: Newton’s second law of motion states F=m×a, where F is the force, m is the mass, and a is the acceleration. 

Given equal forces on both bodies and the mass relation provided, the relationship between the accelerations is inversely proportional to their masses. When comparing body A (with mass m) and body B (with mass 55m), the acceleration of A will be 5 times greater than that of B. Therefore, the acceleration of A is D. Nine times that of B.

60. A body is moved through a displacement of 15 km towards the north. The total displacement will be zero when the body covered the same displacement towards:

A. North

B. South

C. West

D. East

Explanation: Displacement is a vector quantity, which means it has both magnitude and direction. To nullify (or counteract) a displacement of 15 km to the north, the body would need to move 15 km in the exact opposite direction, which is the south. Thus, to have a total displacement of zero, the body should move 15 km towards the South.

61. When the range of a projectile is equal to one fourth of the height, then which one is true:

A. Tanθ = 10

B. Tanθ = 12

C. Tanθ = 16

D. Tanθ = 1

Explanation: For a projectile launched on level ground, the maximum height H it reaches is given by 2×sin⁡222gu2×sin2θ​ and the range R is given by 2×sin⁡(2)gu2×sin(2θ)​, where u is the initial velocity, θ is the launch angle, and g is the acceleration due to gravity. When R is 1441​ of H, after simplifying the equations, it can be deduced that tan tan(θ) is equal to 1. Thus, the correct option is D. Tanθ = 1.

62. The square root of the ratio of elasticity to mass density is equal to:

A. Force

B. Product of frequency and wavelength

C. Co-efficient of viscosity

D. Refractive index

Explanation: The square root of the ratio of elasticity (Y) to mass density (ρ) gives the speed of sound in the medium, represented as v. This speed of sound is also equal to the product of frequency (f) and wavelength (λ). Therefore, v=f×λ.

63. If the pressure of air enclosed in a long tube is increased 10 times, then the speed of sound will:

A. Increase

B. Decrease

C. Remain constant

D. Become zero

Explanation: The speed of sound in gases is given by v=γρP​​, where γ is the adiabatic constant, P is the pressure, and ρ is the density. If the pressure is increased, but the gas is not allowed to expand (as the tube is long), the density also increases proportionally. Hence, the speed remains constant.

64. The expression, Cp – R is equal to:

A. Cp

B. R

C. Cp – CN

D. R + Cp

Explanation: For an ideal gas, CpCv=R, where Cp is the heat capacity at constant pressure, Cv is the heat capacity at constant volume, and R is the universal gas constant. Thus, CpR=Cv. Here, CN seems to be a typographical error for Cv.

65. In a certain process, 200J of heat energy is supplied to a system and at the same time 50J of work is done by the system. The increase in the internal energy of the system is:

A. 25J

B. 100J

C. 150J

D. 250J

Explanation: According to the first law of thermodynamics: ΔU=QW, where ΔU is the change in internal energy, Q is the heat supplied, and W is the work done by the system. So, Δ=200−50=150ΔU=200J−50J=150J.

66. Work done during isochoric process is:

A. Negative

B. Maximum Negative

C. Maximum Positive

D. Zero

Explanation: An isochoric process occurs at constant volume. Since there’s no change in volume, no work is done by the system (Work W=PΔV, and Δ=0ΔV=0 for isochoric process). Thus, work done is zero.

67. Two charges of magnitude q1 = 1 μc and q2 = 5 μc, are separated at a distance r = 1 x 10-3m apart, the ratio of the magnitude of the forces acting on them will be:

A. 1:5

B. 2:25

C. 1:3

D. 1:1

Explanation: According to Coulomb’s law, the force due to two charges is directly proportional to the product of their charges and inversely proportional to the square of the distance between them. The force on each charge due to the other will be equal in magnitude but opposite in direction, so the ratio is 1:1.

68. The square root of the ratio of elasticity to mass density is equal to:

A. Force

B. Product of frequency and wavelength

C. Co-efficient of viscosity

D. Refractive index

Explanation: The speed of sound in a medium is represented by = v=ρE​​ where E is the modulus of elasticity and ρ is the mass density. This speed can also be expressed as the product of the frequency (f) and the wavelength (λ), so =×v=f×λ.

69. If the pressure of air enclosed in a long tube is increased 10 times, then the speed of sound will:

A. Increase

B. Decrease

C. Remain constant

D. Become zero

Explanation: The speed of sound in a gas primarily depends on the temperature and not the pressure, especially if the density changes correspondingly with pressure, which keeps the speed constant.

70. The expression, Cp – R is equal to:

A. Cp

B. R

C. Cp – CN

D. R + Cp

Explanation: The relation between heat capacities at constant pressure (Cp) and constant volume (CN) with the gas constant (R) is given by: CpCN=R. Thus, CpR is equivalent to CN.

71. In a certain process, 200J of heat energy is supplied to a system and at the same time 50J of work is done by the system. The increase in the internal energy of the system is:

A. 25J

B. 100J

C. 150J

D. 250J

Explanation: Using the first law of thermodynamics, Δ=−ΔU=QW, where ΔΔU is the change in internal energy, Q is heat supplied, and W is work done by the system. Using the given values, Δ=200−50=150ΔU=200J−50J=150J.

72. Work done during an isochoric process is:

A. Negative

B. Maximum Negative

C. Maximum Positive

D. Zero

Explanation: An isochoric process implies a constant volume process. Since there’s no volume change, no work is done by or on the system. Thus, the work done is zero.

73. Two charges of magnitude q1 = 1 μc and q2 = 5 μc are separated at a distance r = 1 x 10^-3m apart. The ratio of the magnitude of the forces acting on them will be:

A. 1:5

B. 2:25

C. 1:3

D. 1:1

Explanation: According to Coulomb’s Law and Newton’s third law, the force of attraction or repulsion between two charges is equal in magnitude but opposite in direction. So, the forces acting on the two charges due to each other will have the same magnitude.

74. For 0.5 elements of conductance, resistance will be:

A. 1Ω

B. 2Ω

C. 20Ω

D. 10Ω

Explanation: Conductance (G) is the reciprocal of resistance (R). If G = 0.5, then R = 1=10.5=2ΩG1​=0.51​=2Ω.

75. 1 volt x 1 ampere is equal to:

A. 1 coulomb

B. 1 newton

C. 1 watt

D. 1 hp

Explanation: Power (P) is the product of voltage (V) and current (I). Therefore, 1 volt x 1 ampere gives a power of 1 watt.

76. A wire of length l has resistivity p, if the wire is divided in two halves, then resistivity of each halve is:

A. p/2

B. p

C. 2/p

D. p/3

Explanation: Resistivity is an intrinsic property of the material of the wire and does not change regardless of the wire’s length, area, or volume. Thus, when the wire is divided, the resistivity remains the same.

77. In how many hours will a 1000 watt AC consume one unit of electricity?

A. 0.5 hr

B. 1 hr

C. 1.5 hr

D. 2 hr

Explanation: One unit of electricity is equivalent to 1 kilowatt-hour (kWh). Thus, a 1000 watt (or 1 kW) appliance will consume 1 kWh in 1 hour.

78. Of the following, the copper conductor that has the least resistance must be:

A. Thick, short and cool

B. Thin, long and hot

C. Thick, long and hot

D. Thin, short and cool

Explanation: Resistance in a conductor is inversely proportional to its cross-sectional area (thickness) and directly proportional to its length. Additionally, as temperature increases, resistance in most conductors (like copper) also increases. Thus, a thick, short, and cool conductor would have the least resistance.

79. A certain x-ray tube requires a current of 5 mA at a voltage of 60 kV. The rate of energy dissipation (in watts) is:

A. 560

B. 300

C. 200

D. 800

Explanation: Power (P) is calculated as the product of current (I) and voltage (V). Using the formula =×P=I×V, =5×10−3×60×103=300P=5×10−3A×60×103V=300W.

80. Ampere second is the unit of:

A. Power

B. Charge

C. Potential difference

D. Current

Explanation: Charge (Q) is calculated as the product of current (I) and time (t). Thus, its unit is Ampere times second (A.s).

81. The magnitude of the magnetic flux through area A has exactly the same value at angles:

A. 0° and 180°

B. 90° and 180°

C. 180° and 270°

D. 270° and 360°

Explanation: Magnetic flux is maximal when the magnetic field is perpendicular to the area. At 0° and 180°, the field is perpendicular, thus having the same flux magnitude.

82. A hydrogen atom that has lost its electron is moving east in a region where the magnetic field is directed from south to north. It will be deflected:

A. Up

B. Down

C. North

D. South

Explanation: Using the right-hand rule for positive charges (as a hydrogen atom that has lost an electron is positively charged), if the thumb represents the direction of current (east), and the fingers represent the direction of the magnetic field (south to north), the palm would push upwards, indicating the direction of the force.

83. Where the magnetic force acts on a charged particle, change occurs in:

A. Magnitude of velocity only

B. Magnitude and direction of velocity

C. Only the direction of velocity

D. Neither direction nor magnitude of velocity

Explanation: The magnetic force on a charged particle changes its direction but does not change the particle’s speed (or magnitude of its velocity).

84. A constant magnetic field of 5T is passing through a static conducting loop of area 0.8m^2, the induced emf is:

A. 4 V

B. 10 V

C. Zero

D. 32 V

Explanation: Faraday’s law states that an emf is induced in a loop when there’s a change in magnetic flux. If the magnetic field is constant and the loop is static, there’s no change in flux, so the induced emf is zero.

85. Magnetic flux linked with a conducting loop “A” increases at the rate of 15 weber/second and in another conducting loop “B” decreases at the rate of 15 weber/seconds, the magnitude of induced emf of:

A. Loop A will be equal to loop B

B. Loop A will smaller than loop B

C. Loop B will smaller than loop A

D. Both will be zero

Explanation: According to Faraday’s law, the magnitude of the induced emf in a loop is proportional to the rate of change of magnetic flux. Even though the flux is increasing in one loop and decreasing in the other, the rate of change is the same, so the magnitude of the induced emfs will be equal.

86. For induced emf to be produced in a coil, the magnetic flux linked with a coil must:

A. Only decrease

B. Only increase

D. Be changed

C. Be constant

Explanation: Faraday’s law states that an emf is induced in a loop when there’s a change in magnetic flux. Whether the flux increases or decreases doesn’t matter; what’s important is that there’s a change.

87. For half wave rectification, the number of diodes needed in a circuit is:

A. 1

B. 2

C. 3

D. 4

Explanation: Half-wave rectification allows one half of an AC wave to pass while blocking the other half. This can be achieved using a single diode.

88. The frequency of a light beam “A” is half of that of light beam “B”, the ratio E4/Es of photons energies is:

A. 1/2

B. 1/4

C. 1

D. 2

Explanation: Energy of a photon is directly proportional to its frequency. Thus, if the frequency of photon A is half of that of B, its energy will also be half. The ratio E4 (A) to Es (B) would be 1/2.

89. The wavelength of photon “A” is half the wavelength of photon “B”. The energy of a photon “A” is:

A. Half the energy of a photon B

B. One-fourth the energy of a photon B

C. Equal to the energy of photon B

D. Twice the energy of photon B

Explanation: Energy is inversely proportional to wavelength. So, if the wavelength of photon A is half that of B, its energy will be double that of B.

90. If frequency of light is greater than threshold frequency then in the photoelectric effect the number of electrons increases with an increase in ________ of light:

A. frequency

B. kinetic energy

C. intensity

D. momentum

Explanation: In the photoelectric effect, while the kinetic energy of emitted electrons depends on the frequency of the incident light, the number of electrons emitted is determined by the light’s intensity.

91. The ratio of shortest wavelength to longest of Lyman series is:

A. P

B. 4/3

C. 3/4

D. 4

Explanation: The Lyman series corresponds to electron transitions to the n=1 orbit. The shortest wavelength is from infinity to n=1, and the longest is from n=2 to n=1. The energy (and thus the inverse of wavelength) difference is greater between infinity and n=1 than between n=2 and n=1 by a ratio of 4/3.

92. Half-life of a radioactive element is 100 days. How much quantity of the 88g of such an element will remain after three half-lives:

A. 8g

B. 10g

C. 11g

D. 44g

Explanation: After each half-life, the quantity of the element halves. After 3 half-lives (100 days x 3), 88g will have halved three times: 44g, then 22g, then 11g.

93. At the end of 15 min, 1/32 of a sample of radioactive polonium remains. The corresponding half-life is:

A. 0.469 min

B. 32 min

C. 15 min

D. 3 min

Explanation: If only 1/32 remains, it means 5 half-lives have passed (since 125251​ = 1/32). Therefore, 5 half-lives took 15 minutes, implying one half-life is 3 minutes.

94. Two equal and opposite charges of 10 C are separated at a distance of 10cm. The electric potential at the midpoint between the charges is:

A. 20 V

B. 10 V

C. 5 V

D. 0 V

Explanation: When two charges are equal and opposite, the electric potential due to each charge at the midpoint will be equal in magnitude but opposite in sign. As a result, they will cancel each other out, resulting in a net electric potential of 0 V.

95. The forces of repulsion between two alike charges are 10N in a vacuum. When a material of Ep​ = 2 is placed between them, the new force will be:

A. 20 N

B. 15 N

D. 5 N

C. 10 N

Explanation: The electric force between charges in a medium is =F=EpF​. Given Ep​ is 2, the force will be half of the original force in vacuum, i.e., 5 N.

96. The slope of the charge-time graph for a charging capacitor gives:

A. Current

B. Voltage

C. Force

D. Energy

Explanation: The rate of change of charge with respect to time is the current. Thus, the slope of a charge-time graph represents the current flowing through the capacitor.

97. Two small charged objects attract each other with a force F when separated by a distance d. If the charge on each object is reduced to one-fourth of its original value and the distance between them is reduced to d/2, the new force is:

A. F/16

B. F/8

C. F/4

D. F/2

Explanation: Using Coulomb’s law, the force between charges is directly proportional to the product of the charges and inversely proportional to the square of the distance between them. If both charges are reduced to 1/4 and distance is halved, the new force is =×(1/4×1/4)(1/2)2=/8F=(1/2)2F×(1/4×1/4)​=F/8.

98. Something odd was happening all around the world. (Identify the underlined clause)

A. Subordinate clause

B. Independent clause

C. Dependent clause

D. Relative clause

Explanation: The given clause, “Something odd was happening all around the world,” is a complete thought that can stand alone. Therefore, it is an independent clause.

99. We met rather few people who spoke English. (The sentence is)

A. Complex

B. Simple

C. Compound

D. Compound complex

Explanation: A complex sentence has one independent clause and at least one dependent clause. In this sentence, “We met rather few people” is the independent clause, and “who spoke English” is the dependent clause.

100. Identify the sentence with correct punctuation and capitalization.

A. The automobile dealer handled three makes of cars: Toyota, Suzuki, and Honda.

B. The automobile dealer handled three makes of cars; Toyota, Suzuki, and Honda.

C. The automobile dealer handled three makes of cars: toyota, suzuki, and honda.

D. The automobile dealer handled three makes of cars, Toyota, Suzuki, and Honda.

Explanation: In the list of car brands, a colon is used to indicate the start of a list. The brands are proper nouns and thus should be capitalized, and the sentence should end with a period.

101. Which of the following sentences is in the correct order?

A. We up right drove to Karachi in two days.

B. We to Karachi drove in two days right up.

C. We right drove to Karachi up in two days.

D. We drove right up to Karachi in two days.

Explanation: This sentence correctly orders the verb and its complements, making it clear and easy to understand. The phrase “right up to Karachi” properly describes the directness and urgency of the journey.

102. The size of synaptic cleft (the gap separating nerve cells) in electrical synapse is:

A. 0.2 nm

B. 2 nm

C. 20 nm

D. 200 nm

Explanation: The synaptic cleft in electrical synapses is approximately 20 nm.

103. The activities like sleeping, walking, and dreaming are controlled by which part of the brain:

A. Pons

B. Cerebellum

C. Olfactory bulb

D. Hippocampus

Explanation: The pons plays a key role in sleep and arousal, among other functions.

104. The hormone “Calcitonin” is secreted by:

A. Thyroid gland

B. Parathyroid gland

C. Pancreas

D. Adrenal gland

Explanation: Calcitonin is produced in the C cells of the thyroid gland and plays a role in calcium homeostasis.

105. Fallopian tube is also known as:

A. Oviduct

B. Ovaries

C. Uterus

D. Cervix

Explanation: The fallopian tube is another name for the oviduct, which connects the ovaries to the uterus.

106. The syphilis is a _______ disease:

A. Bacterial

B. Fungal

C. Protozoic

D. Viral

Explanation: Syphilis is caused by the bacterium Treponema pallidum.

107. The most common symptom of gonorrhea in males is:

A. Skin rash on neck

B. Joint pain

C. Painful urination

D. Chancre

Explanation: Gonorrhea is a bacterial STD that can lead to painful urination among other symptoms.

108. G. J. Mendel chose _________ as an experimental plant for his study:

A. Medicago satia

B. Pisum sativum

C. Delonix regio

D. Lens culinaris

Explanation: Mendel carried out his classic genetics experiments with Pisum sativum, the pea plant.

109. When round and yellow color seeded plant (RrYy) is self crossed the number of plants with round and yellow color seeds obtained is ________ in 32 plants:

A. 18

B. 9

C. 3

D. 1

Explanation: A dihybrid cross between plants with genotype RrYy results in a phenotypic ratio of 9:3:3:1 for round-yellow, round-green, wrinkled-yellow, and wrinkled-green seeds respectively. In a set of 32 plants, about 9/16 would be round-yellow, which is 18 plants.

110. In crossing over, the exchange of maternal and paternal chromatid parts occurs while homologous chromosomes are paired during:

A. Prophase of meiosis I

B. Prophase of meiosis II

C. Metaphase of meiosis I

D. Metaphase of meiosis II

Explanation: Crossing over, or recombination, occurs during the prophase of meiosis I when homologous chromosomes pair up in a process known as synapsis.

111. Drosophila melanogaster is the scientific name of:

A. Mouse fly

B. Fruit fly

C. Flash fly

D. Sand fly

Explanation: Drosophila melanogaster is the scientific name for the fruit fly, a common model organism in genetics research.

112. A condition that renders the individual less able to form a blood clot is:

A. Alport’s syndrome

B. Coffin-Lowry syndrome

C. Hemophilia

D. Thalassemia

Explanation: Hemophilia is a genetic disorder where the blood doesn’t clot normally because it lacks sufficient blood-clotting proteins.

113. Intra-specific struggle refers to:

A. Competitive between members of the same species

B. Competitive between members of different species

C. Environmental struggle

D. Prey or predation

Explanation: “Intra-specific” refers to within the same species. So, intra-specific struggle refers to competition between individuals of the same species for resources.

114. Identify the fossil bird among the following:

A. Equus

B. Python

C. Archaeopteryx

D. Dawn horse

Explanation: Archaeopteryx is considered to be an ancient transitional species between modern birds and their dinosaur ancestors.

115. Which of the following organelles is regarded as a self-replicating organelle:

A. Endoplasmic reticulum

B. Golgi bodies

C. Mitochondria

D. Vacuole

Explanation: Mitochondria have their own DNA and can replicate independently within the cell, making them self-replicating organelles.

116. The prokaryotic cell wall is composed of:

A. Cellulose

B. Chitin

C. Murein

D. Pectin

Explanation: The prokaryotic cell wall, particularly in bacteria, is composed of murein (or peptidoglycan).

117. ________ is found in the exoskeleton of crabs.

A. Cellulose

B. Chitin

C. Murein

D. Hemi-cellulose

Explanation: Chitin is the main component of the exoskeletons of insects, arachnids, and crustaceans (like crabs).

118. Choose the unsaturated fatty acid among the following:

A. Palmitic acid

B. Arachidic acid

C. Stearic acid

D. Oleic acid

Explanation: Oleic acid is a monounsaturated omega-9 fatty acid, making it unsaturated.

119. The conjugated molecule that is primarily present in egg albumin is:

A. Lipoprotein

B. Nucleoprotein

C. Glycolipid

D. Glycoprotein

Explanation: Albumin in egg whites is primarily a glycoprotein, which means it’s a protein linked with a carbohydrate.

120. FSH (Follicle Stimulating Hormone) stimulates spermatogenesis by stimulating ________ cells to complete the development of sperms.

A. Leydig

B. Inhibin

C. Sertoli

D. TSH

Explanation: Sertoli cells support and nourish the developing sperm cells and respond to FSH, facilitating the process of spermatogenesis.

121. The process of photophosphorylation takes place in:

A. Nucleolus

B. Chloroplast

C. Nucleus

D. Golgi bodies

Explanation: Photophosphorylation is the process of ATP formation during light reaction in photosynthesis. This occurs in the thylakoid membrane of the chloroplasts.

122. Which of the following hormones is released by the posterior lobe of the pituitary gland?

A. Antidiuretic hormone

B. Growth hormone

C. Estrogen

D. Prolactin

Explanation: The posterior pituitary releases two hormones: oxytocin and antidiuretic hormone (ADH). ADH helps regulate water balance in the body.

123. Which of the following agents can easily pass through filter paper?

A. Bacteria

B. Fungi

C. Yeast

D. Virus

Explanation: Viruses are much smaller than bacteria, fungi, and yeast. They can pass through filters that can trap bacteria, a property historically used to identify viruses.

124. HIV belongs to a special class of virus known as:

A. Retrovirus

B. Flavivirus

C. Norovirus

D. Tetarvirus

Explanation: HIV is a retrovirus. Retroviruses have RNA as their genetic material and use an enzyme called reverse transcriptase to produce DNA from RNA.

125. The capsule of a bacterial cell is a sticky gelatinous structure made up of protein and:

A. Lipid

B. Carbohydrate

C. Nucleotide

D. Alcohol

Explanation: The bacterial capsule is mainly composed of polysaccharides (carbohydrates) and provides protection to the bacterial cell.

126. The transgenic plants, such as golden rice, are produced primarily with the help of:

A. Escherichia coli

B. Proteus mirabilis

C. Agrobacterium tumefaciens

D. Ralstonia solanacearum

Explanation: Agrobacterium tumefaciens is used as a natural genetic engineer to introduce foreign genes into plant cells, resulting in the creation of transgenic plants.

127. Which of the following chemicals is used to preserve biological specimens?

A. Formalin

B. Glutaraldehyde

C. Ethylene oxide

D. Iodine

Explanation: Formalin, which is a solution of formaldehyde in water, is commonly used to preserve biological specimens.

128. Coral reefs are the characteristic feature of phylum:

A. Porifera

B. Coelenterata

C. Arthropoda

D. Platyhelminthes

Explanation: Coral reefs are made up of coral polyps, which belong to the phylum Coelenterata (or Cnidaria).

129. According to the sliding filament hypothesis, the release of ________ ions from the sarcoplasmic reticulum causes the reorientation of certain components in this filament, permitting them to bind with extensions from the thick myosin filaments.

A. Sodium

B. Calcium

C. Magnesium

D. Potassium

Explanation: The sliding filament theory proposes that muscle shortening and contraction occur due to the interaction between actin and myosin filaments. Calcium ions released from the sarcoplasmic reticulum bind to troponin, causing a shift that allows myosin heads to bind with actin.

130. The green glands found in arthropods are concerned with:

A. Excretion

B. Respiration

C. Digestion

D. Circulation

Explanation: In arthropods, green glands (also known as antennal or maxillary glands) are excretory organs that help in osmoregulation and excretion of waste products.

131. The Purkinje fibers are specialized fibers found in:

A. Human heart

B. Human lungs

C. Human stomach

D. Human brain

Explanation: Purkinje fibers are specialized muscle fibers located within the heart. They play a crucial role in the heart’s electrical conduction system, helping coordinate the heart’s contractions.

132. The largest body of lymphoid tissue in the human body is

A. Tonsils

B. Pancreas

C. Spleen

D. Thymus

Explanation: The spleen is the largest lymphoid organ in the body, responsible for filtering blood and recycling old red blood cells.

133. The immune cells that clean up pus as a part of the healing process is:

A. Natural killer cell

B. Macrophages

C. Antibodies

D. Neutrophils

Explanation: Macrophages are cells that engulf and digest cellular debris, foreign substances, and microbes, making them key components in the healing process.

134. Lipid synthesis occurs in which one of the following organelle of the cells?

A. Ribosome

B. Golgi bodies

C. Mitochondria

D. Endoplasmic reticulum

Explanation: The smooth endoplasmic reticulum is the site where lipids are synthesized and metabolized.

135. The testes are male gonads which are situated outside the abdomen within a skin pouch called:

A. Vas deferens

B. Epididymis

C. Scrotum

D. Vasa efferentia

Explanation: The scrotum is a pouch of skin containing the testicles in males.

136. The term polydipsia refers to:

A. A condition in which an abnormally large volume of urine is produced

B. A condition of excessive thirst

C. A condition of excessive hunger

D. A condition in which the concentration of blood decreases in the body

Explanation: Polydipsia is excessive thirst or excess drinking usually associated with diseases such as diabetes.

137. Lactose is composed of:

A. Glucose + Fructose

B. Glucose + Galactose

C. Glucose + Glucose

D. Fructose + Glucose

Explanation: Lactose is a sugar composed of the monosaccharides glucose and galactose.

138. Saprophytic bacteria are:

A. Autotrophs

B. Decomposers

C. Parasites

D. Photosynthetic

Explanation: Saprophytic bacteria obtain their nutrients from dead organic matter, hence they are decomposers.

139. In the human heart, the right atrium communicates with the right ventricle through:

A. Inter-auricular septum

B. Ventricular septum

C. Tricuspid valve

D. Bicuspid valve

Explanation: The right atrium and right ventricle are separated by the tricuspid valve.

140. The first segment of the small intestine is:

A. Duodenum

B. Jejunum

C. Ileum

D. Colon

Explanation: The duodenum is the first section of the small intestine where initial digestion occurs.

141. Which one of the following factors does not affect the rate of enzyme action?

A. Outer membrane

B. Inner membrane

C. Granum

D. Stroma

Explanation: Enzyme action is primarily influenced by factors like temperature, pH, and substrate concentration. Stroma, the fluid-filled space inside the chloroplast, doesn’t directly affect enzyme action rate.

142. Light reaction occurs in the ________ of the chloroplast:

A. Outer membrane

B. Inner membrane

C. Granum

D. Stroma

Explanation: Light reactions of photosynthesis occur in the thylakoid membranes of the granum in chloroplasts.

143. How many pieces of ribonucleic acid (RNA) make up the genome of the influenza virus?

A. 4

B. 6

C. 8

D. 10

Explanation: The influenza virus genome is segmented and consists of 8 separate RNA molecules.

144. In a Mendelian cross for a heterozygous flower (Pp), what is the probability that the dominant allele will be in sperm and the recessive in the egg:

A. 0.05

B. 0.5

C. 0.25

D. 0.75

Explanation: Given that the flower is heterozygous (Pp), the probability of the dominant allele (P) being in the sperm is 0.5, and the probability of the recessive allele (p) being in the egg is also 0.5. The combined probability is 0.5 x 0.5 = 0.25.

145. Mastication causes exocrine glands under the tongue and in the back of the mouth to secrete a watery liquid called:

A. Bolus

B. Bile

C. Pancreatic juice

D. Saliva

Explanation: Mastication, or chewing, stimulates the salivary glands to produce saliva which helps in the digestion of food.

146. In phylum Ascheiminths (Nematoda), the nervous system consists of a nerve ring which encircles the _______ and sends its branches to different parts of the body.

A. Lips

B. Teeth

C. Stomach

D. Pharynx

Explanation: In nematodes, the nerve ring surrounds the pharynx, from which nerves extend both anteriorly and posteriorly, innervating various parts of the body.

147. Salmonella typhi causing typhoid and clostridium tetani causing tetanus are which type of bacteria?

A. Bacilli

B. Spirilla

C. Cocci

D. Comma

Explanation: Both Salmonella typhi and Clostridium tetani are rod-shaped bacteria, which are categorized as bacilli.

148. Who formulated the principle that ‘Ontogeny recapitulates phylogeny’?

A. Von Baer

B. Haeckel

C. Herbert Spencer

D. Darwin

Explanation: The idea that the development of an organism (ontogeny) expresses all the intermediate forms of its ancestors throughout evolution (phylogeny) was proposed by Ernst Haeckel.

149. Aquatic mammals belong to the order _______

A. Cetacea

B. Pholidota

C. Chiroptera

D. Proboscidea

Explanation: The order Cetacea includes aquatic mammals like whales, dolphins, and porpoises.

150. All of the following paired organs/structures have homology except:

A. Human hand: bat’s wing

B. Bat’s wing: wings of butterfly

C. Horse front leg: front flippers of whale

D. Wings of bird: Wings of flying lizards

Explanation: Bat’s wings and the wings of butterflies do not share a common ancestral structure, so they are not homologous. They are examples of convergent evolution where similar structures arise independently in unrelated species.

151. The head of some phages is icosahedral, which means that the head possesses:

A. 6 sides

B. 10 sides

C. 15 sides

D. 20 sides

Explanation: An icosahedron is a geometric shape that has 20 equilateral triangle faces, therefore 20 sides.

152. In sickle cell hemoglobin molecule, glutamic acid is replaced by:

A. Proline

B. Glutamine

C. Valine

D. Glycine

Explanation: In sickle cell disease, the mutation results in the substitution of the amino acid valine for glutamic acid at the sixth position in the beta chain of hemoglobin.

153. Where do phospholipids arrange themselves in a cell?

A. Inside the nucleus

B. Inside the cytoplasm

C. In the plasma membrane

D. Inside the mitochondrial matrix

Explanation: Phospholipids form the fundamental structure of the plasma membrane. They arrange themselves in a bilayer with their hydrophilic (water-attracting) heads facing outward and their hydrophobic (water-repelling) tails facing inward.

PUNJAB MDCAT SOLVED PAST PAPER-2023

PUNJAB MDCAT SOLVED PAST PAPER-2023

For F.S.C. and Non-F.Sc. Students 

Time Allowed: 210 Minutes (3- ½ hours)

Instructions: 

i. Read the instructions on the MCQ Response Form carefully. 

ii. Choose the Single Best Answer for each question.

iii. Each correct Answer carries One Mark. There is No Negative Marking  

iv. Candidates are strictly prohibited from giving any identification mark except Roll. No. & Signature in the specified columns only. 

Biology

Q.1. Test cross is made to check the genotype of a trait. Which of the following crosses is a test cross?

(A) Unknown × At

(B) Unknown × u

(C) Unknown × AB

(D) Unknown x

Explanation: A test cross is performed to determine the genotype of an individual with a dominant phenotype. It involves crossing the individual with a homozygous recessive individual. Hence, the choice with a defined recessive allele like “At” would be appropriate.

Q.2. During sliding of actin filaments, ATP is used for?

(A) Cross bridge formation

(B) Cross bridge breaking

(C) Dragging filaments

(D) Shortening of filaments

Explanation: In the process of muscle contraction, ATP provides the energy required to detach the cross-bridge formed between actin and myosin, allowing the muscle to relax.

Q.3. The cell membrane of a muscle cell is:

(A) Sarcoplasm

(B) Sarcolemma

(C) Sarcomere

(D) Myofibrils

Explanation: The sarcolemma is the specialized cell membrane surrounding muscle fibers (cells). It plays a crucial role in muscle contraction and signal transduction.

Q.4. Irregular striations and involuntary control is related to?

(A) Cardiac muscle cells

(B) Fibroblastic cartilage cell

(C) Skeletal muscle cells

(D) Smooth muscle cells

Explanation: Smooth muscle cells are characterized by their lack of striations, unlike skeletal and cardiac muscle cells. They are found in walls of organs and are under involuntary control.

Q.5. Cartilage is a type of which tissue?

(A) Connective tissues

(B) Epithelial tissues

(C) Muscular tissues

(D) Nervous tissues

Explanation: Cartilage is a flexible connective tissue found in many areas in the bodies of humans and other animals.

Q.6. Production of Follicle-stimulating hormone (FSH) and Luteinizing Hormone (LH) from the pituitary gland of female is controlled by:

(A) Ovary

(B) Uterus

(C) Stimulated follicles

(D) Hypothalamus

Explanation: The hypothalamus produces gonadotropin-releasing hormone (GnRH), which stimulates the pituitary gland to release FSH and LH.

Q.7. The term menopause means:

(A) Start of menstruation

(B) Start of ovulation

(C) Stop of ova production

(D) Degeneration of ovaries

Explanation: Menopause refers to the natural cessation of menstrual cycles and fertility in women due to the aging of ovaries, marking the end of the reproductive phase in a woman’s life.

Q.8. The labour pains to expel fetus are due to action of oxytocin on:

(A) Myometrium

(B) Endometrium

(C) Perimetrium

(D) Ovaries

Explanation: Oxytocin induces contractions in the myometrium (muscular layer) of the uterus, which are essential for the process of childbirth.

Q.9. In human females, what is the small, pear-shaped, muscular, distensible, sac-like organ where gestation takes place?

(A) Uterus

(B) Kidney

(C) Fallopian tube

(D) Stomach

Explanation: The uterus is the female reproductive organ where the fetus develops during gestation.

Q.10. The function of hyaluronidase enzyme is:

(A) Penetration of the sperm into the ovum

(B) Entrance of the sperm into the vagina

(C) Entrance of the sperm into the fallopian tube

(D) Inactivation of sperms after their failure to fertilize ovum

Explanation: Hyaluronidase enzyme helps spermatozoa penetrate the ovum by liquefying the protective layer around it.

Q.11. All the following are disinfectants to control bacterial growth except:

(A) Hydrogen peroxide

(B) Tincture of Iodine

(C) Potassium Permanganate

(D) Mercuric chloride

Explanation: While hydrogen peroxide has antiseptic properties, it is typically not used as a primary disinfectant against bacterial growth.

Q.12. Which of the following is an example of spiral shaped bacteria?

(A) Pseudomonas

(B) Diplococcus pneumoniae

(C) Hyphomicrobium

(D) Escherichia coli

Explanation: Pseudomonas is a genus of bacteria that includes several spiral-shaped species.

Q.13. Bacterial cell have the ability to maintain its shape through:

(A) Cell wall

(B) Capsule

(C) Cell wall & slime

(D) Slime

Explanation: The cell wall provides rigidity and structural support to bacterial cells, maintaining their shape.

Q.14. Helicobacter pylori is an example of:

(A) Spiral

(B) Cocci

(C) Bacilli

(D) Pleomorphic

Explanation: Helicobacter pylori is a spiral-shaped bacterium associated with stomach ulcers.

15. The method of bringing oxygenated air into contact with a gas exchange surface is called?

(A) Photorespiration

(B) Ventilation

(C) Gas transport

(D) Respiration

Explanation: Ventilation refers to the process of moving air (or water in some organisms) in and out of the lungs, ensuring oxygenated air comes into contact with the gas exchange surfaces for the efficient exchange of oxygen and carbon dioxide.

Q.16. Smooth muscles are found in all mentioned below except.

(A) Trachea

(B) Bronchi

(C) Bronchioles

(D) Alveoli

Explanation: While smooth muscles are found in the trachea and bronchi, the primary function of the smooth muscles in the bronchioles is to regulate airflow. The question seems to have an error as smooth muscles are indeed found in bronchioles.

Q.17. The final destination of lymph is:

(A) lymph node

(B) lymphoid organs

(C) lymph capillaries

(D) subclavian vein

Explanation: The lymph, after being filtered through the lymph nodes, eventually drains into two main ducts in the thoracic region, which subsequently empty into the left and right subclavian veins, returning the fluid to the bloodstream.

Q.18. Descending aorta is bifurcated into two vessels which on further division from femoral are that supply blood to high muscles of legs:

(A) iliac vein

(B) iliac artery

(C) temporal artery

(D) sciatic artery

Explanation: The descending aorta bifurcates into the common iliac arteries. These then further divide to give rise to arteries like the femoral artery, which supplies the thigh muscles.

Q.19. Which of the following is the main point of Darwinism?

(A) Over production

(B) Variation

(C) Disuse of organ

(D) Perceived unity of life

Explanation: The central tenet of Darwinism is the concept of variation, upon which natural selection acts, leading to the evolution of species over time.

Q.20. Which of the following type of organs are supposed to be functional in ancestral species than in present day species?

(A) Vestigial organs

(B) Homologous organs

(C) Analogous organs

(D) Embryonic organs

Explanation: Vestigial organs are remnants of structures that served important functions in the organism’s ancestors but have lost their main functions over time.

Q.21. Which of the following is NOT an example of disuse of organs?

(A) Snake’s legs

(B) Shedding of milk teeth

(C) Muscle atrophy

(D) Movement of ear

Explanation: Snake’s legs are considered vestigial structures, remnants from ancestral forms that had legs.

Q.22. According to evolutionary studies, prokaryotes may have arisen around:

(A) 2.5 billion years ago

(B) 3.5 billion years ago

(C) 4.5 billion years ago

(D) 5.5 billion years ago

Explanation: Evidence from ancient rocks suggests that prokaryotic life existed on Earth approximately 3.5 billion years ago.

Q.23. The enzyme which can work on pH 7 of the medium is?

(A) Pepsin

(B) Sucrase

(C) Chymotrypsin

(D) Pancreatic lipase

Explanation: Pancreatic lipase works in the slightly alkaline pH of the small intestine, which is around pH 7.

Q.24. Which enzyme is different from others?

(A) Pepsin

(B) Trypsin

(C) Chymotrypsin

(D) Lactase

Explanation: While Pepsin, Trypsin, and Chymotrypsin are proteolytic enzymes (break down proteins), Lactase is an enzyme that breaks down the sugar lactose.

Q.25. Competitive inhibitors are ______________?

(A) Homologous to substrate

(B) Analogous to substrate

(C) Smaller than substrate

(D) Larger than substrate

Explanation: Competitive inhibitors often resemble the substrate and compete for the same active site, hence they are analogous to the substrate.

Q.26. The active sites of enzymes are composed of?

(A) Few Nucleotides

(B) Few Nucleosides

(C) Few Saccharides

(D) Few Amino Acids

Explanation: Enzymes are proteins, and their active sites are composed of specific arrangements of a few amino acids that interact with the substrate.

Q.41. NAD is an important?

(A) Enzyme

(B) Coenzyme

(C) Hormone

(D) Vitamin

Explanation: NAD (Nicotinamide adenine dinucleotide) is a coenzyme that plays a crucial role in many redox reactions in cells.

Q.42. What is true about Oxytocin?

(A) Acts on kidney tubules

(B) Stimulates JCSH

(C) Perform cellular functions

(D) Dilate milk ducts of mammary glands

Explanation: Oxytocin is a hormone that triggers the contraction of the uterus during childbirth and helps in the ejection of milk from the mammary glands.

Q.43. In a typical nerve, the action potential duration is:

(A) 1 millisecond

(B) 1 microsecond

(C) 1 second

(D) 1 minute

Explanation: A typical action potential in neurons lasts about 1 millisecond.

Q.44. During resting membrane potential condition:

(A) Outer surface of neuron is more positive

(B) Inner surface of neuron is more positive

(C) Both of these surfaces are equally positive

(D) Both of these surfaces are equally negative

Explanation: In the resting state of a neuron, the outer surface is more positive compared to the inner surface due to the distribution of ions.

Q.45. The value of active membrane potential of the neuron is:

(A) 0.01 volts

(B) 0.02 volts

(C) 0.05 volts

(D) 0.07 volts

Explanation: The typical resting membrane potential of a neuron is around -70 mV or -0.07 volts. During action potential, the membrane potential can reach around +30 mV or 0.03 volts.

Q.46. The chemical messengers that transmit action potential across the synapse in the form of chemicals are?

(A) Synaptic knob of synapse

(B) Reflex actions

(C) Neurotransmitters

(D) Acetylcholinesterase

Explanation: Neurotransmitters are the chemical substances released at the end of a nerve fiber which allow the transmission of nerve impulses across synapses.

Q.47. The pressure receptors that receive deep pressure stimulus in human body are called;

(A) Meissner Corpuscles

(B) Pacinian Corpuscles

(C) Red Blood Corpuscles

(D) White Blood Corpuscles

Explanation: Pacinian Corpuscles are specialized mechanoreceptors that can detect vibrations and deep pressure changes within the skin.

Q.48. Cells of different types can be distinguished on the basis of surface:

(A) Phospholipids

(B) Globular proteins

(C) Glycolipids and glycoproteins

(D) Cholesterol

Explanation: Glycolipids and glycoproteins are major components on the cell surface, playing a critical role in cell-to-cell recognition and interaction.

Q.49. Mitosis, a type of cell division is observed in?

(A) E.coli

(B) Nostoc

(C) Blue green algae

(D) Cyanobacteria

Explanation: Mitosis is typically observed in eukaryotic organisms. Blue green algae, though prokaryotic, is often used interchangeably with Cyanobacteria.

Q.50. Cell wall of prokaryotic cell is composed of:

(A) Carbohydrates

(B) Carbohydrates & Proteins

(C) Proteins

(D) Proteins & Lipids

Explanation: The cell wall of prokaryotic cells, especially bacteria, is primarily composed of carbohydrates, specifically peptidoglycan.

Q.51. Which structure of prokaryotic cell will play the role of mitochondrion?

(A) Spores

(B) Cyst

(C) Nucleoid

(D) Mesosomes

Explanation: In prokaryotic cells, mesosomes play a role similar to mitochondria in eukaryotic cells. They are involved in cellular respiration and the production of ATP.

Q.52. Proteins and lipids are modified into Glycoproteins & Glycolipids by which cell organelle?

(A) Golgi complex

(B) Rough Endoplasmic Reticulum

(C) Smooth Endoplasmic Reticulum

(D) Ribosomes & chromosomes

Explanation: The Golgi complex modifies proteins and lipids that have been built in the endoplasmic reticulum, converting them to glycoproteins and glycolipids.

Q.53. Cyclosis and amoeboid movements are due to:

(A) Microtubule

(B) Cilia

(C) Microfilaments

(D) Intermediate filaments

Explanation: Microfilaments, primarily composed of actin, play a major role in cellular movements like cyclosis and amoeboid movements.

Q.54. Number of major layer/layers of cell wall in gram-positive bacteria is/are:

(A) 2

(B) 1

(C) 3

(D) 4

Explanation: Gram-positive bacteria have a thick peptidoglycan layer and a plasma membrane, making it two major layers.

Q.55. Most of the monosaccharides form a ring structure when in solution. For example, ribose forms a five-cornered ring known as:

(A) Glucopyranose

(B) Ribofuranose

(C) Glyceradehyde

(D) Acetaldehyde

Explanation: Ribose, when in solution, forms a five-membered ring structure called ribofuranose.

Q.56. Which statement is true regarding both starch and cellulose?

(A) They are both polymers of glucose.

(B) They are geometric isomers of each other

(C) They can both be digested by humans.

(D) They are both used for energy storage in plants.

Explanation: Both starch and cellulose are polymers of glucose. However, they have different structures and functions.

Q.57. Which type of bond must be broken for water to vaporize? 

(A) Popular covalent bonds 

(B) Polar covalent bonds

(C) Hydrogen bonds 

(D) Covalent bonds

Explanation: When water vaporizes, the hydrogen bonds between water molecules are the ones that are broken. The covalent bonds within the water molecules themselves remain intact.

Q.58. The specific heat of vaporization of water plays an important role in the regulation … produced by…. 

(A) reduction
(B) oxidation 

(C) redox 

(D) none of above

Explanation: The specific heat of vaporization is the amount of energy needed to change one gram of a liquid substance to a gas at constant temperature and pressure. The given options seem unrelated to the context of vaporization, but “reduction” fits the best in the context of the question though it’s not an ideal fit.

Q.59. Biologically, plays key roles in maintaining the integrity of … bilayer membranes. 

(A) Hydrophilic exclusion
(B) hydrophobic exclusion
(C) heat of vaporization 

(D) Specific heat capacity

Explanation: Hydrophobic exclusion refers to the tendency of nonpolar molecules to cluster together in aqueous solutions, which is the fundamental principle for the formation of lipid bilayers in cellular membranes.

Q.60. Energy is … by the breakdown of complex molecules into simpler ones, such reactions are called catabolic reactions. 

(A) Released 

(B) Consumed 

(C) Produced 

(D) Destroyed

Explanation: Catabolic reactions are exergonic reactions, meaning they release energy. When complex molecules are broken down into simpler ones, energy stored in the bonds of the complex molecules is released.

Q.61 … ……. forms almost three-fourth of the body by weight
(A) water
 

(B) Protein

(C) Carbohydrates 

(D) Lipids

Explanation: Water makes up a significant portion of the human body, approximately 60-75%, depending on age and body composition.

Q.62. The major enzymes involved in transfer of phosphate group from ATP to Glucose is: 

(A) Isomerase 

(B) Dehydrogenase

(C) Kinase
(D) Decarboxylase

Explanation: Kinase is the enzyme responsible for transferring a phosphate group from ATP to another molecule, in this case, glucose.

Q.63. Out of 36 ATPs, how many are produced in electron transport chain? 

(A) 28

(B) 30

(C) 32

(D) 34

Explanation: While the exact number can vary slightly based on conditions and the efficiency of the process, about 32 ATP molecules are produced from one molecule of glucose through the electron transport chain.

Q.64. Which is the major event in electron transport chain? 

(A) ATP synthesis 

(B) Decarboxylation 

(C) Substitution

(D) Isomerisation

Explanation: The primary function of the electron transport chain is the production of ATP through the process called oxidative phosphorylation.

Q.65. Which of the following is used in baking? 

(A) Aerobic respiration 

(B) Anaerobic respiration 

(C) External respiration

(D) Internal respiration

Explanation: Anaerobic respiration, specifically fermentation, is used in baking. It causes the dough to rise due to the production of carbon dioxide by yeast.

Q.66. The provirus of HIV is structurally and chemically made up of? 

(A) ssRNA

(B) dsRNA 

(C) ssDNA

(D) dsDNA

Explanation: After entering a host cell, the RNA genome of HIV is reverse-transcribed into double-stranded DNA (dsDNA), which integrates into the host cell’s genome and becomes a provirus.

Q.67. The molecules used to control virus during infection of animal cells are: 

(A) Interferon 

(B) Histone

(C) Antigen

(D) Serum

Explanation: Interferons are proteins produced by animal cells in response to viral infections and play a role in the immune response to inhibit the replication of viruses.

Q.68. Virus is what type of agent? 

(A) Cellular agent 

(B) Infectious agent 

(C) Non-infectious agent 

(D) Non-protein agent

Explanation: Viruses are infectious agents that can replicate only inside the living cells of an organism. They can infect all forms of life.

CHEMISTRY

Q.1. What are vinyl alcohol and acetaldehyde? 

(A) Tautomers 

(B) Position isomers

(C) Chain isomers 

(D) Metamers

Explanation: Vinyl alcohol and acetaldehyde are tautomers, which means they can rapidly interconvert by the movement of a proton.

Q.2. Oxidation number of free magnesium is?

(A) +2 

(B) 0 

(C) +1

(D) +3

Explanation: Magnesium has an oxidation state of +2 in its compounds because it loses two electrons to achieve a stable electronic configuration.

Q.3. Select the metal which is extracted from bauxite?

(A) Al 

(B) Ca

(C) Mg

  (D) Cu

Explanation: Bauxite is the principal ore for the extraction of aluminum (Al).

Q.4. Which of the following has the highest atomic radius in its period? 

(A) Alkali metals

(B) Alkaline earth metals 

(C) Chalcogens
(D) Halogens

Explanation: Among the given options, alkali metals have the largest atomic radii in their respective periods.

Q.5. The electronic configurations of some elements are given below. Recognize the element ______ belongs to group IIIA.
(A) 1s2 2s2 2p3 

(B) 1s2 2s2 2p4

(C) 1s2 2s2 2p1

(D) 1s2 2s2 2p2

Explanation: Elements with an electronic configuration ending in p1 belong to group IIIA.

Q.6. All alkali metals react with chlorine gas to form white metal chlorides salt. The ……………… salt formed is: 

  (A) Insoluble

(B) Soluble in water to give neutral solution of pH 7 

(C) Soluble in water to give acidic solution pH 1

(D) Soluble in water to give alkaline

Explanation: Alkali metal chlorides are soluble in water and give an alkaline solution when dissolved.

Q.7. Which one of them is amphoteric in nature?
(A) Lithium oxide

(B) Beryllium oxide

(C) Calcium oxide

(D) Potassium oxide

Explanation: Beryllium oxide (BeO) is amphoteric, which means it can act as both an acid and a base.

Q.8. Which of the following is the correct electronic configuration of iron (II) ion (atomic number = 26)? 

(A) [Ar] 4s0, 3d6

(B) [Ar] 4s2, 3d6 

(C) [Ar] 4s2, 3d4 

(D) [Ar] 4s2, 3d5

Explanation: In the iron (II) ion, two electrons are removed, primarily from the 4s orbital, leading to the configuration [Ar] 4s0, 3d6.

Q.9. Which of the functional groups are present in ethyl acetate? 

(A) Aldehyde group

(B) Carboxyl group 

(C) Ester group

(D) Ether group

Explanation: Ethyl acetate has the formula CH3COOCH2CH3, which represents an ester functional group.

Q.10. What is the molecular formula of pyridine molecule?

(A) C6H5N 

(B) C5H5N

(C) C5H5NH 

(D) C6H6N

Explanation: Pyridine has the molecular formula C5H5N. It is a basic heterocyclic organic compound similar to benzene but with one CH group replaced by nitrogen.

Q.11. The addition reaction of 2-butene with HBr produces?

(A) 1-bromobutane

(B) 2-bromobutane

(C) 1.3 dibromobuane

(D) 2.3 dibromobutane

Explanation: The addition of HBr to 2-butene follows Markovnikov’s rule, wherein the bromine atom attaches to the more substituted carbon. Thus, the product is 2-bromobutane.

Q.12. A compound of phosphorus oxide has 43.6% of Oxygen. Its empirical formula is?

(A) P2O5

(B) P2O3

(C) P1O2

(D) PO2

Explanation: Based on the given percentage composition and atomic weights, we deduce the ratio of phosphorus to oxygen to arrive at the empirical formula P2O3.

Q.13. Which element is used as standard to determine atomic mass of an element?

(A) H

(B) C

(C) P

(D) Cl

Explanation: Carbon-12 isotope is the standard used to define atomic mass units, making carbon the reference element for atomic weight determination.

Q.14. The average weight of atoms of an element compared to the weight of one atom of _______ is called atomic weight.

(A) Carbon

(B) Helium

(C) Hydrogen

(D) Nitrogen

Explanation: The atomic weight is traditionally defined as the average relative weight of an atom compared to one atom of hydrogen, which was considered as the standard.

Q.15. Which of the following electronic configuration is correct for 24Cr?

(A) 1s2,2s2,2p5,3s2,3p6,4s1,3d5

(B) 1s2,2s2,2p6,3s2,3p6,4s2,3d4

(C) 1s2,2s2,2p5,3s2,3p6,4s1,3d6

(D) 1s2,2s2,2p5,3s2,3p6,4s1,3d5

Explanation: The correct electronic configuration for Chromium (24Cr) is [Ar] 4s2 3d4. This configuration reflects its placement on the periodic table and its atomic number.

Q.16. The same moles of H2, N2 and O2 are present in 0.1 cc volume at STP. Which one has the greatest number of molecules:

(A) N2

(B) H2

(C) O2

(D) Number of molecules are equal

Explanation: If the moles are the same, the number of molecules will be the same for all gases, given Avogadro’s law.

Q.17. Which is not a property of liquid?

(A) Osmotic pressure

(B) Freezing point

(C) Diffusion

(D) Melting

Explanation: Osmotic pressure is a property associated with solutions, particularly in semi-permeable membranes, and not a standalone property of pure liquids.

Q.18. Which type of forces exist between iodine molecules?

(A) Dipole-dipole forces

(B) Dipole-induced dipole forces

(C) Instantaneous dipole-induced dipole forces

(D) Non-polar forces

Explanation: Iodine molecules are non-polar, and the forces existing between them are Van der Waals forces, also known as instantaneous dipole-induced dipole forces.

Q. 19. In anisotropic crystals which property does not change with the change in direction of crystalline structure?

(A) Molar mass

(B) Electrical conductance

(C) Thermal conductance

(D) Refractive index

Explanation: The molar mass of a substance remains constant irrespective of the direction of the crystalline structure or any other factors.

Q. 20. In metals, why does electric conductivity decrease with the increase of temperature?

(A) Because metal ions oscillations hinder electron movement

(B) Because electron movement decreases with the increase of temperature

(C) Because electron direction changes at high temperature

(D) Because they pass heat current through collision of electrons.

Explanation: At higher temperatures, the vibrations of metal ions increase. These oscillations interfere with the smooth flow of electrons, reducing conductivity.

Q.21. Why does fluorine have less electron affinity as compared to bromine?

(A) Thick small electronic cloud

(B) Electronegativity

(C) Seven electrons in

(D) Higher

Explanation: Fluorine’s small size and dense electron cloud result in increased electron repulsions, leading to lower electron affinity compared to bromine.

Q.22. Which oxyacid of halogen is a strong oxidizing agent?

(A) HClO4

(B) HClO3

(C) HClO

(D) HClO2

Explanation: Perchloric acid (HClO4) is a strong oxidizing agent due to its capability to donate oxygen.

Q.23. Which of the following transition metal shows a 3d5 configuration in its +2 oxidation state?

(A) Fe + 2

(B) Cu + 2

(C) Mn + 2

(D) Zn + 2

Explanation: Iron (Fe) in its +2 oxidation state (Fe^2+) has a 3d6 configuration.

Q.24. Which of the following reagents can be used to distinguish between 1-pentyne and 2-pentyne?

(A) 1% alkaline dilute KMNO4

(B) AgNO3 + NH4OH

(C) Br2/CCl4

(D) K2Cr2O7 + H2SO4

Explanation: Alkaline KMnO4 can oxidize terminal alkynes (like 1-pentyne) but not internal alkynes (like 2-pentyne).

Q.25. In SN1 reactions, the correct order of reactivity of alkyl halide is?

(A) Tertiary > secondary > primary

(B) Primary > secondary > tertiary

(C) Secondary > primary > tertiary

(D) Primary > tertiary > secondary

Explanation: In SN1 reactions, the rate-determining step involves the formation of a carbocation. Tertiary carbocations are the most stable, followed by secondary and primary.

Q.26. What is the IUPAC name of diisopropyl ketone?

(A) 2,4-Dimethylpentan-3-one

(B) 1.3-Diisopropylpropan-2-one

(C) 2,4-Dimethylpentan-2-one

(D) 1,3-Dimethylpropan-2-one

Explanation: Diisopropyl ketone has the structure CH3-C(CH3)2-CO-C(CH3)2-CH3, which corresponds to the IUPAC name 2,4-Dimethylpentan-3-one.

Q. 27 The appearance of a silver mirror in Tollens’ test indicates the presence of which of the following?

(A) An aldehyde

(B) A ketone

(C) An acid

(D) An alcohol

Explanation: The Tollens’ test is used to distinguish between aldehydes and ketones. Aldehydes react with Tollens’ reagent to form a shiny silver mirror, indicating the presence of an aldehyde.

Q.28. Competitive inhibitors stop an enzyme from working by:

(A) Blocking the active site of the enzyme

(B) Changing the shape of the enzyme

(C) Merging with the substrate instead

(D) Combining with the product of the reaction

Explanation: Competitive inhibitors bind to the active site of the enzyme, preventing the substrate from binding, hence inhibiting the enzyme’s activity.

Q.29. If phenol is treated with 3 moles of conc. HNO3 in the presence of H2SO4 what will be the product?

(A) picric acid

(B) o-nitro phenol

(C) p-nitro phenol

(D) o-nitro phenol and p-nitro phenol

Explanation: Treating phenol with 3 moles of concentrated HNO3 in the presence of H2SO4 results in trinitration, producing picric acid (2,4,6-trinitrophenol).

Q.30. When 6d orbital is filled, the entering electron goes into?

(A) 7p

(B) 7f

(C) 7d

(D) 7s

Explanation: After the 6d orbitals are filled, electrons will enter the 7p orbitals based on the aufbau principle.

Q.31. Which element has the ground state electronic configuration of 1s2, 2s2, 2p6, 3s2, 3p6?

(A) Ar

(B) Cl

(C) Na

(D) S

Explanation: The given configuration (1s2, 2s2, 2p6, 3s2, 3p6) corresponds to the noble gas Argon (Ar) with an atomic number of 18.

Q.32. What is the proton (atomic number) of an element that has four unpaired electrons in its ground state?

(A) 22

(B) 6

(C) 14

(D) 26

Explanation: Titanium (Ti) with an atomic number of 22 has the ground state configuration [Ar] 4s2 3d2, which means it has 2 unpaired electrons in its d orbitals.

Q.33. Which one of the following gases has the lowest density under room conditions?

(A) Neon

(B) Nitrogen

(C) Oxygen

(D) Fluorine

Explanation: Among the given options, Neon (Ne) is a noble gas and has the lowest atomic number, making it the lightest and thus having the lowest density.

Q.34. The process of heat flow between hotter and colder gases remains continued until all the molecules have equal.

(A) Average translational kinetic energy

(B) Average rotational kinetic energy

(C) Average translational potential energy

(D) Average vibrational kinetic energy

Explanation: Heat flow in gases primarily affects the translational motion of the molecules. When they achieve thermal equilibrium, their average translational kinetic energy becomes equal.

Q.35. What is the ultimate fate of reversible reaction?

(A) A state when there is no net concentration change.

(B) Completion of reaction

(C) Complete consumption of reactants

(D) Complete consumption of products

Explanation: In a reversible reaction, equilibrium is reached when the forward reaction rate equals the reverse reaction rate, leading to no net change in reactant or product concentrations.

Q.36. In a reversible reaction, when a product is removed, the equilibrium will shift towards the:

(A) Product side

(B) Reactant side

(C) Both sides one by one

(D) No effect

Explanation: According to Le Chatelier’s principle, removing a product will cause the equilibrium to shift to the right, towards the product side, to produce more of it.

Q.37. The rate of reaction between two specific time intervals is called?

(A) Average rate of reaction

(B) Instantaneous rate of reaction

(C) Rate of reaction

(D) Initial rate

Explanation: The average rate of reaction is determined by evaluating the change in concentration of a reactant or product over a specific time interval.

Q.38. How will the rate of reaction be if the slope of the curve is steeper near the start of the reaction?

(A) Greater

(B) Constant

(C) Equilibrium

(D) Lesser

Explanation: A steeper slope at the reaction curve’s start indicates a faster initial reaction rate.

Q.39. What causes energy changes in chemical reactions?

(A) Bond formation and breakage

(B) Bond formation

(C) Bond breakage

(D) Ionic bonds

Explanation: Energy changes during chemical reactions are due to the breaking of old bonds (which consumes energy) and the formation of new bonds (which releases energy).

Q.40. What is the unit of heat capacity?

(A) JK – 1

(B) kJmol – 1

(C) kJ/K

(D) kJ.mol

Explanation: Heat capacity is defined as the amount of heat required to increase a substance’s temperature by one degree Kelvin, so its unit is Joules per Kelvin (JK^(-1)).

PHYSICS

Q.1. Which of the following the fractional change in resistance per kelvin?

(A) Temperature coefficient of resistivity

(B) Conductivity

(C) Resistivity

(D) Temperature coefficient of resistance

Explanation: The fractional change in resistance per Kelvin is termed the temperature coefficient of resistivity. It quantifies how the electrical resistivity of a metal changes with temperature.

Q.2. According to maximum power transfer theorem, which of the following is the ………… delivered by the battery to the output?

(A) E2/4r

(B) E2/2r

(C) E2/5r

(D) E2/3r

Explanation: According to the maximum power transfer theorem, for maximum power to be delivered to the output, the load resistance should be equal to the internal resistance of the source. The power in such a case is given by E^2/4r.

Q.3. Velocity selector will select only those charge particles whose velocity is given by:

(A) V = E/B

(B) V = B/E

(C) V = E × B

(D) V = 0

Explanation: For a velocity selector using perpendicular electric and magnetic fields, the velocity of particles unaffected by these fields is given by V = E/B.

Q.4. The induced emf produced in the coil is sometimes called as?

(A) Back emf

(B) Self-inductance

(C) Motional emf

(D) Mutual inductance

Explanation: Back emf refers to the emf induced in a coil/motor when it is running, opposing the applied voltage. It’s a result of Lenz’s law.

Q.5. The quantity t has the same units as that of?

(A) Current

(B) Magnetic induction

(C) Charge

(D) Emf

Explanation: The change in magnetic flux with time (∆Φ/∆t) has units of emf or voltage.

Q.6. Working principal of magnetic levitation train is according to?

(A) Lenz law

(B) Faraday law

(C) Max planks law

(D) Ohm law

Explanation: Magnetic levitation trains, or maglev trains, work on the principle of Lenz’s law. The train levitates above the tracks due to the repulsion between like magnetic fields.

Q.7. The expression for the emf produced by A. C. generator is?

(A) NAB Cos

(B) NAB Sin

(C) NAB

(D) ILB Cos

Explanation: The emf produced by an AC generator is e = NABω sin(ωt). When taking into account the angle, it becomes e = NABω cos(θ).

Q.8. For the positive half cycle i.e., 0 – T/2, the diode D:

(A) Is forward biased

(B) Is reverse biased

(C) Behaves as open

(D) Show maximum resistance

Explanation: During the positive half-cycle of AC input, the diode gets forward biased and allows current to flow through.

Q.9. The process of ejection of loosely bound electrons from a certain photo sensitive surface absorption of photon is called:

(A) Photoelectric effect

(B)  Compton effect

(C) Pair production

(D) Black body radiation

Explanation: The photoelectric effect refers to the phenomenon where electrons are ejected from the surface of a material (typically metals) when light of sufficient energy strikes it.

Q.10. In Compton effect, a photon of a certain wavelength collides with a stationary electron wavelength of the emitted photon is:

(A) Longer

(B) Shorter

(C) Same

(D) Infinite

Explanation: The Compton effect or Compton scattering is the phenomenon where x-ray or gamma ray photons are deflected by electrons, leading to a shift (usually an increase) in the photon’s wavelength.

Q.11. The Balmer series of hydrogen is important because it:

(A) Is in the visible region

(B) Is the only one for which the quantum theory can be used

(C) Is the only series that occurs for hydrogen

(D) Involves the lowest possible quant number n

Explanation: The Balmer series corresponds to transitions in the hydrogen atom that result in the emission of photons in the visible light spectrum.

Q.12. The SI unit of equivalent dose is:

(A) Sievert

(B) Gray

(C) Mass

(D) Rad

Explanation: Sievert (Sv) is the SI unit that measures the biological effect of ionizing radiation.

Q.13. The decay rate of radioactive substance is:

(A) Decreases exponentially with time

(B) Constant with time

(C) Varies inversely with time

(D) Decreases linearly with time

Explanation: The decay of a radioactive substance is described by an exponential function, implying that the rate of decay decreases over time.

Q.14. The relation between gray and rad is given as: 1 Gy = ______ rad.

(A) 100

(B) 0.01

(C) 0.001

(D) 10

Explanation: 1 Gray (Gy) is equal to 100 rad. Both are units of absorbed dose of ionizing radiation.

Q.15. In the displacement-time graph, if the slope is constant then the velocity is:

(A) Constant

(B) Variable

(C) May be variable or constant

(D) Infinite

Explanation: In a displacement-time graph, a constant slope represents constant velocity.

Q.16. The circular line has a fix distance from ________?

(A) A fix point

(B) Any point

(C) A point on a circle

(D) A point from outside

Explanation: A circle is defined as a set of points that are equidistant from a fixed center point.

Q.17. If a particle is moving with uniform circular motion, then:

(A) Velocity and acceleration are perpendiculars

(B) Velocity and acceleration are antiparallel

(C) Velocity and acceleration are parallel

(D) Zero acceleration

Explanation: In uniform circular motion, while velocity is tangential to the circle, acceleration (centripetal acceleration) is directed towards the center of the circle.

Q.18. One degree is equal to:

(A) n/180 radians.

(B) n/90 radians.

(C) n/270 radians.

(D) n/360 radians.

Explanation: There are 22π (or 360360 ) radians in a full circle. Hence, 11 is equivalent to / 180π/180 or /180n/180 radians.

Q.19. An observer standing near the sea shore observes 54 waves per minute. If the wavelength of the water wave is 10m then the velocity of water wave is:

(A) 9 m/s

(B) 540 m/s

(C) 5.4 m/s

(D) none

Explanation: Wave speed = frequency × wavelength. Using the given values: (54/60) × 10 = 9 m/s.

Q.20. If the length of second pendulum becomes four times, then its time period will become:

(A) Four times

(B) Two times

(C) Half

(D) One fourth

Explanation: Time period of a pendulum is proportional to the square root of its length. If length becomes four times, time period becomes twice.

Q.21. First law of thermodynamics concerns with the conservation of

(A) Heat

(B) Work

(C) Momentum

(D) Energy

Explanation: The first law of thermodynamics, also known as the Law of Energy Conservation, states that energy cannot be created or destroyed in an isolated system.

Q.22. An ideal gas has molar specific heat Cp at constant pressure. When the temperature of n moles is increased by T the increase in the internal energy is:

(C) n(Cp – R) T

(A) nCp T

(B) n(Cp + R) T

(D) n(2Cp + R) T

Q.23. The ohmmeter of a portable digital multimeter needs:

(A) Internal battery

(B) Wet cell

(C) Voltmeter

(D) Ammeter

Explanation: An ohmmeter needs an internal battery to provide a voltage to measure the resistance.

Q.24. A wire of uniform area of cross-section ‘A’, length ‘L’, and resistance ‘R’ is cut into several parts. What will happen to the resistivity of each part?

(A) It will be doubled

(B) It will be one fourth

(C) It will be halved

(D) It will remain the same

Explanation: Resistivity is a material property and does not change with dimensions.

Q.25. If a 0.5 T field is applied over an area of 2-meter square which lies at an angle of 60° with the field, then the resulting flux will be:

(A) 0.5 T

(B) 0.5 Wb

(C) 0.25 Wb

(D) 0.25 T

Explanation: Magnetic flux = B × A × cos(θ). Using given values: 0.5 × 2 × cos(60°) = 0.5 Wb.

Q.26. The magnitude of magnetic force will be maximum on current-carrying conductor in a magnetic field if the conductor is placed?

(A) Parallel to magnetic field

(B) At 45-degree in magnetic field

(C) Perpendicular to magnetic field

(D) Antiparallel in magnetic field

Explanation: The force on a current-carrying wire in a magnetic field is given by F = I × L × B × sin(θ), where θ is the angle between the current direction and the magnetic field. This force is maximum when θ = 90°, meaning the wire is perpendicular to the magnetic field.

Q.27. In principle, the transformer consists of two coils of copper, electrically insulated from each other, wound on the same?

(A) Iron core.

(B) Copper core

(C) Gold core

(D) Steel core

Explanation: The purpose of the iron core in a transformer is to provide a path for the magnetic flux, ensuring efficient transfer of energy from the primary coil to the secondary coil.

Q.28 In Compton Effect, the incident photon when compared to the scattered photon is of:

(A) Greater energy

(B) Greater frequency

(C) Greater energy and momentum

(D) Equal energy

Explanation: In the Compton Effect, the scattered photon always has less energy (and thus a longer wavelength) than the incident photon because some of the energy of the incident photon is transferred to the recoiling electron.

Q.29. Which of the following is the longest wavelength of radiation for the Paschen series?

(A) 187000000 m

(B) 187000000 / m

(C) 0.00000187 m

(D) 0.00000187 / m

Explanation: Paschen series refers to the spectral lines resulting from electronic transitions in hydrogen from higher energy levels to n=3. Given the values, 0.00000187 m is a wavelength, making it the right answer.

Q.30 Which of the following is the correct definition of variable velocity?

(A) Unequal distances are covered in equal intervals of time

(B) Equal displacements are made in unequal intervals of time

(C) Unequal displacements are made in equal intervals of time

(D) Equal displacements are made in equal intervals of time

Explanation: Variable velocity means that the rate of change of displacement with respect to time (velocity) is not constant. This would occur if unequal distances are covered in equal intervals of time.

Q.31 The velocity-time plot for a moving particle shows a straight line. This means:

(A) The particle has a constant acceleration

(B) The particle has never turned around

(C) The particle has zero displacement

(D) The data is insufficient

Explanation: A straight line on a velocity-time graph indicates that the rate of change of velocity (acceleration) is constant.

Q.32. A man is in a car that is moving with the velocity of 36km/hr. His speed with respect to the car is:

(A) 10m/s

(B) 36m/s

(C) Zero

(D) Infinite

Explanation: Relative to the car he’s in, the man’s speed is zero.

Q.33. On which car there is resultant force?

(A) Car moving on a straight horizontal road

(B) Car moving at constant speed around a bend

(C) Car moving uphill at a constant speed

(D) Car that is stationary

Explanation: Even if a car is moving at a constant speed around a bend, it’s changing direction, which means there’s a centripetal force acting on it.

Q.34. If two equal masses are in motion with same individual speeds, we can conclude that:

(A) Their momentums are same

(B) Their momentums can be different from each other

(C) Their kinetic energies are different from each other.

(D) Their total energies are same

Explanation: Momentum is the product of mass and velocity. If mass and speed are the same for both, then their momentums will also be the same.

Q.35. The work done by a variable force can be found by dividing the:

(A) Force into small intervals

(C) Both force and displacement into small intervals

(B) The displacement into small intervals

(D) By taking displacements at different angles

Explanation: By dividing both force and displacement into small intervals, we can integrate to find the total work done.

Q.36. Two bodies with kinetic energies in the ratio of 4:1 are moving with equal linear momentum. The ratio of their masses is:

(A) 1 : 2

(B) 1 : 1

(C) 4 : 1

(D) 1 : 4

Explanation: Since kinetic energy (K.E) is proportional to 2 mv 2 and momentum (p) is proportional to mv, a fourfold increase in K.E with the same momentum means the faster body is 1/4th the mass of the slower one.

Q.37. A fisherman lifts a fish of mass 250g from rest through a vertical height of 1.8m. The fish gains a speed of 1.1 m/s. What is the energy gained by the fish?

(A) 0.15 J

(B) 4.3 J

(C) 4.4 J

(D) 4.6 J

Explanation: The energy gained by the fish is a combination of its potential energy (mgh) and kinetic energy 122)( 21mv 2). Adding these together gives the total energy gained. Plugging in the values, we get: 

0.25×9.8×1.8+0.5 ×0.25×1.12=4.30.25×9.8×1.8+0.5×0.25×1.1 2=4.3 J.

Q. 38. One radian is analogous to:

(A) 57o3′

(B) 57o3″

(C) 57o18′

(D) 57o18″

Explanation: 1 radian is approximately equal to 180π180  degrees, which is roughly 57.2958 degrees or 57o18′.

Q. 39. Wave trough refers to the:

(A) Wave length

(B) Wave speed

(C) Highest point of the wave

(D) Lowest point of the wave

Explanation: The trough is the opposite of the crest, meaning it’s the lowest point on a wave.

Q.40. When a wave goes from one medium to another, there is no change in the:

(A) Frequency

(B) Amplitude

(C) Wavelength

(D) Velocity

Explanation: As waves move from one medium to another, their speed and wavelength change, but their frequency remains constant.

Q.41. If transverse waves are passing through medium, then particles of the medium:

(A) Remain stationary

(B) Move away

(C) Move toward

(D) Move In Simple Harmonic Motion

Explanation: In a transverse wave, particles of the medium move perpendicular to the direction of the wave, exhibiting a simple harmonic motion.

Q.42. The speed of sound in an ideal gas depends upon:

(A) Temperature and amplitude

(B) Frequency and fog

(C) Temperature and density

(D) Density and amplitude

Explanation: The speed of sound in a gas is dependent on the temperature and density of the gas.

Q.43. What will be the frequency of this light when detected on Earth?

(A) 2.04 × 10^13Hz

(B) 4.37 × 10^14Hz

(C) 4.57 × 10^14Hz

(D) 4.70 × 10^14Hz

Explanation: The Doppler effect for light can be calculated using the formula. Since the star is receding, the frequency decreases.

Q.44. As per Coulomb’s law, the force of attraction or repulsion between two-point charges is directly proportional to the:

(A) Sum of the magnitude of charges

(B) Square of the distance between them

(C) Product of the magnitude of charges

(D) Cube of the distance

Explanation: Coulomb’s law states that the electric force between two charges is directly proportional to the product of their charges and inversely proportional to the square of the distance between them.

Q. 45 Electric field intensity inside a hollow charged sphere is:

(A) Maximum

(B) Zero

(C) Negative

(D) Positive

Explanation: Inside a hollow charged sphere, the electric field is always zero due to the symmetry of charge distribution.

Q.46. Gauss law cannot be used to find which of the following quantity?

(A) Electric field intensity

(B) Electric flux density

(C) Charge

(D) Permittivity

Explanation: Gauss’s law relates the electric flux through a closed surface to the charge enclosed by that surface. Permittivity is a constant and is not directly determined by Gauss’s law.

Q.47. A charged particle moves in a uniform electric field between two oppositely charged parallel metal plates. To calculate the force acting on the particle due to the electric field, which quantity is not required?

(A) Particle speed

(B) Particle charge

(C) Plate separation

(D) Potential difference between the plates

Explanation: Force on a charged particle in an electric field is given by F=qE where q is the charge and E is the electric field intensity. The speed of the particle doesn’t affect this force.

Q.48. What is the potential difference between two points in an electric field if it takes 600J energy to move a charge of 2 C between these two points?

(A) 300 J

(B) 1200 J

(C) 800 J

(D) 0 J

Explanation: The potential difference (V) between two points is defined as the work done (W) per unit charge (Q) moved between the points, V = W/Q. Thus, V = 600J/2C = 300J (or Volts, since a Joule per Coulomb is a Volt).

Q.49. To store the electric charge the ultra-capacitors use _________ effect.

(A) Double layer

(B) Single layer

(C) Triple layer

(D) Quadruple layer

Explanation: Ultra-capacitors, also known as supercapacitors, store electric charge using the electric double layer effect.

Q.50. A capacitor of capacitance ‘C’ has a charge ‘Q’ and stored energy is ‘w’. If the charge increased to ‘2Q’. The stored energy will be:

(A) 4w

(B) 2w

(C) W/4

(D) W/2

Explanation: The energy (w) stored in a capacitor is given by =122w= 21CV 2 , where V is the potential difference. If Q is doubled, then V is also doubled (since Q = CV). Substituting this into the equation for w gives an energy of 4w.

English

Q.1. ________ methods don’t work. Choose the correct option:

(A) This

(B) These

(C) The

(D) That

Explanation: “Methods” is plural, so the correct demonstrative adjective is “these”.

Q.2. _______________ a game of tennis.

(A) Has

(B) Had

(C) Have

(D) Have had

Explanation: When referring to a non-specific time or experience in the past, “have” (followed by a past participle verb, implied here) is the correct auxiliary verb.

Q.3. The pen is expensive; still I _______ it.

(A) Bought

(B) Will buy

(C) Have bought

(D) buyed

Explanation: The context suggests a past action that’s already completed. The correct past tense of “buy” is “bought”.

Q.4. Punctuate the given sentence correctly.

(A) My father is Chairman of the Committee on Internal Relations. He also heads the Discipline Committee.

(B) My father is Chairman of the Committee on Internal Relations-he also heads the discipline Committee.

(C) My father is Chairman of the Committee on Internal Relations: he also heads the Discipline committee.

(D) My father is Chairman of the Committee on Internal Relations, but he also heads the Discipline Committee.

Explanation: The sentence is presenting two distinct statements. Separating them with a period provides clarity.

Q.5. Which of the following sentences is correct?

(A) I want to live near my parents live.

(B) I want to live where my parents live.

(C) I want to live where, my parents live.

(D) I want to live where: my parents live.

Explanation: “Where” is the correct relative adverb to use to refer to a place or location.

Q.6. Choose the sentence that is grammatically correct:

(A) He weighed himself two maunds.

(B) He weighed two maunds.

(C) He weighed themselves two maunds.

(D) He weighed itself two maunds.

Explanation: The verb “weighed” is used here to denote a measurement. Thus, the correct sentence structure is “He weighed two maunds.”

Q.7. Choose the correct option.

(A) We felt as if the ground was slipping beneath our feet.

(B) Digging

(C) Slipping

(D) Bursting

Explanation: The correct idiomatic expression is “the ground was slipping beneath our feet,” which denotes a feeling of instability.

Q.8. A few years ago, it ___________ that human beings do not belong to this earth.

(A) Found

(B) Was found

(C) Were found

(D) Had found

Explanation: The passive voice “was found” is appropriate here because the focus is on the discovery, not who made the discovery.

Q.9. A shoal of fish __________ killed by the fishermen.

(A) has been

(B) have

(C) had

(D) has

Explanation: “Shoal of fish” is a singular collective noun. The correct form to use in the passive voice with a singular subject is “has been.”

Q.10. At last the fly was ___________.

(A) Trapped by the spider

(B) Entertained by the spider

(C) Invited by the spider

(D) Spared by the spider

Explanation: Given the natural behavior between a spider and a fly, “trapped” is the most plausible action.

Q.11. All children are silly people. Some silly people are rich people. All rich people ———– if the following conclusions are NECESSARILY TRUE?

I. Some silly people are children.

II. Some rich people are children.

III. Some silly people are big shots.

(A) I and III

(B) II

(C) II and III

(D) I and II

Explanation:

I is true as it’s given that “All children are silly people.”

II is potentially true because some of the silly people, who are also children, could be rich.

III is not necessarily true based on the given information.

QUETTA MDCAT SOLVED PAST PAPER-2023

QUETTA MDCAT SOLVED PAST PAPER-2023

For F.S.C. and Non-F.Sc. Students 

Time Allowed: 210 Minutes (3- ½ hours)

Instructions: 

i. Read the instructions on the MCQ Response Form carefully. 

ii. Choose the Single Best Answer for each question.

iii. Each correct Answer carries One Mark. There is No Negative Marking  

iv. Candidates are strictly prohibited from giving any identification mark except Roll. No. & Signature in the specified columns only. 

1. Which of the following is not true about canal rays?

(A) 1.6726 x 10^-27 kg

(B) 9.54 x 10^7 C/kg

(C) Show deflection under electric & magnetic fields

(D) Do not cause mechanical motion

Explanation: Canal rays, or positive rays, are composed of positive ions. They show deflection under electric and magnetic fields based on their charge and are directly related to the e/m ratio. However, they do not cause mechanical motion.

2. The solubility of carboxylic acid in water gradually decreases with:

(A) Decreases in molecular mass

(B) Increases in molecular mass

(C) Increases the amount of acid

(D) Decreases the amount of water

Explanation: As the molecular mass of carboxylic acid increases, its non-polar hydrocarbon part becomes dominant, making the molecule less polar and reducing its solubility in water.

3. The polar part of the soap and detergents dissolve in water molecules due to:

(A) Dipole–dipole forces

(B) Dipole–induced dipole forces

(C) Hydrogen bonding

(D) London Dispersion forces

Explanation: Soaps and detergents have polar parts that can form hydrogen bonds with water, making them soluble.

4. When equal moles of reactants A and B are allowed to react according to the following balanced equation (2 A + B product), the limiting reactant in this chemical equation will be:

(A) Reactant A

(B) Reactant B

(C) Reactant A and B

(D) No Limiting reactant

Explanation: For every mole of B, you need 2 moles of A. If you start with equal moles, A will run out first.

5. The hydrocarbon which has isolated rings and is aromatic?

(A) Phenanthrene(B) Triphenylmethane

(C) Diphenylether(D) Anthracene

Explanation: Anthracene has three fused benzene rings, which make it aromatic, and the rings are isolated from each other.

6. Which functional group is characteristic of an ester?

(A) R-O-R(B) R-CO-R

(C) R-CO-OR (D) R-CO-OH

Explanation: Esters have the RCOOR’ functional group where R and R’ are alkyl or aryl groups.

7. For each of the following, choose the sentence with the correct semicolon placement:

(A) The pen was expensive, still I bought it.

(B) The pen was expensive still, I bought it.

(C) The pen; was expensive still I bought it.

(D) The pen was expensive; still, I bought it.

Explanation: A semicolon is used to link two independent clauses that are closely related in thought.

Choose a synonym for the word in capital letters:

8. The VICISSITUDES of life are unexpected, nothing remains the same.

(A) Changes (B) Evils

(C) Mistakes(D) Rules

Explanation: Vicissitudes refer to the changes or variations that happen unexpectedly, especially in one’s life.

Choose the correct option:

9. You will have to catch the morning flight, so you ______ better get ready.

(A) Would(B) May

(C) Should (D) Had

Explanation:  The phrase “you better” is commonly followed by “should” to give advice or suggest an action.

10. The most appropriate antonym for ‘abridge’ is:

(A) Condense(B) Lengthen

(C) Shorten(D) Reduce

Explanation: “Abridge” means to shorten or condense. The opposite of this action would be to “lengthen.”

11. Which of the following is correct?

(A) Forty years seems a long time.

(B) Forty years seem long time.

(C) Forty years seems long time.

(D) Forty years seem a long time.

Explanation: “Forty years” as a duration (despite being plural in form) is treated as singular when referring to a period, hence “seems” is the correct verb form.

12. A boy is waiting for you for a long time. In this sentence, the word ‘boy’ is?

(A) Proper noun(B) Common noun

(C) Abstract noun(D) Collective noun

Explanation: A “boy” is a general noun that can refer to any male child, hence it’s a common noun.

13. I expect you ________ from college by this time next year and will be looking for a job. Fill in the blank with the most appropriate choice:

(A) Will graduate(B) Will be graduate

(C) will graduated (D) will have graduated

Explanation: Given the future expectation, the perfect future tense “will have graduated” is the most appropriate.

14. Air at sea level is dense. This is a practical application of:

(A) Boyle’s law (B) Charles law

(C) Avogadro’s law(D) Dalton’s law

Explanation: Boyle’s law relates the pressure and volume of a gas, and at sea level where pressure is highest, air is denser.

15. Which of the following acid can show cis-trans isomerism?

(A) Malonic acid(B) Maleic acid

(C) Succinic acid(D) Lactic acid

Explanation: Maleic acid has a cis configuration which allows it to show cis-trans isomerism.

16. What is the reactivity state of Phenols?

(A) Less reactive (B) More reactive

(C) Neutral(D) Nonreactive

Explanation: Phenols are less reactive than alcohols due to resonance stabilization.

17. Why do electrons have opposite spins when they are in the same orbital?

(A) The condition reduces friction

(B) This condition provides more energy

(C) This condition results in zero magnetism and removes the charge of the electron

(D) This condition results in less repulsion and opposite magnetic fields

Explanation: Electrons with opposite spins in the same orbital have a paired spin state, which minimizes repulsion between them.

18. Which of the following is not an endothermic reaction?

(A) Combustion of methane

(B) Decomposition of water

(C) Dehydrogenation of ethane or ethylene

(D) Conversion of graphite to diamond

Explanation: Combustion reactions are exothermic as they release energy. Methane, being the primary component of natural gas, is widely used as a fuel for heating, cooking, and electricity generation. Its combustion is a significant source of energy in many parts of the world. However, the release of carbon dioxide during this process contributes to greenhouse gas emissions and global warming.

19. Which order of reaction obeys the expression? 

1/2t 1/2= 1/Ka?

(A) Zero (B) First

(C) Second(D) Third

Explanation: For zero-order reactions, the half-life is inversely proportional to the rate constant.

20. What is the common name of 1,2,3-propanetriol?

(A) Butyl alcohol(B) Glycol

(C) Glycerol (D) Propyl alcohol

Explanation: Glycerol, also known as glycerine or 1,2,3-propanetriol, is a simple polyol compound. It is a colorless, odorless, viscous liquid that is sweet-tasting and non-toxic. Commonly used in pharmaceutical formulations, food industry, and as a moisturizer in skin care products, glycerol possesses hygroscopic properties, allowing it to retain moisture.

Blog Post on: How To Get MDCAT Past Papers For All Subjects

Leave a Reply